Está en la página 1de 121

Nivel I (Quinto Grado)

1. Gloria suma siete números y obtiene 2020. Uno de los números en la suma es 225. Ella sustituye
el número 215 por el 106. ¿Qué respuesta obtiene ella?
Solución:
Restamos a la suma de los números el número conocido: 2020 − 225 = 1795.
Al resultado obtenido en la operación anterior sumar el número por el que se sustituye el número
conocido: 1795 + 106 = 1901

2. Los 6 números naturales impares más pequeños se escriben en las caras de un dado. Toni lo tira
tres veces y suma los resultados. ¿Cuál de los siguientes números no puede ser la suma?
A) 21 B) 3 C) 20 D) 19 E) 29
Solución:
Los 6 números naturales impares más pequeños que se pueden escribir en las caras del dado
son: 1; 3; 5; 7; 9 y 11, la suma que no puede obtener con esos números es 20 porque por
propiedad de los números impares, la suma de tres impares siempre da como resultado un
número impar, además los otros números se pueden obtener por combinaciones de los números
de la siguiente manera:
3= 1+1+1
19 = 1 + 7 + 11 = 3 + 5 + 11
21 = 1 + 9 + 11 = 3 + 7 + 11 = 5 + 7 + 9
29 = 7 + 11 + 11
3. Laura quiere sombrear un cuadrado 2 × 2 de la figura adjunta. ¿Cuántas
posibilidades tiene?
Solución:
Las posibilidades que tiene de sombrear un cuadrado de 2 × 2 son 8 y se
muestran a continuación:

4. Tenemos tres cartulinas y en cada una se ha escrito un número de tres cifras.


Como se ve en la figura dos de las cifras están tapadas. La suma de los tres
números es 826. ¿Cuál es la suma de las dos cifras ocultas?
Solución:
Ubicando los números en la tabla de valores tenemos que:
C D U
2 4 3
1 7
+ 2 6
8 2 6
De la suma de las unidades tenemos 3 + 7 + 6 = 16, es decir ubicamos el 6 y va 1 a las decenas,
al sumar los dos numeros conocidos y el uno que llevamos tenemos: 4 + 2 + 1 = 7 para obtener
2 en el resultado entonces se debe colocar el número 5 en el espacio en blanco para obtener 12
y colocar el 2 en la casilla de las decenas y llevar 1 a las centenas, al sumar los dos números
conocidos de la centena más el 1 que llevamos tenemos: 2 + 1 + 1 = 4, entonces para obtener
8 nos falta 4 y ubicamos el 4 en el espacio en blanco de las centenas, por lo que la suma de los
números ocultos es: 4 + 5 = 9.
5. Un triángulo grande se divide en triángulos equiláteros, como se ve en la
figura. El lado del triángulo pequeño gris mide 1 m. ¿Cuál es el perímetro
del triángulo grande?
Solución:

Como el lado del triángulo pequeño mide 1 m, entonces el lado del


triángulo sombreado en negro es igual a 2 veces la medida del lado del
triángulo pequeño es decir mide 2 m y como el lado del triángulo grande
mide 2 veces la medida del lado del triángulo sombreado en negro más la
medida del lado del triángulo pequeño, entonces el lado que sirve de base
del triángulo grande mide:1𝑚 + 2(2𝑚) = 5𝑚

Los lados del triángulo sombreado en café miden lo mismo y es igual a


la suma de la medida del lado del triángulo sombreado en gris con la del
triángulo sombreado en negro, es decir: 1𝑚 + 2𝑚 = 3𝑚. Por lo tanto la
medida de los lados diagonales del triángulo grande miden
3𝑚 + 2𝑚 = 5𝑚, por lo que podemos concluir que el triangulo grande es
equilátero y su perímetro es igual a 3𝑙 = 3(5𝑚) = 15𝑚
6. En el jardín de una bruja hay 30 animales: perros, gatos y ratones. La bruja convierte 6 perros
en gatos. Luego convierte 5 gatos en ratones. Ahora su jardín tiene la misma cantidad de perros,
gatos y ratones. ¿Cuántos gatos había al principio?
Solución:
Por la información brindada en el problema la única forma es que después de convertir 6 perros
en gatos, y 5 gatos en ratones haya la misma cantidad de animales es que de todos haya 10
animales entonces al haber 10, significa que la cantidad inicial de perros era 10 + 6 = 16, la
cantidad de gatos era 10 + 5 − 6 = 9 y la de ratones era 10 − 5 = 5, entonces la cantidad
inicial de gatos es 9.
7. Con bloques de dimensiones 1 cm × 1 cm × 2 cm, se pueden construir torres como las que se
muestran en la imagen de abajo. ¿Qué altura tiene una torre que se construye de la misma manera
con 28 bloques?

Solución:
Si continua el patrón la figura que sigue tendrá 6 bloques en la base, 5 acostados 4, después
parados, 3 acostados, 2 parados y 1 acostado para un total de 21 bloques, esta torre tiene de
altura 9 cm, la siguiente figura tiene 7 bloques en la base, 6 acostados 5, después parados, 4
acostados, 3 parados, 2 acostado y 1 parado, para un total de 28 bloques, esta torre tiene de altura
11 cm.
8. ¿Qué fracción del rectángulo es la zona sombreada?
Solución:
1 1 1
La región sombreada corresponde a 3 𝑑𝑒 del rectángulo grande. Por tanto, La respuesta es 3 ∙
4
1 1
= 12
4

9. Distribuir en los círculos mostrados en la figura de la derecha los números de


tres dígitos 111, 112, 121, 122, 211, 212, 221, 222, sin repeticiones, de modo
que los números escritos en círculos que están unidos entre sí por un segmento
no tengan más de una coincidencia (es decir, pueden tener exactamente una
coincidencia o no tener coincidencias).
Solución:
Que no tenga más de una coincidencia, se refiere a que en las cifras de las cifra de las unidades,
decenas y centenas, puede haber como mucho una sola de ellas repetida. Cada círculo tiene tres
círculos vecinos. Se pueden separar los números en dos grupos.

10. Cambia de lugar 1 de los 13 palillos y haz que quede formada una igualdad verdadera, porque
5≠ 2+5

Solución:
Observar que 2+8=10
Entonces mover uno de los palillos de la izquierda del igual para formar una X:
𝑋 = 𝐼𝐼 + 𝑉𝐼𝐼𝐼
11. Jimena dibujó un triángulo con longitudes 6, 10 y 11. Carlos dibujó un triángulo equilátero con
el mismo perímetro. ¿Cuánto mide cada uno de los lados del triángulo que dibujó Carlos?

Solución:

El perímetro del triángulo de Jimena es 6 + 10 + 11 = 27, así que cada uno de los lados del
27
triángulo equilátero mide 3 = 9.

12. En la figura se muestra un cubo de cartón, desdoblado. Hansel sumó


correctamente los números en las caras opuestas del cubo. ¿Cuáles son los
resultados que obtuvo Hansel?
Solución:

Al armar el cubo las parejas de lados opuestos son {1,3}, {2.4}, {5,6}. Así que las sumas que
obtiene Hansel son 4, 6 y 11.

13. Dos láminas cuadradas transparentes tienen algunos cuadrados opacos como se muestra en la
figura. Se sobreponen en la cuadricula de la derecha. ¿Cuál de las figuras queda visible?

Solución:
La lámina de la izquierda tapa al perro y al reloj. La de la derecha tapa la taza y el pájaro. La
mariposa queda visible.
14. Cada una de las llaves abre solo uno de los candados. Los números de las llaves corresponden
a las letras de los candados. ¿Qué numero está escrito en la llave con el signo de interrogación?

Solución:

Observemos las cantidades de cada letra y de cada número: De las letras hay:
𝐷 = 5, 𝐴 = 4, 𝐵 = 3 𝑦 𝐻 = 3. De los números hay cuatro 1’s, cuatro 4’s, dos 2’s y dos 8’s.
Entonces deducimos que alguno de 1 o 4 corresponde a D y que 2 y 8 corresponden a B y H, en
algún orden. Tenemos entonces que la llave corresponde al único candado que no tiene A, y éste
es BHD. Como DAD es el único que tiene letra repetida, debe corresponder a la llave 414 y así
ya sabemos que D corresponde a 4, y A corresponde a 1. Ahora observamos que HAB debe
corresponder a 218 o a 812, pero 218 no es opción, así que se deben corresponder H con 8, y B
con 2.
15. Un grupo de niñas están en un círculo. Gaby es la cuarta a la izquierda de Vero y la séptima a
la derecha de Vero. ¿Cuántas niñas hay en el grupo?
Solución:
Entre Vero y Gaby hay 3 niñas de un lado y hay 6 niñas del otro, suman 9 más Vero y Gaby
para un total de 11.
16. La figura muestra el calendario de cierto mes del año.
Desafortunadamente le cayó tinta encima. ¿En qué día de la
semana cayó el 27 de ese mes?
Solución:
Como el jueves es 2, también caen en jueves los días 2 + 7 = 9, 2 + 14 = 16, 2 + 21 = 23 y
2 + 28 = 30. El 27 cae en lunes.
17. A una competencia se inscribieron inicialmente 19 hombres y 11 mujeres. Deben formarse 8
equipos de tal forma que cada equipo tenga el mismo número de personas y además cada equipo
debe tener el mismo número de hombres que de mujeres. ¿Cuántas personas deben inscribirse
al club, como mínimo, para que eso sea posible?

Solución:
Para lograr en un principio el mismo número de hombres que de mujeres hacen falta 8 mujeres.
Así habrá 38 personas. Como debe haber 8 equipos, el número de personas debe ser múltiplo de
8, pero 40/8 = 5 así que no podrá haber el mismo número de hombres que de mujeres. Entonces
deben juntarse 48 personas y así en cada uno de los 8 equipos quedarán 3 hombres y 3 mujeres.
Entonces faltan 48 - (19 + 11) = 18 personas (5 hombres y 13 mujeres).
18. Marcela tiene 20 pesos. Cada una de sus cuatro hermanas tiene 10 pesos. ¿Cuántos pesos tiene
que darle Marcela a cada una de sus hermanas para que las cinco tengan la misma cantidad?
Solución:
Entre las cuatro hermanas y Marcela tienen 60 córdobas en total.
Al dividir este total entre las cinco, le correspondería 12 córdobas a cada una.
Entonces Marcela tendrá que dar a cada hermana 2 córdobas.

19. De cuántas formas se pueden colorear tres cuadritos de la cuadrícula, ver figura 1. De modo que
en cada fila y en cada columna haya un único cuadrito coloreado.
Solución:
Numeremos los cuadritos, para orientarnos. Pintemos el 1, después el 5,
y por último 9. Ahora coloreamos 1, después el 6, y de último el número
8. Esto nos da una configuración valedera de dos formas, para el número
1, o sea para colorear el primer cuadrito
Continuemos ahora con otra posición. Tomemos el número 2; continuemos con el número 4 y
terminemos con el número 9; luego 2, después el número 6 y terminamos con el número 7. Eso
nos da otra configuración, también valedera de dos formas para el número 2, o sea para colorear
el segundo cuadrito.
Continuando de esa manera con el número 3, 4, hasta el 9, podemos concluir que existen
18 formas de colorear la cuadrícula.
20. De la ciudad A a la ciudad B hay 3 caminos, de la ciudad A a la ciudad C hay 5 caminos, de la
ciudad B a la D hay 2 caminos y de la ciudad C a la D hay dos caminos. Si un camino que une
dos ciudades no pasa por otra, ¿cuántas formas hay de ir de la ciudad A a la D?
Solución:
Método 1.
Mostremos las ciudades encerradas en círculos y los caminos
rotulados, con números y letras, como se muestra en la figura 1.
Entonces, para ir desde la ciudad A hasta la ciudad D, pasando por
la ciudad B, tomaríamos los siguientes caminos:
A 2a 2b D
4a 4b
6a 6b
Tabla 1. Hay seis formas de ir de A a D
pasando por B.
A 1c 1d D
3c 3d
5c 5d
7c 7d
9c 9d
Tabla 2. Hay 10 formas de ir de A a D pasando por la ciudad
C.
En total 16 formas de desde la ciudad A la D.
Método 2.
Primero. Aplicando el principio multiplicativo, tenemos para desde la ciudad A a la ciudad B,
hay 3 formas; luego hay 2 formas de ir de la ciudad B a la ciudad D, por tanto, aplicando el
principio, existen 3x2 = 6 formas de la ciudad A la ciudad D.
Segundo. De la ciudad A a la ciudad C, hay 5 formas; y de la ciudad C a la ciudad D hay 2
formas, por tanto, aplicando el mismo principio, tenemos que hay 5x2 = 10 formas de ir desde
A hasta D.
Finalmente, aplicando el principio aditivo o sumativo, existen 6 + 10 = 16 formas de ir de la
ciudad A hasta la ciudad D.
21. En la mitología Griega, a los supuestos habitantes de la Luna se les conoce como Selenitas.
Ellos son seres muy inteligentes, pero tienen un alfabeto muy elemental que consiste de las dos
letras A y L. El nombre de cada Selenita es una secuencia de 5 o menos de esas letras. Por
ejemplo, su presidente se llama ALAL. Si todos los Selenitas tienen nombres distintos, y ya no
hay más nombres disponibles, ¿cuántos Selenitas hay en la Luna?
Solución:
En total de todos los Selenitas serán las distintas combinaciones y secuencias de las letras A y
L, comenzando con las de longitud 1, de longitud 2, …, hasta las de longitud 5.

Así, de longitud 1, hay 2: A y L; de longitud 2 hay 4: AA, AL, LA LL; de longitud 3 hay 8; de
longitud 4, hay 16; y de longitud 5 hay: 32.

Así que, total hay 2 + 4 + 8 + 16 + 32 = 62 Selenitas en la Luna.


22. Mario escribió un dígito y después escribió otro a la derecha; posteriormente sumo 19 al número
de dos dígitos que tenía y obtuvo 72. ¿Cuál es el primer dígito que escribió Mario?
Solución:
Resolviendo de adelante hacia atrás tenemos si al 72 le resto los 19 que sumo originalmente
tenemos que 72 − 19 = 53, el resultado es el número de 2 dígitos escrito inicialmente y el
primer digito que escribió Mario fue el 5.
23. El profesor escribió en la pizarra los números 1, 2, 3, 4, 5, 6, 7, 8, 9 y 10. Un alumno borra
algunos números de tal forma que entre los números que quedaron escritos en la pizarra no hay
un número que sea igual al doble del otro. ¿Cómo máximo cuántos números pueden quedar
escritos en la pizarra?
Solución:
Si no deben quedar números que sean el doble de otro, el 2 no debe estar por ser el doble de 1,
el 4 por ser el doble de 2, el 6 por ser el doble de 3, el 8 por ser el doble de 4, el 10 por ser el
doble de 5, entonces los números que quedaron en la pizarra son: 1, 3, 5, 7 y 9 que en total son
5 números como máximo
24. José es el encargado de la limpieza de un edificio. Él trabaja todos los días impares del mes (por
ejemplo, trabaja el 7 de mayo, 11 de agosto, 23 de diciembre, etc.) y, además, trabaja todos los
martes y todos los sábados. ¿Cómo máximo cuántos días consecutivos puede trabajar José?
Solución:
Se trata que José trabaje la mayor cantidad de días, como trabaja los días impares, entonces
busquemos días consecutivos impares y esto ocurre cuando el mes tiene 31 días, porque luego
sigue el día 1 del mes siguiente. Además, los martes y sábados deben ser días pares, es decir:

Por lo tanto, como máximo José puede trabajar 6 días consecutivos.

25. Entre seis amigos compran 16 caramelos. Ángel compra un caramelo, Bryan compra dos y
Javier compra tres. Carlos Gil compra tantos caramelos como su hermano, Pablo Pérez compra
el doble de su hermano y Enrique Martínez el triple de su hermano. ¿Cuál es el apellido de
Ángel?
Solución:
Dado que en total son 16 caramelos y son 6 amigos. Hacemos la siguiente tabla para ordenar la
información. Para que se complete los 16 caramelos y se cumplan las condiciones del ejercicio
la distribución para los nombres con apellidos será la siguiente.
Nombre Caramelo Nombre y Apellido Caramelo Total
Ángel 1 Enrique Martínez Triple de Ángel (3 caramelos)
Bryan 2 Pablo Pérez Doble que Bryan (4 caramelos)
Javier 3 Carlos Gil Lo mismo del hermano (3 16
caramelos)
Total 6 10
Por tanto, el apellido de Ángel es Martínez.
26. En un edificio se enumeraron todas las puertas de las oficinas utilizando placas que contenían
un digito cada una. Por ejemplo, para numerar la décimo cuarta puerta se utilizaron dos placas,
una con el número 1 y otra con el 4. Si en total se utilizaron 53 placas, determine la cantidad de
puertas numeradas.
Solución:
Las puertas del 1 al 9 usan una placa cada una, de donde 53 − 9 = 44 placas son usadas para
formar números de dos dígitos, los cuales usan dos placas cada uno. Luego hay 44 ÷ 2 = 22
números de dos dígitos, y así hay 9 + 22 = 31 puertas en total.
27. Julián, María, Nicolás y Luisa tienen una mascota cada uno, entre las siguientes: un gato, un
perro, un pez y un canario. La mascota de María tiene pelo, la de Luisa cuatro patas, la de
Nicolás es un ave y se sabe que a Julián y a María no les gustan los gatos. Determine que mascota
tiene cada uno.
Solución:
Como la mascota de Nicolás es un ave, esta debe ser el canario. La mascota de María tiene pelo,
así que no puede ser ni pez ni canario, ni tampoco gato ya que a ella no le gustan; se sigue que
María tiene un perro. Y como la mascota de Luis tiene cuatro patas y no es perro (que es de
María), esta debe ser un gato. La mascota restante, el pez, es de Julián

28. En la figura se ven cinco rectángulos, en los que cada lado está etiquetado con un número entero.
Estos rectángulos se colocan, sin rotarlos ni darles la vuelta, en las posiciones I a V que se
muestran abajo, de forma que las etiquetas en los lados que se tocan deben ser iguales.
Determine qué rectángulo debe colocarse en la posición I. Argumente por qué esta es la única
posibilidad.

Solución:
Veamos cuales fichas pueden ocupar la posición I. Si colocamos a B, la ficha en II debe tener
un 5 a la izquierda; al colocar D obligamos a la ficha en IV a tener un 6 arriba; y asimismo al
colocar E vemos que la ficha en II debe tener un 6 a la izquierda. Ya que no existen tales fichas,
se sigue que sólo A o C pueden colocarse en I. Pero si A está en I, entonces la ficha en II debe
tener un 4 a la izquierda, y por ende debe ser E; sin embargo, no hay fichas con un 6 a la izquierda
que puedan colocarse en III, una contradicción. Por tanto C debe ocupar la posición I, y a partir
de esto fácilmente determinamos que II, III, IV y V están ocupados respectivamente por A, E,
D y B.
29. Cien niñas se disponen a jugar pin-pon; para ello se sientan en sillas numeradas del 1 al 100. El
juego tiene las siguientes reglas: si una niña está sentada en una posición par, ella dice pin, si
está sentada en una silla con número múltiplo de 3 ella dice pon, en otro caso dice el número de
su silla. Así por ejemplo la primera niña dice uno, la segunda dice pin, la tercera dice pon, la
cuarta dice pin, la quinta dice cinco, la sexta dice pin-pon, y continúan de esa manera. Determine
la cantidad de niñas que dicen solamente pin, la cantidad que dicen solamente pon, la cantidad
que dicen pin-pon y la cantidad que dicen un número.
Solución:
Entre los números del 1 al 100 hay 50 números pares, 33 múltiplos de 3, y 16 múltiplos de 3
pares (es decir, múltiplos de 6). Ya que las niñas en asientos divisibles por 6 dicen pin-pon,
vemos que 50 − 16 = 34 de ellas dicen sólo pin, 33 − 16 = 17 sólo dicen pon y 16 dicen pin-
pon. Por último, para hallar el número de niñas que dicen un número restamos del total de niñas
el número de ellas que dicen pin o pon, obteniendo 100 − 34 − 17 − 16 = 33 niñas.

30. Daniela participa en un juego que consiste en sacar 40 bolitas de una caja. Las bolitas son
blancas o negras. Las reglas dicen que ella recibirá C$ 5 por cada bolita blanca que saque y
deberá pagar C$ 10 por cada bolita negra que saque. Si al final del juego ella ganó C$ 20.
Determine la cantidad de bolitas blancas que Daniela sacó.
Solución:
Si Daniela ganó C$ 20 en total, quiere decir que ella sacó 4 bolitas blancas, con las que ganó los
C$ 20, y de que las 36 bolitas restantes no obtienen ganancia alguna. Según los datos del
problema, 2 bolitas blancas se equilibran con 1 bolita negra, por lo que entre las 36 bolitas en
cuestión el número de bolitas blancas debe ser igual al doble del número de bolitas negras, es
decir estos números son respectivamente 24 y 12. Por tanto la cantidad total de bolitas blancas
que sacó Daniela es 24 + 4 = 28.
31. Carolina y Alejandro hacen fila para comprar palomitas e ingresar al estreno de su película
favorita en el Cine. En determinado momento, Carolina observa que delante de ella hay siete
personas y Alejandro sabe que hay 20 personas en total haciendo fila. Si Alejandro está justo
delante de Carolina en la fila, ¿Cuál es la cantidad de personas haciendo fila que están entre
Alejandro y el último de la fila?
Solución:
Carolina está detrás de Alejandro, por lo que, contando a Carolina, después de Alejandro hay
13 personas. Así, entre Alejandro y el último hay 12 personas.
32. Los cuatro clasificados a la Olimpiada Matemática de Centroamérica y el Caribe se han comido
11 manzanas en total. Cada uno de ellos se ha comido, al menos, una manzana y ninguno ha
comido la misma cantidad de manzanas que otro de ellos. Tres de ellos han comido nueve
manzanas en total y uno de estos se ha comido exactamente tres manzanas. ¿Cuál es la cantidad
de manzanas comidas por el Olímpico que más manzanas comió?
Solución:
Al mencionar que tres de ellos han comido nueve en total, se deduce que el otro comió dos. De
estos tres que comieron nueve en total, uno comió tres, así que los otros dos comieron seis. De
esas seis manzanas para los dos Olímpicos restantes, solo se puede deducir que uno comió cinco
y el otro una (no es posible 6 + 0, ni 4 + 2 ni 3 + 3).

33. El cubo que se muestra en la figura tiene un entero positivo escrito en cada
cara. Los productos de los dos números en caras opuestas son iguales. ¿Cuál
es la suma más pequeña posible de los seis números del cubo?
Solución:
Sea 𝑥 el número de la cara opuesta a 5, 𝑦 el número de la cara opuesta a 10 y 𝑧 el dela cara
opuesta con 15, de la información brindada en el problema tenemos que:
𝑥 ∙ 5 = 𝑦 ∙ 10 = 𝑧 ∙ 15
El mínimo común múltiplo de 5, 10 y 15, es el menor producto que se puede obtener al
multiplicar dos caras opuestas, el cual es: m.c.m (5; 10; 15) = 30, entonces:
𝑥 ∙ 5 = 30 → 𝑥 = 6
𝑦 ∙ 10 = 30 → 𝑦 = 3
𝑧 ∙ 15 = 30 → 𝑧 = 2
Por lo tanto la suma más pequeña de las caras es: 2 + 3 + 5 + 6 + 10 + 15 = 41
34. Benjamín escribe un número entero positivo en el primer círculo y luego rellena los otros cinco
círculos siguiendo las instrucciones que se ven sobre las flechas. ¿Cuántos de los números en
los círculos son divisibles por 3?

Solución:
Iniciemos rellenando los círculos desde el 1 y así sucesivamente, de la siguiente manera:

Podemos observar que sin importar los números que se ubiquen en los círculos siempre van a
haber solo 2 números que son divisibles por 3.
Por lo tanto, solo dos números son divisibles por 3.
35. Julio y Mateo están lanzando bolas a dos “pirámides” idénticas de 15 latas. Julio derriba 6 latas
y obtiene un total de 25 puntos. Mateo derriba 4 latas. ¿Cuántos puntos anota Mateo?

Solución:
Como las pirámides son idénticas, las latas que derribo Julio de abajo hacia arriba tienen los
siguientes números: de la base la numero 3, de la segunda línea las latas numeradas con 8 y 2,
en la tercera fila la numero 3 y en la primer fila la numero 4, si sumamos estos números
tendremos: 3 + 8 + 2 + 3 + 4 = 20, pero hasta aquí solo tenemos 5 latas y eran 6 es decir que
hay una más ubicada arriba para terminar la pirámide y como la suma de las 6 es 25 entonces
25 − 20 = 5, que es el número que tiene la sexta lata que derribo, Julio.

De lo anterior tenemos que las latas que derribo Mateo de abajo hacia arriba fueron: la número
8 en la segunda fila, 4 en la tercera fila, 9 en la cuarta fila y la 5 en la última fila, al sumar los
números obtendremos la cantidad de puntos que obtuvo Mateo: 8 + 4 + 9 + 5 = 26.
Por lo tanto Mateo obtuvo un total de 26 puntos
36. En la figura se muestra un tablero de 4 × 4 donde se deben escribir los números 1, 2, 3,4 de
manera que aparezcan todos en cada fila y cada columna. El tablero de 4 × 4 ha sido dividido
en cuatro tableros más pequeños de 2 × 2 que también deben contener los números 1, 2, 3, 4.
¿Cuál es el dígito que debe estar en P?

Solución:
En el tablero superior izquierdo de 2 × 2 deben estar los números del 1 al 4, por tanto la única
posibilidad es:

En el tablero de 4 × 4, en la primera fila podemos ubicar el 2 y además en la primera columna


deben estar los números del 1 al 4, por tanto la única posibilidad es:
En el tablero de 4 × 4, en la cuarta columna deben estar los números del 1 al 4, por tanto la
única posibilidad es:

Finalmente el único valor que debe tener P es 1.


37. La figura muestra el plano de la casa de Renata, que entra en su casa
desde el porche y atraviesa cada puerta exactamente una vez. ¿En qué
habitación termina?
Solución:
Como entra por el porche primero atraviesa la puerta que lo lleva a la habitación 5, luego pasa
a la habitación 1, posteriormente a la 2, luego a la habitación 4 por la primera puerta y sale a la
habitación 3 por la segunda puerta para después pasar a la habitación 2.

También pudo haber entrado por la habitación 2 a la 3 para después entrar a la 4 y volver a salir
a la habitación 2.

Por lo tanto, Renata termina en la habitación 2.


38. Thor tiene siete piedras y un martillo. Cada vez que golpea una piedra con el martillo obtiene
cinco piedras más pequeñas. Hace esto varias veces. ¿Cuántas veces debe de golpear una piedra
con el martillo si quiere tener 23 piedras en total?
Solución:
Si golpea una piedra obtiene 5 pequeñas, más las 6 grandes que le quedan tendría en total
11 ≠ 23.
Si golpea dos piedras obtiene 10 pequeñas, más las 5 grandes que le quedan tendría en total
15 ≠ 23.
Si golpea tres piedras obtiene 15 pequeñas, más las 4 grandes que le quedan tendría en total
19 ≠ 23.
Si golpea cuatro piedras obtiene 20 pequeñas, más las 3 grandes que le quedan tendría en total
23 = 23.
Por lo tanto, debe de repetir el proceso de golpear una piedra 4 veces.
39. La pieza que se muestra en la figura de la derecha está formada por 10
cubos pegados. Se sumerge en un bote de pintura que la cubre por
completo. ¿Cuántos de los cubos tienen pintadas exactamente cuatro de
sus caras?
Solución:
Al enumerar las caras como se muestra en la figura podemos observar
que solo las caras con los números 2, 3, 4, 5, 6, 7, 8 y 9 tienen 4 caras libres las cuales son las
que se pueden pintar.
Por lo tanto, 8 cubos son los que tienen pintadas exactamente cuatro de sus caras
40. Cada región del diagrama se comenzó a pintar con uno de cuatro
colores, según se indica con las letras en mayúsculas: rojo (R),
amarillo (A), negro (N) y gris (G). Sabiendo que dos regiones que se
tocan deben tener colores diferentes, ¿De qué color debe ir la región
marcada con X?
Solución:
Se debe de pintar como se indica en la figura:

Por lo tanto, el color que debe ir la región marcada con X es el rojo.


Nivel II (Sexto Grado)
1. Tania escogió dos números primos cuya suma es 18. Susana escogió dos números primos cuya
suma es 14. Si los cuatro números escogidos son distintos entre sí, calcule la diferencia entre el
mayor número que escogió Susana y el menor número que escogió Tania.
Solución:
Los primeros números primos son: 2; 3; 5; 7; 11; 13; 17…
Susana escogió dos números primos cuya suma es 14.
Hay dos posibilidades:
7 + 7 = 14. Queda descartado porque son números iguales.
3 + 11 = 14. Por tanto, Susana escogió 3 y 11.
Tania escogió dos números primos cuya suma es 18.
Hay dos posibilidades:
7 + 11 = 18. Queda descartado porque Susana escogió 11
5 + 13 = 18. Por tanto. Tania escogió 5 y 13.
La diferencia entre el mayor y menor número que escogió Susana y Tania es: 11 – 5 = 6
2. Ordena los números del 1 al 9 en los círculos (sin que haya repeticiones) de tal forma que cada
flecha signifique "mayor que". En otras palabras, si hay una fecha que sale del número a y va
en dirección del número b, entonces a > b.

¿Cuál es la suma de los números que deben ir en los círculos sombreados?


Solución:
En el círculo ubicado en la parte superior derecha, debe ir el número mayor (9) ya que salen dos
fechas.
En el círculo ubicado en la parte inferior derecha, debe ir el número menor (1) ya que ingresan
dos fechas.

Ordenando los números convenientemente se tiene:

La suma de los números ubicados en los círculos sombreados es: 6 + 3 = 9


3. Carlos tiene cuatro tarjetas (llamadas P, Q, R y S) y cada una contiene un número de dos dígitos:

Carlos quiere obtener un número par de 8 dígitos y que sea lo mayor posible, ¿En qué orden
debe ubicar las tarjetas?
Solución:
Para obtener un número par y que sea lo mayor posible, la tarjeta R tiene que ir en el extremo
derecho y la tarjeta S en el extremo izquierdo, seguidamente la tarjeta Q y P, formando el
número: 96 55 23 36.
Por lo tanto, Carlos debe ubicar las tarjetas en el siguiente orden: SQPR.
4. Cada casilla de un tablero de 8 × 11 se va a pintar de rojo, verde o azul, de tal forma que cada
subtablero de 2 × 2 tenga al menos una casilla de cada uno de los tres colores. ¿Cuántas casillas
rojas puede haber como máximo?

Solución:
Encontrando un patrón con las condiciones del problema:

Casillas de color rojo: 8 × 6 = 48.


Casillas de color azul: 5 × 4 = 20.
Casillas de color verde: 5 × 4 = 20.
Por lo tanto, como máximo se puede contar con 48 casillas de color rojo.
5. Dani escribió todos los números pares entre 1 y 119. ¿Cuántas veces escribió el dígito 1?
¿Cuánto vale la suma de todos los dígitos que escribió Dani? Explica tus respuestas.
Solución:
Entre el 1 y el 10 escribió el 1 solo una vez, entre el 11 y el 20 escribió el uno 4 veces, entre el
21 y el 99 escribió el uno ninguna vez, entre el 100 y el 109, lo escribió 5 veces y del 110 al 119
lo escribió 10 veces, en total Dani escribió 20 veces el dígito 1. La suma de todos los dígitos
que escribió Dani se calcula de la siguiente manera:
De los dígitos: 2 + 4 + 6 + 8 = 20
De los múltiplos de 10: 1 + 2 + 3 + 4 + 5 + 6 + 7 + 8 + 9 + 1 + 2 = 48
Los pares del 21 al 29: 4 + 6 + 8 + 10 = 28
Los pares del 31 al 39: 5 + 7 + 9 + 11 = 32
Los pares del 41 al 49: 6 + 8 + 10 + 12 = 36
Los pares del 51 al 59: 7 + 9 + 11 + 13 = 40
Los pares del 61 al 69: 8 + 10 + 12 + 14 = 44
Los pares del 71 al 79: 9 + 11 + 13 + 15 = 48
Los pares del 81 al 89: 10 + 12 + 14 + 16 = 52
Los pares del 91 al 99: 11 + 13 + 15 + 17 = 56
Los pares del 101 al 109: 3 + 5 + 7 + 9 = 24
Los pares del 111 al 119: 4 + 6 + 8 + 10 = 28
Por lo tanto la suma de todos los dígitos que escribió Dani es:
20 + 48 + 28 + 32 + 36 + 40 + 44 + 48 + 52 + 56 + 24 + 28 = 456
6. Sea 𝐴 = 𝑥 ∙ 𝐵 − 4. Se sabe que si 𝐵 = 9 entonces 𝐴 = 18. Si 𝐵 = 27, ¿Cuánto vale A?
Solución:
18+4 22
Sabemos que 18 = 𝑥 ∙ 9 − 4 de donde 𝑥 = 9 = 9 .
22
Entonces, si B = 27, tenemos que 𝐴 = ∙ 27 − 4 = 62
9

7. Si 𝑛 = 4725 ∙ 𝑎, ¿Cuál es el menor entero positivo 𝑎 de modo que 𝑛 sea un cubo perfecto
múltiplo de 2?
Solución:
Sabemos que (n = 4725.a = 52.33.7.a. Para que 𝑛 sea un cubo perfecto es necesario multiplicar
por 5.72, pero como también debe ser múltiplo de 2, entonces debemos agregar 23. Por lo tanto,
a = 5.72.23 = 1960).
8. El rectángulo ABCD tiene perímetro 4 m y está dividido en 4 regiones por 4 segmentos
horizontales y 5 segmentos verticales. Si la suma de las longitudes de los segmentos verticales
es igual a 44 cm, Calcule la suma de las longitudes de los segmentos horizontales.
Solución:
Dado que 1m = 100 cm, entonces 4m = 4(100 cm) = 400 cm que es el perímetro del rectángulo
ABCD.

Además la longitud de los cinco segmentos verticales equivale a la longitud de la altura del
rectángulo es decir ℎ = 44 𝑐𝑚. De la fórmula del perímetro del rectángulo tenemos que:

𝑃 = 2𝑏 + 2ℎ
400 𝑐𝑚 = 2𝑏 + 2(44 𝑐𝑚)
2𝑏 = 400 𝑐𝑚 − 88𝑐𝑚
312 𝑐𝑚
𝑏= = 156 𝑐𝑚
2
La longitud de la base del rectángulo equivale a la longitud de los 4 segmentos horizontales, lo
que es igual a 156 cm.

9. Utilizando cuatro rectángulos idénticos se forma un rectángulo mayor, como se muestra en la


figura. La longitud del lado más pequeño del rectángulo mayor es 10 cm. ¿Cuál es el perímetro
del rectángulo mayor?

Solución:
Según la figura el lado menor de uno de los rectángulos es la mitad del lado mayor del
rectángulo, por tanto, mide 5 cm.
Luego, el lado mayor del rectángulo mayor es: 5 + 10 + 5 = 20 𝑐𝑚.
El perímetro del rectángulo mayor es:2(𝑎 + 𝑏) = 2(10 + 20) = 60 𝑐𝑚.
10. Cuatro primas, Ema, Iva, Rita y Zina, tienen las edades de 3, 8,12 y 14 años, pero no
necesariamente en ese orden. La suma de las edades de Zina y Ema es divisible por 5. La suma
de las edades de Zina y Rita también es divisible por 5. ¿Cuántos años tiene Iva?
Solución:
Las únicas sumas que dan una cantidad divisible por 5 son 12 + 8 y 12 + 3, así Zina debe tener
12, Ema tiene 3, Rita 8 y, por lo tanto, Iva tiene 14.
11. En la función de teatro de hoy un sexto de la audiencia son niños. Dos quintos de los adultos
son hombres. ¿Qué fracción de la audiencia son mujeres adultas?
Solución:
La fracción de la audiencia que corresponde a adultos es:
1 5
1− =
6 6
2 5 2 10
De esta fracción, 5 son hombres, es decir, 6 × 5 = 30; por tanto:

25 10 15 1
− = =
30 30 30 2
1
La fracción de la audiencia corresponde a mujeres adultas es: .
2

12. Cada número en una lista se obtiene de la siguiente manera: los primeros dos números son 2 y
3; después cada número es la cifra de la derecha del número que se obtiene al multiplicar los
dos anteriores en la lista. (Por ejemplo, los primeros 5 números de la lista son: 2, 3, 6, 8, 8.)
¿Qué número aparece en la posición 2020 de la lista?
Solución:
La lista quedaría de la siguiente forma: 2, 3, 6, 8, 8, 4, 2,... Después de los dígitos 2 y 3, se repite
la secuencia de números 6, 8, 8, 4, 2, 8. Como los dos primeros no se repiten, obtendremos el
dígito que está 2018 posiciones después de la primera aparición del 6.
Notemos que 2015 = 6 x 336 + 2, así que el dígito en la posición 2020 de la lista es el 8.

13. Mauricio construyó con cubitos la figura que se muestra. Si quiere guardarla en una caja, ¿cuáles
son las medidas de la caja rectangular más pequeña en la que se puede guardar la figura?

Solución:
Hacia la derecha la medida es 5, hacia arriba es 3, hacia el fondo es 4.
La medida de la caja será 3 × 4 × 5
14. Martín quiere colorear los cuadrados del rectángulo de tal manera que una tercera parte de los
cuadrados sean azules, la mitad sean amarillos y el resto sean rojos. ¿Cuántos deben ser rojos?

Solución:
En total hay 18 cuadritos, de manera que 6 deben ser azules y 9 deben ser amarillos. Entonces
3 deben ser rojos.
15. El diagrama muestra un rectángulo de dimensiones 7 x 11 que contiene
dos circunferencias. Cada una de las circunferencias toca al rectángulo
en tres de sus lados. ¿Cuál es la distancia entre los centros de las
circunferencias?
Solución:
Dada una circunferencia, la distancia de su centro a cada
uno de los lados del rectángulo que toca es la medida de su
7
radio, que resulta ser 2
Entonces, la distancia entre los centros mide
7
11 − 2 × 2 = 4

16. El cuadrado 𝐴𝐵𝐶𝐷 fue dividido


en nueve cuadritos congruentes. Calcular el área de la región sombreada
si el lado del cuadrado mide 6𝑚.

Solución:

El área del cuadrado 𝐴𝐵𝐶𝐷 equivale a (6𝑚)2 = 36𝑚2 ,

Como ha sido dividido en nueve cuadritos congruentes significa que cada


36
cuadrito tiene un área de = 4𝑚2, de aquí que cada lado de los cuadritos
9
mide 2𝑚. El área de la región sombreada está compuesta por cuatro
triángulos rectángulos congruentes más un cuadrito. Para calcular el área
2×4
de uno de los triángulos tenemos: = 4𝑚2 . Por tanto, el área de la
2
región sombreada es: 4(á𝑟𝑒𝑎 𝑑𝑒𝑙 𝑡𝑟𝑖á𝑛𝑔𝑢𝑙𝑜) + 𝑎𝑟𝑒𝑎 𝑑𝑒𝑙 𝑐𝑢𝑎𝑑𝑟𝑖𝑡𝑜 =
4(4𝑚2 ) + 4𝑚2 = 20𝑚2

17. Un abogado invitó a 5 personas a una conferencia. Los nombres de las seis personas que se
reunieron alrededor de una mesa circular eran: Andrés, Luis, Guillermo, Carlos, Eduardo y
Marcos. Las profesiones de estos eran: médico, psicólogo, ingeniero,
sociólogo, profesor y abogado.

El profesor tenía discrepancias con Carlos, se sentó frente a Andrés;


el médico se sentó frente a Luis; Luis se sentó entre el sociólogo y el
profesor; Marcos se sentó a la derecha del ingeniero y frente al
abogado; el ingeniero se sentó frente a Eduardo junto al médico y a la izquierda del profesor,
¿Quién tenía discrepancias con Carlos?

Solución:
La manera de analizar el problema es a través de un gráfico, ubicando a cada uno de los invitados
según cumplan cada condición.

De lo que se puede concluir que Marcos es el profesor, por lo tanto es quien tiene discrepancias
con Carlos.

18. La operación A consiste en restar 10 y la operación B consiste en multiplicar por 4/5. A un


número se le aplicó la operación A y luego la operación B, de esta forma el resultado final fue
24. ¿Cuál hubiese sido el resultado final si las operaciones se realizan en el otro orden (primero
B y luego A)?
Solución:
Planteando los enunciados:
𝐴: – 10
4
𝐵: × 5
Sea “𝑥” el número, entonces:
4
(𝑥 − 10) = 24
5
4𝑥 – 40 = 24 × 5
4𝑥 = 120 + 40
4𝑥 = 160
𝑥 = 40
Realizando las operaciones en forma inversa, tenemos que:
4
40 × − 10
5
8 × 4 – 10
32 – 10
22
Si las operaciones se realizan en orden inverso, el resultado final es 22.
19. Determine el menor número natural N que satisface todas las siguientes condiciones:
 Existen dos dígitos adyacentes de N cuyo producto es 2.
 Existen dos dígitos adyacentes de N cuyo producto es 0.
 Existen dos dígitos adyacentes de N cuyo producto es 1.
 Existen dos dígitos adyacentes de N cuyo producto es 8.
Aclaración: dos dígitos son adyacentes si se encuentran uno al lado de otro.
Solución:
Existen dos dígitos adyacentes de N cuyo producto es 1. Por tanto, el número N empieza con
las cifras 1 y 1, porque el producto de ambos números es 1.
Existen dos dígitos adyacentes de N cuyo producto es 2. Por tanto, el número N debe poseer la
cifra 2 seguido del 1.
Existen dos dígitos adyacentes de N cuyo producto es 0. Por tanto, el número N debe poseer la
cifra 0 y ésta debe ir último porque si no tendríamos otro producto igual a cero.
Existen dos dígitos adyacentes de N cuyo producto es 8. Por tanto, el número N debe poseer la
cifra 4 seguido de la cifra 2 porque éstos al multiplicarse resulta 8.
Finalmente: N = 11240.
Por lo tanto, el menor número natural N que satisface todas las condiciones es 11240.
20. Carlos tiene cuadrados verdes de tamaño 1 × 1, cuadrados amarillos de 2 × 2 y cuadrados rojos
de 3 × 3. Él quiere crear un cuadrado usando estos cuadrados, en el cual aparezcan los tres
colores. ¿De cuántas formas se puede hacer ese cuadrado utilizando todos los colores?
Solución:
Aplicando un poco de lógica, el cuadrado tendría la siguiente forma.
De donde se puede apreciar que, se necesitan 4 verdes de 1 × 1, 3
amarillos de 2 × 2 y 1 rojo de 3 × 3, para un total de un cuadrado de
5 × 5.
Analizando la columna de la derecha, se pueden reacomodar de 3!/2! = 3 formas, tratando los
bloques verde – verde como si fuera uno solo bloque. La esquina superior izquierda tiene dos
formas, así tenemos 2 × 3 = 6 formas distintas. Finalmente, se puede hacer rotar el cuadrado
en cuatro posiciones diferentes.
Por tanto el número de formas distintas so 3 × 2 × 4 = 24.
21. Laura, Iván, Valeria y María quieren estar juntos en una foto. María y Laura son las mejores
amigas y quieren estar juntas. Iván quiere estar junto a Laura porque le gusta. ¿De cuántas
formas pueden acomodarse para la foto?
Solución:
Sean L = “Laura”, I = “Iván”, V = “Valeria” y M = “María”. Entonces María y Laura se puede
representar de la siguiente forma: ML o LM. Igualmente, IL o LI, representa a Iván junto a
Laura. Valeria puede estar en cualquier lugar.
Luego, MLI o ILM, representa a María junto a Laura y Laura junto a Iván, por tanto hay 4
formas que son las siguientes: VMLI o MLIV o VILM o ILMV.
22. Ana Claudia quiere escoger dos días de la semana para trotar; en esos días trotará cada semana.
Si no quiere trotar dos días consecutivos, ¿de cuántas maneras puede escoger los días?
Solución:
Rotulemos los días de la semana de la siguiente manera: L =” lunes”, M = “martes”,
W = “miércoles”, J = “jueves”, V = “viernes”, S = “sábado” y D = “domingo”.
Sin pérdida de generalidad, vamos asumir que comienza a trotar el primer día el L, el siguiente
día para trotar serían los días, W o J o V o S, menos el martes siguiente y el domingo anterior
al lunes, es decir tiene 4 opciones. De igual forma, si comienza el segundo día el M, el siguiente
día para trotar sería J o V o S o D, menos el día W, y el día lunes anterior al día M, teniendo
nuevamente 4 opciones para escoger el día para trotar.
De esa manera, por cada día que ella escoja en la semana, le quedan 4 opciones para el segundo
día, de tal forma que en los 7 días de la semana, habrá un total de 4x7 = 28 maneras de escoger
los días.

23. Considere el número de 46 dígitos de la forma


112223333444445555556666666777777778888888889999999999
Determine la menor cantidad de dígitos que deben intercambiarse o cambiarse para que el
número resultante sea divisible por 1, 2, 3, 4, 5 y 6?
Solución
a) Para que el número sea divisible por 5 debe terminar en 0 o 5, pero para que también sea
divisible por 2 debe terminar en par, por lo tanto, debe cambiarse el último 9 por un 0.
b) Por otro lado, para que sea divisible por 4, el número formado por los dos últimos dígitos
debe ser divisible por 4. Vemos que 90 no es divisible por 4, por lo que debe cambiarse el
siguiente 9 por un número par (0, 2, 6, 8).
c) Además, para que sea divisible por 3, la suma de los dígitos debe ser múltiplo de 3, y esto
será suficiente para que también sea divisible por 6 (pues será divisible por 2 y 3).
La suma de los dígitos del número original es:
2 + 6 + 12 + 20 + 30 + 42 + 56 + 72 + 90 = 330
Pero al cambiar el último y penúltimo 9 por un 0, la suma será: 321 y la cual siempre resulta
un múltiplo de 3.
112223333444445555556666666777777778888888889999999900
Por lo tanto de lo anterior podemos notar que solamente debemos cambiar dos dígitos.
24. En la lista de números 1, 2, . . . , 9999, los dígitos del 0 al 9 se reemplazan con las letras de la
𝐴 a la 𝐽, respectivamente. Por ejemplo, el número 501 se reemplaza por la cadena "𝐹𝐴𝐵" y 8243
se reemplaza por la cadena "𝐼𝐶𝐸𝐷". La lista resultante de 9999 cadenas se ordena
alfabéticamente. ¿Cuántas cadenas aparecen antes de que "𝐶𝐻𝐴𝐼" esté en esta lista?
Solución
Comencé haciendo una lista, como esta:
𝐴=0
𝐵=1
𝐶=2
𝐷=3
𝐸=4
𝐹=5
𝐺=6
𝐻=7
𝐼=8
𝐽=9
Podemos usar la lista para hacer coincidir la letra de 𝐶𝐻𝐴𝐼 con los números apropiados. Termina
como: 2708.
Entonces, restamos 1 de 2708, porque queremos encontrar los números que
vienen antes 2708, y para hacer esto, no podemos contar el número en sí mismo, por lo tanto,
lo restamos para encontrar los otros números. 2708 − 1 = 2707
25. ¿Es posible repartir los números 12 ; 22 ; 32 ; 42 ; 52 ; 62 𝑌 72 en dos grupos, de manera que la suma
de los números de cada grupo sea la misma?
(b) ¿Y para los números12 ; 22 ; 32 ; 42 ; 52 ; 62 ; 72 ; 82 𝑌 92 ?
Solución:
(a) Sí se puede. La suma de los 7 números es 140, luego la suma de cada grupo debe ser 70. En
el grupo donde está 72 = 49 se debe agregar números que sumen 21 para llegar a 70, lo cual se
logra con 1, 4 y 16. Por lo tanto los grupos son {12, 22, 42, 72} y {32, 52, 62}.
(b) No se puede porque la suma 12 + 22 + ⋯ + 92 tiene 5 sumandos impares y por lo tanto es
impar.

26. Un estudiante de la ASJT tiene cinco billetes de distintos valores. ¿Cuántas sumas diferentes de
dinero puede formar con los cinco billetes?
Solución:
Cada billete puede ser o no elegido. Puesto que cada billete está ligado a 2 caminos (ser o no ser
elegido), el número total será de 25 formas. Pero esto incluye el caso en que ningún billete sea
elegido.
Por tanto, el número de sumas de dinero pedidas es = 25 − 1 = 31.
Otra solución:
Se pueden elegir 1 billete de los 5; 2 billetes de los 5; … ; 5 billetes de los 5. Entonces el número
de sumas de dinero pedido es

5 5 5 5 5
( ) + ( ) + ( ) + ( ) + ( ) = 5 + 10 + 10 + 5 + 1 = 31.
1 2 3 4 5
Otra solución:
Otra solución sería construir un diagrama de árbol.
27. En la figura se muestra una letra L formada por 16 cuadraditos. Como se puede ver, la letra L
tiene 6 cuadraditos de alto, 4 cuadraditos de ancho y 2 cuadraditos de grosor:

Si queremos formar ahora una letra L similar, pero ahora de 30 cuadraditos de alto, 12
cuadraditos de ancho y 3 cuadraditos de grosor, ¿Cuántos cuadraditos necesitamos en total?
Solución:
Graficando la letra L con 30 cuadraditos de alto, 12 de ancho y 3 de grosor:

Para hallar el total de cuadraditos vamos a contar en dos partes y utilizaremos el principio de
multiplicación:
Total de cuadraditos = Región 1 + Región 2
Total de cuadraditos = 12(3) + (30 – 3)(3)
Total de cuadraditos = 36 + (27)(3)
Total de cuadraditos = 36 + 81
Total de cuadraditos = 117
En total se necesitan 117 cuadraditos.
28. Determine el menor número capicúa que es múltiplo de 16 y dé como respuesta la suma de sus
dígitos.
Aclaración: Un número capicúa es aquel que se lee igual de izquierda a derecha que de derecha
a izquierda. Por ejemplo, 11; 606 y 3773 son números capicúas.
Solución:
Probemos si dicho número capicúa tiene dos cifras:
16 × 1 = 16
16 × 2 = 32
16 × 3 = 48
16 × 4 = 64
16 × 5 = 80
16 × 6 = 96
Con los resultados mostrados podemos afirmar que los múltiplos de 16 terminan en cifra par: 0;
2; 4; 6 o 8.
̅̅̅̅̅
Probemos ahora si el número capicúa tiene tres cifras: 𝑎𝑏𝑎
𝑎 ≠ 1, porque los múltiplos de 16 terminan en cifra par, de manera que el valor de “𝑎” podría
ser: 𝑎 = 2.
Sigamos probando:
16 × 13 = 208
16 × 14 = 224
16 × 15 = 240
16 × 16 = 256
16 × 17 = 272
Sumando los dígitos del número capicúa encontrado: 2 + 7 + 2 = 11.
29. Se muestran dos rectas numéricas que tienen diferentes escalas y han sido dispuestas en paralelo.
Determine qué fracción corresponde al punto marcado con un signo de interrogación.

Solución:
Se tienen dos escalas de medida cuyas longitudes están en la misma proporción. Sea “x” la
medida de la segunda escala que corresponde a 7 unidades en la primera escala.
Tomando las equivalencias entre ambas escalas, podemos plantear:
11 − 2 7 − 2
=
15 − 3 𝑥 − 3
9 5
=
12 𝑥 − 3
3 5
=
4 𝑥−3
3(𝑥 − 3) = 20
3𝑥 − 9 = 20
3𝑥 = 20 + 9
29
𝑥=
3
29
Por lo tanto el punto marcado con el signo de interrogación le corresponde la fracción 3
30. ¿En qué columna aparecerá el número 2020?
A B C D E F G H I
1 2 3 4 5
9 8 7 6
10 11 12 13 14
18 17 16 15
19 20 21 ⋯ ⋯
Solución:
Podemos notar que en total hay 9 columnas en las cuales se ubican los números como se
muestran.
Se pueden ir probando los números para ver las posiciones que se encuentran, en las primeras
dos filas están los números del 1 al 9, si vemos la secuencia cada dos filas hay 9 números. Si
dividimos el número que se desea conocer la posición entre las 9 columnas se verá una secuencia
con los residuos como se muestra en la siguiente tabla.
Si el residuo es El número está en la columna
0 B
1 A
2 C
3 E
4 G
5 I
6 H
7 F
8 D
Se pide en que columna aparecerá el número 2020, entonces:
2020 = 224(9) + 𝟒
Por tanto, el 2020 aparecerá en la columna G.
31. El cuadrado siguiente tiene un área de 36 cm2 , en él se han sombrado
tres regiones como se indica en la figura. Si se sabe que el área total
sombreada es de 27 cm2 , ¿cuál es el valor de a + b + c + d?
Solución:
 Tracemos una diagonal al cuadrado para formar dos triángulos uno de base 𝑏 y otro de base 𝑐.
Dado que el cuadrado es de área36 𝑐𝑚2, entonces cada lado mide 6 𝑐𝑚.

 Todos los triángulos tienen la misma altura la cual corresponde al lado del cuadrado que es de
6 𝑐𝑚.
 El total de área sombreada es de 27 𝑐𝑚2, es decir:
𝑎ℎ 𝑏ℎ 𝑐ℎ 𝑑ℎ
+ + + = 27
2 2 2 2

(𝑎 + 𝑏 + 𝑐 + 𝑑) = 27
2
La altura es igual a 6 𝑐𝑚.
6
(𝑎 + 𝑏 + 𝑐 + 𝑑) = 27
2
3(𝑎 + 𝑏 + 𝑐 + 𝑑) = 27
(𝑎 + 𝑏 + 𝑐 + 𝑑) = 9
32. Los números naturales del 1 al 2020 se escriben en la tabla de
siete filas que se muestra parcialmente en la figura, siguiendo
el orden de la echa. Determine en qué fila y en qué columna
estará ubicado el número 2020.
Solución:
Notemos que los múltiplos de 7 ocupan posiciones especiales
en el tablero. En efecto, si 𝑛 es impar, entonces 7n se encuentra
en la casilla (G; n), mientras que si n es par entonces 7n ocupa
la casilla (𝐴; 𝑛). Ahora bien, 2020 = 7 ∙ 288 + 4 no es un múltiplo de 7, pero sí conocemos la
posición de 7 ∙ 288 = 2016, que por lo anterior está en (𝐺; 288). A partir de 2016 debemos
avanzar 1 casilla a la derecha y después 3 hacia arriba, por lo que 2020 está en la casilla
(𝐷; 289).
33. Se escriben los números enteros positivos desde el uno hasta el 2020, uno a continuación del
otro, sin espacios intermedios, formando una larga secuencia de dígitos:
12345678910111213 … 20162017201820192020
Determine la cantidad de dígitos que se escriben antes de que se escriban tres 7 seguidos.
Solución:
La primera vez que aparecen tres 7 seguidos, ocurre al escribir 77 y 78. Se debe contar entonces
la cantidad total de dígitos al escribir los números del 1 al 76. Del 1 al 9 hay 9 dígitos. Del 10 al
76 hay 67 ∙ 2 = 134 dígitos. En total hay 143 dígitos.
34. Un tren de pasajeros tiene mil vagones. Leo, Eric y Alex trabajan recogiendo tiquetes del tren;
Leo recoge los tiquetes de los vagones múltiplos de 15 y Alex recoge los tiquetes de los vagones
múltiplos de 6. Si los tres coinciden dos veces, la segunda vez en el vagón 780, ¿Cuántas veces
coinciden Alex y Erick, antes de que los tres coincidan por primera vez?
Solución:
Leo, Erick y Alex deben coincidir por primera vez en el vagón 390; es decir, 390 es el mínimo
común múltiplo de los vagones en los cuales recogen tiquetes. Como 390 = 2 ∙ 3 ∙ 5 ∙ 13,
entonces Erick recoge tiquetes en los vagones múltiplos de 13, es decir, Alex y Erick coinciden
en los vagones que son divisibles por 13 y por 6, cada 78 vagones, entonces el múltiplo de 78
más próximo a 390 es 312, de donde 312 ÷ 78 = 4. Por lo tanto, Alex y Erick coinciden 4 veces
antes de que coincidan los tres por primera vez.

35. Determine el número de parejas de números naturales para las cuales el mínimo común múltiplo
es igual a 2020.
Solución:
Note que la descomposición en números primos es 2020 = 22 ∙ 5 ∙ 101, ya que 2, 5 y 101 son
primos. Entonces las parejas buscadas son 2 y 1010, 2 y 2020, 4 y 505, 4 y 2020, 5 y 404, 5 y
2020, 10 y 202, 10 y 2020, 20 y 101, 20 y 2020, además, 1 y 2020, y finalmente 2020 y 2020 y
así la respuesta es 12.
36. En la figura adjunta, hay cuatro tarjetas con una frase distinta en cada
una por uno de sus lados. Del otro lado de cada una de las tarjetas se
escribe uno de los números 2, 5, 7 o 12 (uno en cada tarjeta), de manera
que el número escrito no corresponda con la frase de la tarjeta
respectiva. ¿Cuál es el número escrito detrás de la tarjeta con la frase
Mayor que 10?

Solución:
Como 2, 5 y 7 son primos, detrás de la tarjeta que dice Primo se coloca el número 12.
De los números 2, 5 y 7, como 5 y 7 son impares, detrás de la tarjeta que dice Impar se coloca
el número 2.
Quedan los números 5 y 7, así que detrás de la tarjeta que dice Divisible por 7 se coloca el
número5.
Por lo tanto, detrás de la tarjeta que dice Mayor que 10 se coloca el número 7.
37. En un montón hay 9 fichas. Los dos jugadores, Jorge y Andrea, pueden retirar 1, 3 ´o 4 fichas
alternadamente. Gana el que se lleva la ´ultima ficha. Si empieza el juego Jorge, uno de los dos
jugadores puede ganar siempre. ¿Quién es? Explica cómo lo hace.
Solución:
Andrea siempre podrá ganar el juego con la siguiente estrategia:

 Caso 1: Si Jorge quita del montón 1 ficha, Andrea debe sacar 1 ficha también, luego quedan 7
fichas en el montón, ahora Jorge puede sacar 1, 3 ´o 4 fichas:
 Si Jorge saca 1 ficha, Andrea debe sacar 4 ficha y quedan 2 fichas en el montón y Jorge
solo puede sacar 1 ficha, con lo cual Andrea gana al sacar la ´ultima ficha.
 Si Jorge saca 3 fichas, Andrea debe sacar 4 fichas y gana.
 Si Jorge saca 4 fichas, Andrea debe sacar 3 fichas y gana.
 Caso 2: Si Jorge quita del montón 3 fichas, Andrea debe sacar 4 fichas, luego quedan 2 fichas
en el montón y Jorge solo puede sacar 1 ficha, con lo cual Andrea gana al sacar la ´ultima
ficha.

 Caso 3: Si Jorge quita del montón 4 fichas, Andrea debe sacar 3 fichas, luego quedan 2 fichas
en el montón y Jorge solo puede sacar 1 ficha, con lo cual Andrea gana al sacar la ´ultima
ficha.
38. El cuadrado de la figura se rellena con los números 1, 2, 3, 4 y 5 de tal manera
que cada fila y cada columna contienen cada uno de ellos exactamente una
vez. Además, la suma de los números en cada una de las tres regiones con
bordes en negrita es igual. ¿Qué número está en la esquina superior derecha?
Solución:
Primero hay que obtener la suma de todos los números para así poder obtener, a cuánto debe de
ser igual la suma de cada región, como es un cuadrado de 5 × 5, significa que cada número
aparecerá 5 veces en el cuadrado, por tanto:

5(1) + 5(2) + 5(3) + 5(4) + 5(5) = 5 + 10 + 15 + 20 + 25 = 75


Entonces como está dividido en 3 regiones y la suma de todos los números en cada región es
igual y esta suma es 75 ÷ 3 = 25, entonces los números se deben ubicar de tal manera que se
obtenga esa suma.
Iniciando por la región donde aparece el 2 tenemos que esta se completara de la siguiente
manera:

Como podemos ver la suma de estos números da como resultado 25. Ahora hagamos lo mismo
con la región donde aparece el signo de interrogación.

Podemos ver que la suma de los números en la región superior derecha también da como
resultado 25. Ahora hagamos lo mismo con la región que nos falta:

Por lo tanto, el número está en la esquina superior derecha es el 3


39. Todos los enteros positivos de cuatro cifras que tienen los mismos dígitos que el número 2019
están escritos en la pizarra en orden creciente. ¿Cuál es la diferencia numérica más grande que
hay entre dos números vecinos en la pizarra?
Solución:
Determinemos, en orden creciente, todos los números de cuatro cifras que podemos formar con
los dígitos de 2019.
1029, 1092, 1209, 1290, 1902, 1920
2019, 2091, 2109, 2190, 2901, 2910
9012, 9021, 9102, 9120, 9201, 9210
Notemos que al pasar de 2910 a 9012 se produce la mayor diferencia numérica. Luego la
diferencia entre ellos es 6102.

40. En Cuatrolandia solo se usan los dígitos 1; 2; 3; 4. Jorge que vive en Cuatrolandia escribe
números que tienen cuatro cifras. En cada número que escribe usa solamente dos dígitos
distintos. ¿Cuántos números puede escribir Jorge? Explica como los contaste.
Solución:
Imaginemos que solo usa el 1 y el 2 para formar los números de cuatro cifras, por lo tanto, puede
formar los siguientes 6 números: 1122; 1221; 1212; 2121; 2112; 2211
Algo similar va a ocurrir si solo usa los números: 1 y 3; 1 y 4; 2 y 3; 2 y 4; 3 y 4; claramente por
cada una de esas parejas se pueden formar 6 nuevos números. Por lo tanto la cantidad de
números que puede escribir Jorge es 6 × 6 = 36.
Nivel III (Séptimo Grado)
1. María dibujó un triángulo rectángulo BC, recto en A. Luego, ubicó los
puntos P y Q, como se muestra en la figura, de tal forma que 𝐴𝑃 =
𝑄𝐶 = 4 y 𝐴𝑄 = 𝐵𝑃 = 6. ¿Qué porcentaje del área del triángulo ABC
representa el área de la región sombreada?
Solución:
𝐴𝐶 = 𝐴𝑄 + 𝑄𝐶 = 6 + 4 = 10
𝐴𝐵 = 𝐴𝑃 + 𝑃𝐵 = 4 + 6 = 10
𝐴𝐶 × 𝐴𝐵 10 × 10
𝐴∆𝐴𝐵𝐶 = = = 50
2 2
𝐴𝑄 × 𝐴𝑃 6 × 4
𝐴∆𝐴𝑃𝑄 = = = 12
2 2
𝐴𝑆 = 𝐴∆𝐴𝐵𝐶 − 𝐴∆𝐴𝑃𝑄 = 50 − 12 = 38

50 →→→ 100%
38 →→→ 𝑥
38 × 100
𝑥= = 38 × 2 = 76%
50
2. Sofía escribió un número de dos dígitos y luego insertó un dígito d en la parte central, con lo
cual obtuvo un número de tres dígitos. Si al hacer esto el número original aumentó en 340,
determine el valor de d.
Solución:
̅̅̅
Sea el número de dos dígitos que escribió Sofía: 𝑎𝑏
̅̅̅̅̅
Luego insertó “d” en la parte central: 𝑎𝑑𝑏

̅̅̅ + 340 = 𝑎𝑑𝑏


El número aumentó en 340: 𝑎𝑏 ̅̅̅̅̅
10𝑎 + 𝑏 + 340 = 100𝑎 + 10𝑑 + 𝑏
340 = 90𝑎 + 10𝑑
34 = 9𝑎 + 𝑑
Por tanteo tenemos que: a = 3, d = 7.
Por lo tanto el valor de d es 7
3. Un número natural N es llamado cuasi-divisible si al sumar 1 a cualquiera de sus dígitos
obtenemos un divisor de N. Por ejemplo, 102 es cuasi-divisible porque 1 + 1, 0 + 1 y 2 + 1 son
divisores de 102. Determine el mayor número cuasi-divisible que consta de cuatro dígitos
distintos y dé como respuesta la suma de los cuadrados de sus dígitos.
Solución:
Sea el número natural de cuatro dígitos: 𝑁 = ̅̅̅̅̅̅̅
𝑎𝑏𝑐𝑑
Para que 𝑁 sea el mayor posible, entonces 𝑎 = 9. Como son dígitos distintos 𝑏 = 8, entonces
probando los valores para 𝑐 𝑦 𝑑, se obtiene que 𝑐 = 1 𝑦 𝑑 = 0. Por tanto el número es 9810 y
la suma de los cuadrados de sus dígitos es: 92 + 82 + 12 + 02 = 81 + 64 + 1 + 0 = 146
4. Luis escogió algunos elementos del conjunto {2; 3; 4; 5; 8; 12; 15; 27} y Ernesto se quedó con
los números que sobraron. Se sabe que el producto de los números de Luis es igual al producto
de los números de Ernesto y, además, Luis no escogió el número 8. Calcule la suma de los
números de Ernesto.
Solución:
Dado el conjunto: {2; 3; 4; 5; 8; 12; 15; 27}
Descomponiendo los números compuestos: {2; 3; 4; 5; 4 × 2; 4 × 3; 5 × 3; 3 × 3 × 3}
Si Luis no escogió el número 8, entonces Ernesto escogió el número 8. Ernesto deberá escoger
el número 2, porque el producto sería 16 (8 × 2 = 16), en consecuencia para compensar Luis
deberá escoger 4 y 12, pero como 12 tiene factor 3, también debe elegir al número 15 y 3 así
compensar con el número que debe escoger Ernesto que es 27, porque éste número tiene como
factor 3 × 3 × 3 y finalmente Ernesto escogerá el número 5. Es decir:
Ernesto escogió: 8 × 2 × 27 × 5 = 2160
Luis escogió: 4 × 12 × 15 × 3 = 2160
Hallado la suma de los números que escogió Ernesto: 8 + 2 + 27 + 5 = 42.
5. Daysi dibujó un triángulo y al medir sus ángulos interiores, con la ayuda de un transportador, se
dio cuenta que estas medidas son proporcionales a los números 2; 3 y 7. ¿Qué tipo de triángulo
dibujó Daysi?
Solución:
De la información planteada en el problema, tenemos que:
∡𝐴 ∡𝐵 ∡𝐶
= = =𝑘
2 3 7
De lo anterior tenemos que:
∡𝐴 = 2𝑘; ∡𝐵 = 3𝑘 𝑦 ∡𝐶 = 7𝑘
De la suma de los ángulos interiores de un triángulo tenemos que:
2𝑘 + 3𝑘 + 7𝑘 = 180°
12𝑘 = 180°
𝑘 = 15°
Sustituyendo el valor de 𝑘, tenemos que:
∡𝐴 = 2𝑘 = 2(15°) = 30°
∡𝐵 = 3𝑘 = 3(15°) = 45°
∡𝐶 = 7𝑘 = 7(15°) = 105°
Como el ángulo C es obtuso, por lo tanto, Daysi dibujo un
triángulo obtuso



6. Determine el menor número natural N que satisface las siguientes dos condiciones:
 Existen dos dígitos adyacentes de N cuya suma es 13.
 Existen dos dígitos adyacentes de N cuya suma es 12.
Aclaración: Dos dígitos son adyacentes si se encuentran uno al lado de otro.
Solución:
Sea el menor número natural de tres cifras: 𝑁=𝑎𝑏𝑐 (Se ha escogido tres cifras porque se trata de
encontrar al número más pequeño)

Dos dígitos adyacentes de N cuya suma sea 13.


𝑏 + 𝑐 = 13
Dos dígitos adyacentes de N cuya suma sea 12.
𝑎 + 𝑏 = 12
Vamos a tantear con ésta última expresión ya que es la menor suma:

Por lo tanto, el menor número encontrado es 394.


7. Determine cuántos números capicúas de cinco dígitos son múltiplos de 11. Aclaración: Un
número capicúa es aquel que se lee igual de izquierda a derecha que de derecha a izquierda. Por
ejemplo, 11; 101 y 2772 son números capicúas.
Solución:
̅̅̅̅̅̅̅̅̅
Número capicúa de cinco cifras: 𝒂𝒃𝒄𝒃𝒂
Se sabe que un número es divisible por once cuando se le multiplica por +1 a los lugares impares
y por –1 a los lugares pares. Es decir:

̅̅̅̅̅̅̅̅ = ̅̅
𝑎𝑏𝑐𝑏𝑎 11̅̅
+−+−+
𝑎 − 𝑏 + 𝑐 − 𝑏 + 𝑎 = ̅11 ̅̅̅
2𝑎 − 2𝑏 + 𝑐 = 11 ̅̅̅̅

Para esta situación problemática los múltiplos de 11 son: 0; 11; –11 y 22


PRIMER CASO: Cuando el múltiplo de once es cero (0).
2𝑎 – 2𝑏 + 𝑐 = 0
Probemos cuando 𝒄 = 𝟎:
2𝑎 – 2𝑏 + 0 = 0
2𝑎 – 2𝑏 = 0
2𝑎 = 2𝑏
𝑎=𝑏
𝑎, no puede tomar el valor de cero por ser el primer dígito. En consecuencia, a puede tomar los
valores del 1 al 9. Por tanto, los números serían: 11011; 22022; 33033; 44044; 55055; 66066;
77077; 88088 y 99099. Total: 9 números.
Probemos cuando c = 2, ya que c no puede tomar valores impares.
2𝑎 – 2𝑏 + 2 = 0
2𝑎 + 2 = 2𝑏
𝑎+1=𝑏
𝑎, puede tomar los valores del 1 al 8, no puede tomar el valor de 9 porque más uno tendría dos
dígitos. Por tanto, los números serían: 12221; 23232; 34243; 45254; 56265; 67276; 78287 y
89298. Total: 8 números.
Probemos cuando 𝑐 = 4:
2𝑎 – 2𝑏 + 4 = 0
2𝑎 + 4 = 2𝑏
𝑎 + 2 = 𝑏 (Sacando mitad a la expresión anterior)
𝑎, puede tomar los valores del 1 al 7, no puede tomar el valor de 8 y 9 porque más uno tendríamos
dos dígitos. Por tanto, los números serían: 13431; 24442; 35453; 46464; 57475; 68486 y 79497.
Total: 7 números.
Y así sucesivamente…
Cuando c = 6. Total: 6 números. Los números son: 14641; 25652; 36663; 47674; 58685 y
69696.
Cuando c = 8. Total: 5 números. Los números son: 15851; 26862; 37873; 48884 y 59895.

SEGUNDO CASO: Cuando el múltiplo de once es once (11).


2𝑎 – 2𝑏 + 𝑐 = 11
Probemos cuando 𝑐 = 1, c no puede tomar valores pares:

2𝑎 – 2𝑏 + 1 = 11
2𝑎 − 2𝑏 = 10
𝑎−𝑏 =5
𝑎 =𝑏+5
𝑏, puede tomar los valores del 0 al 4. Por tanto, los números serían: 50105; 61116; 72127; 83138
y 94149. Total: 5 números.
Probemos cuando 𝑐 = 3:
2𝑎 − 2𝑏 + 3 = 11
2𝑎 − 2𝑏 = 8
𝑎−𝑏 =4
𝑎 = 𝑏 + 4
𝑏, puede tomar los valores del 0 al 5. Por tanto, los números serían: 40304; 51315; 62326; 73137;
84348 y 95359. Total: 6 números.
Probemos cuando 𝑐 = 5:
2𝑎 − 2𝑏 + 5 = 11
2𝑎 − 2𝑏 = 6
𝑎−𝑏 =3
𝑎 =𝑏+3
𝑏, puede tomar los valores del 0 al 6. Por tanto, los números serían: 30503; 41514; 52525; 63536;
74547; 85558 y 96569. Total: 7 números.
Probemos cuando 𝑐 = 7:
2𝑎 − 2𝑏 + 7 = 11
2𝑎 − 2𝑏 = 4
𝑎−𝑏 =2
𝑎 =𝑏+2
𝑏, puede tomar los valores del 0 al 7. Por tanto, los números serían: 20702; 31713; 42724; 53735;
64746; 75757; 86768 y 97779. Total: 8 números.
Probemos cuando 𝑐 = 9:
2𝑎 − 2𝑏 + 9 = 11
2𝑎 − 2𝑏 = 2
𝑎−𝑏 =1
𝑎 =𝑏+1
𝑏, puede tomar los valores del 0 al 8. Por tanto, los números serían: 10901; 21912; 32923; 43934;
54945; 65956; 76967; 87978 y 98989. Total: 9 números.
TERCER CASO: Cuando el múltiplo de once es menos once (–11).
2𝑎 − 2𝑏 + 𝑐 = −11
Probemos cuando 𝑐 = 1, 𝑐 no puede tomar valores pares:
2𝑎 − 2𝑏 + 1 = −11
2𝑎 − 2𝑏 = 12
2𝑏 − 2𝑎 = 12
𝑏−𝑎 =6
𝑏 =𝑎+6
𝑎, puede tomar los valores del 1 al 3. Por tanto, los números serían: 17171; 28182 y 39193.
Total: 3 números.
Probemos cuando 𝑐 = 3, 𝑐 no puede tomar valores pares:
2𝑎 − 2𝑏 + 3 = −11
2𝑎 − 2𝑏 = −14
2𝑏 − 2𝑎 = 14
𝑏−𝑎 =7
𝑏 =𝑎+7
𝑎, puede tomar los valores del 1 al 2. Por tanto, los números serían: 18381 y 29392. Total: 2
números.
Cuando 𝑐 = 5. Total: 1 número. El número es: 19591.
CUARTO CASO: Cuando el múltiplo de once es veintidós (22).
2𝑎 − 2𝑏 + 𝑐 = 22
Probemos cuando 𝑐 = 4, 𝑐 ≥ 4:
2𝑎 − 2𝑏 + 4 = 22
2𝑎 − 2𝑏 = 18
𝑎−𝑏 =9
𝑎 =𝑏+9
𝑏, puede tomar sólo cero (0). Por tanto, el número es: 90409. Total: 1 número.
Probemos cuando 𝑐 = 6:
2𝑎 − 2𝑏 + 6 = 22
2𝑎 − 2𝑏 = 16
𝑎−𝑏 =8
𝑎 =𝑏+8
𝑏, puede tomar los valores del 0 al 1. Por tanto, los números serían: 80608 y 91619. Total: 2
números.
Probemos cuando 𝑐 = 8:
2𝑎 − 2𝑏 + 8 = 22
2𝑎 − 2𝑏 = 14
𝑎−𝑏 =7
𝑎 =𝑏+7
𝑏, puede tomar los valores del 0 al 2. Por tanto, los números serían: 70807; 81818 y 92829.
Total: 3 números.
Sumando la cantidad de números encontrados por cada caso:
PRIMER CASO: 9 + 8 + 7 + 6 + 5 = 35
SEGUNDO CASO: 5 + 6 + 7 + 8 + 9 = 35
TERCER CASO: 3 + 2 + 1 = 6
CUARTO CASO: 1 + 2 + 3 = 6
TOTAL: 35 + 35 + 6 + 6 = 82.
Por lo tanto, 82 números capicúas de cinco dígitos son múltiplos de 11.
8. Sea 𝑁 = 3 ∙ 52 ∙ 2𝑥 si 𝑚𝑐𝑑(96, 240, 𝑁) = 24, Calcule el valor de 𝑥.
Solución:
Factorizando tenemos que 96 = 25.3 y 240 = 24.3.5. Como mcd (96,240) = 24.3 y mcd (96, 240,
N) = 24 = 23.3 entonces debe ser N = 3.52.23.
Por lo tanto el valor de 𝑥 es 3.
9. En la figura, la longitud de los lados del cuadrado ABCD es de 4cm. E es el
punto medio de AD y F es el centro del cuadrado. ¿Cuál es el área en cm2
del triángulo BEF?
Solución:
En la figura el cuadrilátero ABFE es un trapecio rectangular cuya base menor 𝐸𝐹 = 2 𝑐𝑚, la
base mayor es: 𝐴𝐵 = 4 𝑐𝑚 y la altura es: 𝐴𝐸 = 2 𝑐𝑚, entonces su área es:
(𝐵 + 𝑏)ℎ (4 + 2) × 2
𝐴= = = 6𝑐𝑚2
2 2
El triángulo EAB es rectángulo en el vértice A, con catetos 2 cm y 4 cm respectivamente,
entonces su área está dada por:
(𝐴𝐵)(𝐴𝐸) 4 × 2
𝐴= = = 4𝑐𝑚2
2 2

Entonces el área del triángulo BEF está dada por:

𝐴∆𝐵𝐸𝐹 = 𝐴𝐴𝐵𝐹𝐸 − 𝐴∆𝐸𝐴𝐵 = 6𝑐𝑚2 − 4𝑐𝑚2 = 2𝑐𝑚2


Por lo tanto, el área del triángulo BEF es 2cm2.
10. El rectángulo de la figura mostrado a la derecha está dividido en 10
cuadrados. El perímetro del cuadrado gris es de 48cm. Calcule el área del
rectángulo.
Solución:
48
Como el perímetro del cuadrado gris es 48 cm, entonces su lado es de 𝑙 = 4 = 12 𝑐𝑚. El lado
del cuadrado gris es igual a tres veces el lado de los cuadraditos pequeños, por lo tanto 𝑙 = 3𝑥,
entonces: 12 = 3𝑥 → 𝑥 = 4 𝑐𝑚, de donde el largo del rectángulo es:
12 𝑐𝑚 + 3(4 𝑐𝑚) = 24 𝑐𝑚.
La medida del lado de los cuadrados medianos es de 𝑦 = 2(4 𝑐𝑚) = 8 𝑐𝑚.
Entonces el ancho del rectángulo es de 𝑙 + 𝑦 = 12 𝑐𝑚 + 8 𝑐𝑚 = 20 𝑐𝑚,
Por lo tanto, el área del rectángulo es igual a 𝐴 = 𝑏ℎ = (24 𝑐𝑚)(20 𝑐𝑚) = 480 𝑐𝑚2 .
11. Calcule la medida de la suma de los cuatro ángulos marcados en la figura.

Solución:
Sean los triángulos ABE y CDE y 𝑥, 𝑦 los ángulos que se
indican en la figura. Como la suma de los 3 ángulos
exteriores de un triángulo es 360°, tenemos en el triángulo
ABE que 𝑎 + 𝑏 + 𝑦 = 360° y en el triángulo CDE,
𝑐 + 𝑑 + 𝑥 = 360°
Luego
𝑎 + 𝑏 + 𝑐 + 𝑑 + 𝑥 + 𝑦 = 2(360°) = 720°
Como 𝑥 + 𝑦 = 180°, por ser suplementarios, entonces
𝑎 + 𝑏 + 𝑐 + 𝑑 + 180° = 720°
𝑎 + 𝑏 + 𝑐 + 𝑑 = 720° − 180°
𝑎 + 𝑏 + 𝑐 + 𝑑 = 540°
Por lo tanto, deducimos que 𝑎 + 𝑏 + 𝑐 + 𝑑 = 540°
12. Un arqueólogo ha descubierto que una antigua civilización usaba 5 símbolos para representar
los números: 𝜃, ∆, ∎, ⋇, †. Estos símbolos corresponden en algún orden a los dígitos
0, 1, 2, 3 𝑦 4. De este modo, cuando escriben ∆∎ † representan un número en base 5: ∆∎ † =
∆ ∙ 52 + ∎ ∙ 51 + † El arqueólogo sabe que los siguientes tres números son consecutivos,
ordenados de menor a mayor: ⋇† ∆∎, ⋇† ∆𝜃 𝑦 ⋇†⋇†. Hallar el valor de cada símbolo y cuáles
son los tres números consecutivos.
Solución:
Tener en cuenta que los números están en base 5, y que algunas de las primeras cifras se repiten.
También tener en cuenta que son consecutivos, por lo tanto, si los tres números son 𝑎, 𝑏, 𝑐;
entonces, 𝑏 = 𝑎 + 1 𝑦 𝑐 = 𝑏 + 1, 𝑜 𝑐 = 𝑎 + 2.
𝜃 = 4, ∆ = 1, ∎ = 3, ⋇ = 2 𝑦 † = 0. Los tres números escritos en clave son: 2013,
2014, 2020
Estos tres números representados en base 5 son:
2013 = 2 ∙ 53 + 0 ∙ 52 + 1 ∙ 51 + 3 = 250 + 0 + 5 + 3 = 258
2014 = 2 ∙ 53 + 0 ∙ 52 + 1 ∙ 51 + 4 = 250 + 0 + 5 + 4 = 259
2020 = 2 ∙ 53 + 0 ∙ 52 + 2 ∙ 51 + 0 = 250 + 0 + 10 + 0 = 260
Finalmente, los tres números consecutivos son: 258, 259, 260
13. Hallar un número entero positivo x tal que
𝑥
̂
= 0,0𝑎25
432
̂ = 0,0𝑎25𝑎25𝑎25 …, o sea, el período es 𝑎25.
Donde a es un dígito. Aclaración: 0,0𝑎25
Solución:
̂ a fracción común.
Primeramente se transforma el número periódico 0,0𝑎25
Sea 𝑘 = 0,0𝑎25𝑎25𝑎25𝑎25 …,
Por reglas se puede hacer
𝑎25 − 0 𝑎25
0,0𝑎25𝑎25𝑎25𝑎25 … , = =
999 999
Entonces, nos queda de la siguiente forma la ecuación
𝑥 𝑎25
=
432 999
Despejando y simplificando fracciones obtendremos,
16
𝑥= ∙ 𝑎25
37
Por condiciones dadas a es un digito y los números dígitos son {0,1,2,3, … ,9}, solamente para a
= 9 podemos obtener un numero entero para x = 400.
Por lo tanto, para a = 9, x = 400.
14. Calcule el área sombreada de la siguiente figura:

Solución:
Si pasamos la parte roja para la parte de arriba que está en blanco formamos medio rectángulo
sombreado, luego
1
𝐴𝑆 = 𝑑𝑒𝑙 𝑟𝑒𝑐𝑡𝑎𝑛𝑔𝑢𝑙𝑜
2
1
𝐴𝑆 = × 12𝑚 × 4𝑚
2
1
𝐴𝑆 = × 48𝑚2
2
𝐴𝑆 = 24𝑚2
15. Hallar el área sombreada de la siguiente figura:

Solución:

Si dividimos el ovalo azul en 2 y lo pasamos para la parte blanca formamos un triángulo de base
de 12cm y de altura de 12 cm luego:
𝑏 × ℎ 12𝑐𝑚 × 12𝑐𝑚
𝐴𝑆 = = = 72𝑐𝑚2
2 2
RTA / 72 cm²

16. ¿Cuántos números de la forma ̅̅̅̅̅̅̅̅̅̅


𝑎𝑏𝑐𝑑𝑒𝑓 existen, tales que su cuádruple es ̅̅̅̅̅̅̅̅̅̅
𝑒𝑓𝑎𝑏𝑐𝑑?

Solución:
̅̅̅̅̅̅̅̅̅̅ ) = (𝑒𝑓𝑎𝑏𝑐𝑑
Por dato: 4(𝑎𝑏𝑐𝑑𝑒𝑓 ̅̅̅̅̅̅̅̅̅̅ )
Descomponiendo en bloques:

̅̅̅̅̅̅̅̅̅̅ + ̅̅̅
(𝑎𝑏𝑐𝑑00 𝑒𝑓 )4 = ̅̅̅̅̅̅̅̅̅̅
𝑒𝑓0000 + ̅̅̅̅̅̅̅
𝑎𝑏𝑐𝑑
̅̅̅̅̅̅̅) + 4(𝑒𝑓
⇒ 400(𝑎𝑏𝑐𝑑 ̅̅̅) = 10 00(𝑒𝑓
̅̅̅) + 𝑎𝑏𝑐𝑑
̅̅̅̅̅̅̅
̅̅̅̅̅̅̅) = 9996(𝑒𝑓
399(𝑎𝑏𝑐𝑑 ̅̅̅ )
399 = 19 × 21 9996 = 476 × 21
̅̅̅̅̅̅̅ ) = 476(𝑒𝑓
⇒ 19(𝑎𝑏𝑐𝑑 ̅̅̅ ) … (∝)

Debemos conseguir dos factores en cada miembro, uno de dos cifras y el otro de 4 cifras,
tenemos:
̅̅̅̅̅̅̅ ) = 1428(𝑒𝑓
∝× 3: 57(𝑎𝑏𝑐𝑑 ̅̅̅ )
El número ̅̅̅̅̅̅̅̅̅̅
𝑎𝑏𝑐𝑑𝑒𝑓 = 142 857 (1a solución)
̅̅̅̅̅̅̅ ) = 1904(𝑒𝑓
∝× 4: 76(𝑎𝑏𝑐𝑑 ̅̅̅ )
̅̅̅̅̅̅̅̅̅̅ = 190 476 (2a solución)
El número 𝑎𝑏𝑐𝑑𝑒𝑓
̅̅̅̅̅̅̅ ) = 2380(𝑒𝑓
∝× 5: 95(𝑎𝑏𝑐𝑑 ̅̅̅)
El número ̅̅̅̅̅̅̅̅̅̅
𝑎𝑏𝑐𝑑𝑒𝑓 = 238 095 (3a solución)
Existen 3 números.
17. Este año hubo más de 800 corredores participando en una carrera. Exactamente el 35% de los
corredores fueron mujeres, y participaron 252 hombres más que mujeres. ¿Cuántos corredores
hubo en total?

Solución:
La cantidad de hombres participantes fue de 100% - 35% = 65%, de manera que participó un
30% más de hombres que de mujeres.
252
Como esta cantidad es de 252, el número total de participantes fue de 0,3 = 840
18. Dos segmentos, cada uno de 1 cm de largo, están marcados en lados opuestos de un cuadrado
de lado 8 cm. Los extremos de los segmentos se unen como se muestra en el diagrama. ¿Cuál
es el área sombreada?

Solución:
El área sombreada es la suma de las áreas de los dos triángulos que se forman. Como la base de
cada uno mide 1 cm y la suma de sus alturas mide 8 cm, el área sombreada es 4 cm2.

19. En la figura mostrada en la derecha, el triángulo 𝐴𝐵𝐶 es equilátero, con


̅̅̅̅ ̅̅̅̅ 𝑦 ̅̅̅̅̅
𝑃𝑁 ∥ 𝐵𝐶 𝑀𝑁 ∥ ̅̅̅̅
𝐴𝐵 , ¿Cuál es la razón entre el perímetro de la
superficie sombreada y el perímetro de la superficie no sombreada?

Solución:
Se sabe que los ángulos internos del triángulo 𝐴𝐵𝐶 miden
60° cada uno por ser equilátero. Además el ∠𝐴𝐵𝐶 =
∠𝐶𝑀𝑁 = ∠𝐴𝑃𝑁 = 60° por ser ángulos correspondientes
entre paralelas, por lo que los ∆𝐴𝑃𝑁 𝑦 ∆𝑀𝑁𝐶 son
semejantes al ∆𝐴𝐵𝐶 por AA, por tanto se concluye que los
triángulos 𝐴𝑃𝑁 𝑦 𝑀𝑁𝐶 también son equiláteros como se
muestra en la figura.
Se pide calcular:
𝑃𝑆 : Perímetro de la superficie sombreada.
𝑃𝑁𝑆 : Perímetro de la superficie no sombreada.

𝑃𝑆 3𝑎 + 3𝑏 3(𝑎 + 𝑏) 3
= = =
𝑃𝑁𝑆 2𝑎 + 2𝑏 2(𝑎 + 𝑏) 2
3
Por tanto la razón entre el 𝑃𝑆 𝑦 𝑃𝑁𝑆 es
2

20. En la figura mostrada a la derecha. Calcule la razón entre el área de la


región sombreada y el área del circulo (∆ 𝐴𝐵𝐶 es equilátero).

Solución:
En el triángulo equilátero 𝐴𝐵𝐶 las seis regiones son equivalentes,
entonces podemos transformar la figura de la siguiente forma: 𝐵𝑂𝐶 es un sector circular con un
1
ángulo central de 120°, equivalente a 3 de 360°, por tanto, la razón entre la región sombreada y
1
el área del circulo es 3.
21. En la casilla del siguiente tablero se va a escribir un número
entero positivo de tal forma que cada número que está en la
fila superior sea igual al producto de los dos números que
están debajo de él. Si los diez números que se van a escribir
son distintos entre sí, determine el mayor valor posible de 𝑏 +
2𝑑.

Solución:

Se descompone 4320 en sus factores primos 25 ∗ 33 ∗ 5, de aquí se puede expresar 4320 en el


producto de los siguientes pares de números:
(60 𝑦 72), (120 𝑦 36), ( 48 𝑦 90), ( 160 𝑦 27), ( 96 𝑦 45), ( 32 𝑦 135). Como los números a
escribirse son distintos solo se cumple para dos de las parejas de números.

Y de estas el mayor valor posible para 𝑏 + 2𝑑 es: 13

3 + 2(4) = 11
3 + 2(5) = 13

22. Cada casilla de un tablero de 10 × 10 se va a pintar de rojo, verde o azul, de tal forma que cada
subtablero de 3 × 3 tenga al menos una casilla de cada uno de los tres colores. ¿Cuántas casillas
rojas puede haber como máximo?
Solución:
Encontrando una distribución de las casillas con las condiciones del problema:

Dónde:
A: Casilla de color azul.
V: Casilla de color verde.
R: Casilla de color rojo.
Casillas de color rojo: 7(10) + 4(3) = 70 + 12 = 82
Por lo tanto, como máximo pueden haber 82 casillas de color rojo.
23. Hay 10 tarjetas numeradas del 1 al 10. En la figura1, se muestran cinco de las tarjetas. Las
restantes se quieren aparear con las que se muestran, de manera que las sumas de las parejas
sean 9, 10, 11, 12 y 13 (sin repetir). ¿De cuántas maneas es posible hacer esto?
Solución:
Coloquemos los números en una tabla, donde
pondremos las tarjetas que se miran en negritas y
las que están ocultas en texto normal. Sumamos
y tachamos con una raya oblicua (eliminamos) las
sumas que están por debajo de 9 y por encima de
13.

Una observación más detallada me dice que, el salir


el 11 solamente una vez, quiere decir que al usar el
11, se elimina automáticamente los números de la
filas y columnas correspondientes. Colocamos una
cruz en esos números.
Finalmente, la respuesta es, por el principio
multiplicativo, tenemos que: la suma 9, aparece 2
veces; el 10, 2 veces, el 11, 1 vez; el 12, 2 veces y
el 13 2 veces, total 16 veces.
Por tanto existen 16 maneras distintas de conseguir esas sumas.
24. En una pizzería ofrecen la versión básica de pizza con queso. Se puede ordenar de queso o se le
puede agregar uno o dos ingredientes entre peperoni, jamón, hongos o aceitunas. Hay tres
tamaños: pequeña, mediana o grande. ¿Cuántos estilos diferentes de pizza hay?
Solución:
Vamos a asumir que se pide una pizza con queso. Tenemos 4 opciones de escoger entre uno o
4 4
dos ingredientes: peperoni, jamón, hongos o aceitunas. Es decir, tenemos ( ) = 4, o ( ) = 6,
1 2
decir, 4 + 6 = 10 formas de seleccionar los ingredientes. Pero también tres tamaños distintos,
lo que hace un total de 3 × 10 = 30.
Por tanto, tenemos 30 estilos diferentes para seleccionar una pizza.
25. En una isla hay 3 tipos de camaleones: 13 camaleones rojos, 15 camaleones azules y 17
camaleones verdes. Cada vez que un camaleón rojo y un camaleón verde se topan, ambos se
convierten en camaleones azules. Cada vez que un camaleón rojo y azul se topan, ambos se
convierten en camaleones verdes. Y, por último, cada vez que un camaleón azul y un camaleón
verde se topan, ambos se convierten en camaleones rojos. Si, por ejemplo, un camaleón se
vuelve azul y luego se vuelve rojo, ambos se volverán verdes. No hay límite en la cantidad de
veces que los camaleones pueden toparse entre sí. ¿Es posible que todos los camaleones se
conviertan en un solo color? Por favor explique y pruebe su respuesta.
Solución
La respuesta es no. La diferencia entre los dos siempre será uniforme, lo que significa que si
queremos obtener otro de otro tipo, tendremos que restar la cantidad 𝑛. Si 𝑛 es impar, entonces
el otro tipo es par, una contradicción. Si 𝑛 es par, el otro también es par, sin embargo, el otro
tiene más camaleones ahora, por lo que este ciclo se repite.
26. Para pesar un objeto usando una balanza, Brady coloca el objeto en un lado de la balanza y
coloca suficientes pesos en cada lado para equilibrar los dos lados de la balanza. El juego de
pesas de Brady contiene el número mínimo necesario para medir el peso del número entero de
cualquier objeto de 1 a 40 libras, inclusive. ¿Cuál es el mayor peso, en libras, de un peso en el
set de Brady?
Solución
En una balanza, podemos colocar algunos de los pesos con el objeto que se pesa. Esto tiene el
efecto de restar del valor de los pesos colocados en el otro lado de la báscula. El resultado es
que podemos obtener todos los valores enteros positivos usando potencias de 3. Para pesar
cualquier número entero de libras hasta 40, necesitamos solo cuatro pesas que sean 1, 3, 9 y 27
libras. El mayor peso es de 27 libras.
27. Calcular el valor de “a”, en:
1𝑎 + 2𝑎 + 3𝑎 + 4𝑎 = 7̇
Solución
Aplicamos descomposición polinómica en cada numeral y agrupamos términos semejantes:

10 + 𝑎 + 20 + 𝑎 + 30 + 𝑎 + 40 + 𝑎 = 7̇
100 + 4𝑎 = ̇ 7
Factorizando, tenemos:
4(25 + 𝑎) = 7 → 25 + 𝑎 = 7
7̇ + 4 = 𝑎 + 7
4 + 𝑎 = 7̇
→𝑎=3
28. María (que es una estudiante obsesiva) eligió un código para su celular cuyos dígitos son
̅̅̅̅̅̅̅̅ (es decir, tiene 5 dígitos) y también se cumple que: 3𝑎 + 𝑏 + 3𝑐 + 𝑑 + 3𝑒 es múltiplo
𝑎𝑏𝑐𝑑𝑒
de 10.
 Por ejemplo, 23289 es un número que cumple esta propiedad porque:
3 × 2 + 1 × 3 + 3 × 2 + 1 × 8 + 3 × 9 = 50 es múltiplo de 10.
 Por otro lado 11111 no es un número válido porque:
3 × 1 + 1 × 1 + 3 × 1 + 1 × 1 + 3 × 1 = 11 no es múltiplo de 10.
María recuerda que el código que eligió es ̅̅̅̅̅̅̅̅
2019𝑒 pero como podemos notar no se acuerda
del último dígito, ¿Cuál es el último dígito?
Solución
De acuerdo a los datos del problema:
o
̅̅̅̅̅̅̅̅ se cumple 3𝑎 + 𝑏 + 3𝑐 + 𝑑 + 3𝑒 = 10
Todo número de cinco dígitos: 𝑎𝑏𝑐𝑑𝑒
Si 𝑒 es el último digito que María no recuerda:
̅̅̅̅̅̅̅̅
2019𝑒
o
3(2) + (0) + 3(1) + (9) + 3𝑒 = 10
o
18 + 3𝑒 = 10
o
Como 10 = {10, 20, 30, 40, 50, 60, … , }
El valor que cumplirá la igualdad y que 𝑒 es un dígito es:
𝑒=4
Comprobación
Número Condición
Valor de 𝑒 ̅̅̅̅̅̅̅̅ o
2019𝑒 3𝑎 + 𝑏 + 3𝑐 + 𝑑 + 3𝑒 = 10
4 20194 3(2) + 0 + 3(1) + (9) + 3(4) = 30
Por lo tanto, el último digito del número de teléfono de María es 4.
29. ¿De cuántas formas pueden sentarse 7 personas alrededor de una mesa, si dos personas
determinadas no deben estar una al lado de otra?
Solución:
Considérense las dos personas que no han de ir juntas como una sola. Entonces hay 6 personas
para sentarse en círculo, que lo pueden hacer de 5! formas. Pero las dos personas consideradas
como una sola pueden ordenarse de entre sí en 2! formas. Así, pues, el número de ordenaciones
de 6 personas sentadas alrededor de una mesa con 2 determinadas de ellas sentadas juntas es
5!·2! = 240.
Pero el número total en que 7 personas pueden sentarse juntas es 6! = 720. Por lo tanto, el
número total que pueden sentarse alrededor de la mesa sin que dos de ellas no estén sentadas
juntas es 720 – 240 = 480 formas.
30. Si cuatro amigos quieren ir al cine y al teatro, ¿de cuántas formas se pueden distribuir 6 entradas
de cine y 7 entradas de teatro, de tal modo que cada uno reciba al menos una entrada de teatro?
Solución:
Cada amigo recibe una entrada de teatro. El resto se reparte entre los 4 amigos, pudiendo uno
de ellos las tres entradas, por tanto, las entradas de teatro se pueden recibir de:
4+3−1 6!
𝐶𝑅4,3= ( ) = 3!3! = 20
3
Las entradas de cine se pueden distribuir de:
4+6−1 9
𝐶𝑅4,6 = ( ) = ( ) = 84.
6 3
Por tanto, los seis amigos pueden repartirse las entradas de 20·84 = 1 680 formas diferentes.
31. El botón D de una calculadora multiplica el número de la pantalla por 10. Al inicio, el número
20 estaba en la pantalla y se apretó 9 veces seguidas el botón D. Finalmente, se restó 1 al
resultado. Calcule la suma de los dígitos del número final.
Solución:
De la información brindada en el problema se tiene que:
Número que estaba en la pantalla al inicio: 20
Se apretó 9 veces seguidas el botón D:
20 × 10 × 10 × 10 × 10 × 10 × 10 × 10 × 10 × 10
20 × 109
Al resultado se restó uno:
20 × 109 – 1
20 000 000 000 – 1
19 999 999 999
Hallando la suma de los dígitos del número final: 1 + 10(9) = 1 + 90 = 91.
Por lo tanto la suma de los dígitos del número final es 91.
32. En la siguiente figura, T es el punto de intersección de los
segmentos BD y EC. Si se cumple que ∡𝐵𝐴𝐶 = 30°,
𝑇𝐸 = 𝑇𝐵 𝑦 𝐶𝑇 = 𝐶𝐷, calcule la medida del ángulo ∡𝐵𝑇𝐶.
Solución:
Sea ∡𝐵𝑇𝐶 = 𝑥. Los triángulos EBT y CDT son isósceles, ∡𝐵𝐴𝐶 = 30°.

En el ∆𝐸𝐵𝑇 que es isósceles, sea ∡𝐵𝐸𝑇 = 𝜃 , también se cumple ∡𝑇𝐵𝐸 = 𝜃 . La suma de dos
ángulos interiores es igual al ángulo exterior del tercer vértice, entonces, 𝑥 = 2𝜃.

En él ∆𝐴𝐵𝐷, la suma de dos ángulos interiores es igual al ángulo exterior del tercer vértice,
entonces, ∡𝐵𝐷𝐶 = 𝜃 + 30°. En el ∆𝐶𝐷𝑇 que es isósceles, se cumple ∡𝐶𝑇𝐷 = 𝜃 + 30°.

Los ángulos θ + 30° y 2θ forman un par lineal, entonces se cumple:


𝜃 + 30° + 2𝜃 = 180°
3𝜃 = 150°
𝜃 = 50°
Hallando ∡𝐵𝑇𝐶: 𝑥 = 2𝜃 = 2(50°) = 100°.

33. Se muestran dos rectángulos ABCD y ECFG. Se sabe que las áreas de los triángulos EBC y
DFG son 3 cm2 y 12 cm2, respectivamente. Calcule la diferencia de las áreas de los triángulos
CDF y EAD.
Solución:
̅̅̅̅ , asignando variables: 𝐵𝐶 = 𝑎, 𝐵𝐸 = 𝑏, 𝐸𝐴 = 𝑐, como ABCD es un
Trazamos la altura 𝐷𝐻
rectángulo entonces se cumple 𝐷𝐴 = 𝑎, 𝐶𝐷 = 𝑏 + 𝑐.

Trazamos la altura ̅̅̅


𝐸𝐼 , como ABCD es un rectángulo, entonces 𝐸𝐼 = 𝑎. Se deduce que el
∆𝐷𝐺𝐹 ≅ ∆𝐷𝐻𝐹, también el ∆𝐶𝐸𝐷 ≅ ∆𝐶𝐻𝐷. Si dichos triángulos son congruentes entonces
también tienen la misma área.

𝑏×𝑎
El área del ∆𝐵𝐸𝐶 = = 3.
2

Nos piden hallar la diferencia de las áreas de los triángulos CDF y EAD.
𝑥 = 𝐴𝛥𝐶𝐷𝐹 − 𝐴𝛥𝐸𝐴𝐷
𝑥 = 𝐴𝛥𝐶𝐻𝐷 + 𝐴𝛥𝐻𝐹𝐷 − 𝐴𝛥𝐸𝐴𝐷
𝑥 = 𝐴𝛥𝐶𝐸𝐷 + 𝐴𝛥𝐻𝐹𝐷 − 𝐴𝛥𝐸𝐴𝐷
(𝑏 + 𝑐)𝑎 𝑐×𝑎
𝑥= + 12 − 2
2 2
𝑏×𝑎 𝑐×𝑎 𝑐×𝑎
𝑥= + + 12 −
2 2 2
𝑏×𝑎
𝑥= + 12
2
𝑥 = 3 + 12
𝑥 = 15
Por lo tanto, la diferencia de áreas de los triángulos CDF y EAD es 15 cm2.
34. El segmento AB ha rotado 38° en sentido anti horario hasta ocupar la posición del segmento AC.
Calcule la medida del ángulo agudo que forman las rectas BC y ℓ.

Solución:
Sea: “x” el ángulo que forman las rectas BC y ℓ, ∡𝐵𝐴𝐶 = 38°, porque el segmento AB ha
rotado 38° en sentido anti horario.

El ∆𝐴𝐵𝐶 es isósceles, donde AB = AC, porque el segmento AB ha rotado 38° en sentido


antihorario, es decir, se trata del mismo segmento, sea ∡𝐵𝐴𝐶 = 𝛼, entonces también se cumple
que ∡𝐴𝐵𝐶 = 𝛼.

La suma de los ángulos interiores del ∆ 𝐴𝐵𝐶 es 180°.


𝛼 + 𝛼 + 38° = 180°
2𝛼 = 180° – 38°
2𝛼 = 142°
𝛼 = 71°
La suma de los ángulos interiores del ∆ 𝐷𝐴𝐶 es 180°.
38° + 48° + 𝑥 + 𝛼 = 180°
86° + 𝑥 + 71° = 180°
157° + 𝑥 = 180°
𝑥 = 23°
Por lo tanto, la medida del ángulo agudo que forman las rectas BC y ℓ. es 23°.

35. Los cuadrados mostrados tienen áreas 4 cm2; 16 cm2 y 1 cm2 (de izquierda a derecha). Calcule
el área del triángulo ABC.
Solución:
El área de un cuadrado está dado por:
A = L2, donde L representa la medida de su lado.
Si se tienen áreas de cuadrados que miden 4 cm2; 16 cm2 y 1 cm2, entonces sus lados miden 2
cm; 4 cm y 1 cm respectivamente.
Vamos a graficar el ∆ 𝐴𝐵𝐶 con los datos obtenidos y completando demás vértices.

Hallando el área del ∆ 𝐽𝐼𝐾, como tiene dos lados iguales, entonces el ∆ 𝐽𝐼𝐾 es isósceles, de
manera que sus ángulos agudos miden 45°.

𝑏×ℎ
𝐴∆𝐽𝐼𝐾 = 2
2×2
𝐴∆𝐽𝐼𝐾 =
2
𝐴∆𝐽𝐼𝐾 = 2𝑐𝑚2
Hallando el área del ∆ 𝐽𝐴𝑀, dicho triángulo tiene un ángulo agudo de
45°, entonces el ∆ 𝐽𝐴𝑀 es isósceles de manera que sus lados miden 2 cm.

2×2
𝐴∆𝐽𝐴𝑀 =
2
𝐴∆𝐽𝐴𝑀 = 2𝑐𝑚2

Hallando el área del ∆ 𝐹𝐸𝐻. Sea ∡𝐹𝐸𝐻 = 𝛼 .

3×1
𝐴∆𝐹𝐸𝐻 =
2
3 2
𝐴∆𝐹𝐸𝐻 = 𝑐𝑚
2
Hallando el área del ∆ 𝐷𝐸𝐶. Los triángulos DEC y FEH son semejantes, por ello ∡𝐶𝐸𝐷 = 𝛼
Hallando “𝑛”
3 1 1
= → 𝑛 = 𝑐𝑚
1 𝑛 3
1
1×3
𝐴∆𝐷𝐸𝐶 =
2
1 2
𝐴∆𝐷𝐸𝐶 = 𝑐𝑚
6
Hallando el área del ∆ 𝐵𝐼𝐻, vamos a trazar la altura “ℎ”. Si ∡𝐵𝐼𝑁 = 45°, entonces
∡𝐼𝐵𝑁 = 45°. El ∆ 𝐵𝐼𝑁 es isósceles, por ello 𝐵𝑁 = 𝐼𝑁 = ℎ. Los triángulos FEH y BHN son
semejantes, en consecuencia ∡𝐵𝐻𝑁 = 𝛼.
Hallando “ℎ”
3 ℎ
= → 3(4 − ℎ) = ℎ → 12 − 3ℎ = ℎ → 12 = ℎ + 3ℎ
1 4−ℎ

4ℎ = 12 → ℎ = 3
4×3
𝐴∆𝐵𝐼𝐻 =
2
𝐴∆𝐵𝐼𝐻 = 6𝑐𝑚2
Finalmente sumando todas las áreas que componen el ∆ 𝐴𝐵𝐶:
𝐴∆𝐴𝐵𝐶 = 𝐴∆𝐽𝐼𝐾 + 𝐴∆𝐽𝐴𝑀 + 𝐴𝐽𝐾𝐿𝑀 + 𝐴𝐿𝐼𝐻𝐺 + 𝐴𝐷𝐸𝐹𝐺 + 𝐴∆𝐹𝐸𝐻 + 𝐴∆𝐷𝐸𝐶 + 𝐴∆𝐵𝐼𝐻

3 1
𝐴∆𝐴𝐵𝐶 = 2 + 2 + 4 + 16 + 1 + + + 6
2 6
10
𝐴∆𝐴𝐵𝐶 = 31 +
6
5 93 + 5 98
𝐴∆𝐴𝐵𝐶 = 31 + = =
3 3 3
98
Por lo tanto, el área del triángulo ABC es 𝑐𝑚2
3

36. Alejandro escogió cuatro números distintos del conjunto {1; 2; 3; 4; 5; 6; 7} y Samuel se quedó
con los otros tres números. Luego, Alejandro multiplicó sus cuatro números y Samuel multiplicó
sus tres números. Se sabe que al sumar el resultado de Alejandro con el resultado de Samuel se
obtiene un número primo, calcule la suma de los dígitos de dicho número primo.
Solución:
Sean los números que escogió Alejandro: 𝑥1 ; 𝑥2 ; 𝑥3 ; 𝑥4
Sean los números que escogió Samuel: 𝑦1 ; 𝑦2 ; 𝑦3
Alejandro multiplicó sus cuatro números y Samuel multiplicó sus tres números. Se sabe que al
sumar el resultado de Alejandro con el resultado de Samuel se obtiene un número primo:
(𝑥1 )(𝑥2 )(𝑥3 )(𝑥4 ) + (𝑦1 )(𝑦2 )(𝑦3 ) = Número primo
Analizando los productos:
Debido a que 4 es múltiplo de 2, entonces estos deben estar en el mismo grupo de números que
escogió uno de las personas, porque de lo contrario el resultado de la suma no será un número
primo si no un número múltiplo 2. De la misma manera sucede con 3 y 6. Por tanto,
Alejandro habrá escogido los números mencionados:
(𝑥1 )(𝑥2 )(𝑥3 )(𝑥4 ) + (𝑦1 )(𝑦2 )(𝑦3 ) = Número primo
(2)(4)(3)(6) + (1)(7)(5) = Número primo
144 + 35 = Número primo
179 = Número primo
Por lo tanto, la suma de dígitos del número primo es: 1 + 7 + 9 = 17.
37. Sean a, b, c y d números reales no nulos. Determine como máximo cuántos números negativos
puede haber entre los siguientes ocho números reales:
𝑎, 𝑏, 𝑐, 𝑑, 𝑎𝑏, 𝑏𝑐, 𝑐𝑑, 𝑑𝑎.
Aclaración: 𝑎𝑏 es el producto de los números a y b. Análogamente, 𝑏𝑐, 𝑐𝑑 𝑦 𝑑𝑎 también
denotan productos.
Solución:
Se trata de obtener la mayor cantidad de números negativos y por ello asignaremos
adecuadamente los signos a los números que no están expresados como productos:
𝑎: –
𝑏: +
𝑐: –
𝑑: +
Por lo tanto, como máximo 6 números negativos puede haber de entre los ocho números reales
del problema.
38. Con los dígitos 𝑎, 𝑏, 𝑐, 𝑑, 𝑒 𝑦 𝑓, que son distintos entre sí, Carlos formó el número de tres dígitos
̅̅̅̅̅ y Emilio formó el número de tres dígitos 𝑎𝑏𝑐
𝑑𝑒𝑓 ̅̅̅̅̅. Si el número de Carlos es mayor en 9 que el
número de Emilio, determine el menor valor posible de 𝑎 + 𝑏 + 𝑐.
Aclaración: Tenga en cuenta que 𝑎 ≠ 0 𝑦 𝑑 ≠ 0.
Solución:
̅̅̅̅̅
Carlos formó el número de tres dígitos: 𝑑𝑒𝑓
Emilio formó el número de tres dígitos: ̅̅̅̅̅
𝑎𝑏𝑐 .
Si el número de Carlos es mayor en 9 que el número de Emilio: ̅̅̅̅̅
𝑑𝑒𝑓 = ̅̅̅̅̅
𝑎𝑏𝑐 + 9
Para que “𝑎 + 𝑏 + 𝑐” sea el menor valor posible a = 1, los valores de b y c vamos a tantear y
obtenemos:
̅̅̅̅̅
𝑑𝑒𝑓 = ̅̅̅̅̅
𝑎𝑏𝑐 + 9
203 = 194 + 9

Por tanto, 𝑎 = 1; 𝑏 = 9 𝑦 𝑐 = 4. Cuya suma es: 1 + 9 + 4 = 14.

Por lo tanto, el menor valor posible de 𝑎 + 𝑏 +


𝑐 es 14.
39. En la figura, BC = 16 cm, AB = 12 cm, E y F son puntos medios. Determine el área del
cuadrilátero sombreado.

Solución: Hacemos los siguientes trazos como se muestra en la figura.

 Como 𝐸 y 𝐹 son puntos medios, entonces 𝐴𝐹 = 6 y 𝐴𝐸 = 8. Por tanto, el rectángulo 𝐴𝐹𝑂𝐸


tiene una área de 48.
 Las diagonales de 𝐴𝐹𝑂𝐸 los dividen en cuatro áreas iguales, es decir que el área del
triángulo 𝐸𝑂𝑃 es 12.
 No resta encontrar el área restante, para ello tomaremos como referencia el trapecio 𝐶𝐷𝐸𝑂,
las diagonales 𝐶𝐸 y 𝐷𝑂 dividen el trapecio en cuatro áreas las cuales las llamaremos
𝐴1, 𝐴2 , 𝐴3 y 𝐴4 . De estas áreas podemos obtener las siguientes relaciones.

(𝑂𝐸)(𝑅𝐶) (6)(8)
𝐴1 + 𝐴2 = 24 ya que [𝐶𝐸𝑂] = = = 24
2 2
(ℎ)(𝐷𝐶) (8)(12)
𝐴2 + 𝐴3 = 48 ya que [𝐶𝐷𝑂] = , siendo ℎ = 𝐸𝐷 = 8, [𝐶𝐷𝑂] = = 48.
2 2
 Despejamos las áreas en función de 𝐴2 .
𝐴1 = 24 − 𝐴2 y 𝐴3 = 48−𝐴2 , aplicando unos de los teoremas de relación de áreas de
trapecios, el cual afirma que (𝐴2 )2 = 𝐴1 ∙ 𝐴3 , entonces.
(𝐴2 )2 = (24 − 𝐴2 )(48−𝐴2 ) ⟹ (𝐴2 )2 = 1152 − 24𝐴2 − 48𝐴2 + (𝐴2 )2
72𝐴2 = 1152 ⟹ 𝐴2 = 16
 Como 𝐴1 + 𝐴2 = 24, entonces 𝐴1 = 24 − 𝐴2 ⟹ 𝐴1 = 24 − 16 ⟹ 𝐴1 = 8
 Por tanto, el área sombreada es [𝐸𝑂𝑃] + 𝐴1 = 12 + 8 = 20

40. Si se cumple que (a + b + c)2 = 196. Hallar las sumas de las cifras del resultado de
S = ̅̅̅̅̅
abc + ̅̅̅̅̅
bca + ̅̅̅̅̅
cab.
Solución: Extraemos raíz cuadrada a ambos lados.
√(𝑎 + 𝑏 + 𝑐)2 = √196
𝑎 + 𝑏 + 𝑐 = 14
Sumando de la siguiente forma:
𝑎 𝑏 𝑐
+ 𝑏 𝑐 𝑎
𝑐 𝑎 𝑏
1554
Por lo tanto, 𝑆 = 1 + 5 + 5 + 4 = 15.
41. Un triangulo rectángulo isósceles es cortado en cada una de las cuatro esquinas de la pieza
cuadrada de papel (en la figura se muestan los triangulos). Encuentre la longitud de la diaginal
d, si la suma de las areas de los triangulo cortados es 200 u2 .

Solución: Nombramos los lados de los triángulos isósceles con las letras 𝑎 y 𝑏, calculamos las
áreas de los triángulos sombreados las cuales son 200 𝑢2 .
𝑎2 𝑎2 𝑏 2 𝑏 2
+ + + = 200 ⟹ 𝑎2 + 𝑏 2 = 200
2 2 2 2

El lado mayor del rectángulo es √2𝑎 y el lado menor del rectángulo es √2𝑏 a como se muestra
en la figura. Por lo tanto, la diagonal es igual a:

2 2
𝑑2 = (√2𝑎) + (√2𝑏) ⟹ 𝑑2 = 2𝑎2 + 2𝑏 2 ⟹ 𝑑 = √2(𝑎2 + 𝑏 2 ) ⟹ 𝑑 = √2(200)
𝑑 = √400 ⟹ 𝑑 = 20
42. En la figura, las letras a, b, c, d, e, f son las áreas de las regiones correspondientes. Todos ellos
son números enteros positivos diferentes entre sí y menores a 10, además cada triángulo
formado por tres regiones tiene área par. ¿Cuál es el valor de f, si el area de la estrella es 31?

Solución: Por dato se sabe que cada triángulo formado por tres regiones tiene área par, y cada
área es menor que 10, podemos afirmar lo siguiente:
Suma de las tres regiones que forman el triángulo
𝑎+𝑓+𝑐
𝑎+𝑓+𝑑 La suma de estas tres
𝑑+𝑓+𝑏 regiones que forman el
𝑒+𝑓+𝑏 triángulo es par.
𝑒+𝑓+𝑐
Dado que 𝑓 esta en todas las sumas, diremos que 𝑓 es par ya que entre 1 − 9 hay 5 números
impares, entonces, llamemos por la letra 𝑃 a las áreas pares y por la letra 𝐼 a las áreas impares.
Además, hay que tener en cuenta lo siguiente:

𝑃 + 𝑃 = 𝑃
𝐼 + 𝐼 = 𝑃
𝑃 + 𝐼 = 𝐼
𝐼 𝑃 𝐼
𝑎+⏞
⏞ 𝑓+⏞
𝑐 =𝑷
𝐼 𝑃 𝐼
𝑎+⏞
⏞ 𝑓+⏞
𝑑=𝑷
𝐼 𝑃 𝐼
⏞+⏞
𝑑 𝑓+⏞
𝑏=𝑷
𝐼 𝑃 𝐼
𝑒+⏞
⏞ 𝑓+⏞
𝑏=𝑷
𝐼 𝑃 𝐼
𝑒+⏞
⏞ 𝑓+⏞
𝑐 =𝑷
Por tanto, 𝑎, 𝑏, 𝑐, 𝑑, 𝑒 son impares y los números impares menor que 10 son: {1, 3, 5, 7, 9},
además la suma del área de la estrella es 31, entonces.
1 + 3 + 5 + 7 + 9 + 𝑓 = 31 ⟹ 𝑓 = 31 − 25 ⟹ 𝑓 = 6
El valor de 𝑓 es 6.
43. Determine cuántos números de cuatro cifras cumplen con las condiciones siguientes: el primer
dígito del número es la cantidad de ceros que aparecen en él, el segundo dígito es la cantidad de
unos que aparecen en él; de manera similar el tercer dígito es la cantidad de dos y el último
dígito es la cantidad de tres.
Solución:

Sea 𝑁 = ̅̅̅̅̅̅̅
𝑎𝑏𝑐𝑑 el número. Buscando todos los posibles valores de a, tenemos que:
 𝑎 = 4, no es posible, pues N estaría formado por cuatro ceros.

 𝑎 = 3, tampoco es posible, pues N contendría tres ceros, pero 𝑁 = 3000, no cumple con la
condición del enunciado.
 Si 𝑎 = 2, entonces debe haber exactamente dos ceros. Pero 𝑐 ≠ 0, ya que N contiene un 2,
por lo que 𝑐 = 1; 2; 3. Si 𝑐 = 1 entonces las dos cifras restantes b y d deben ser cero, de
donde 𝑁 = 2010, pero este número no satisface las condiciones del problema. Si 𝑐 = 2,
también debemos tener 𝑏 = 𝑑 = 0, obteniendo 𝑁 = 2020, el cual si es una solución válida.
Finalmente 𝑐 = 3 es imposible.
 Si 𝑎 = 1, entonces N contiene exactamente un cero. Claramente 𝑏 ≠ 0 pues N contiene un
1, y en consecuencia 𝑐 = 0 o 𝑑 = 0. Dividamos nuevamente en casos:

 Si 𝑐 = 0 entonces N no contiene 2, pero como ya contiene un 1 vemos que 𝑏 = 1; 3. Si


𝑏 = 1, la condición no se cumple pues el número tendría dos 1, mientras que 𝑏 = 3 es
imposible porque no hay espacio para tres 1.
 Si 𝑑 = 0, entonces N no contiene 3, de donde 𝑏 ≠ 3; además 𝑏 ≠ 0 porque N ya contiene
un 1. Por último, 𝑏 = 1 es absurdo, pues N contendría dos 1; luego 𝑏 = 2 𝑦 𝑐 = 1 es la
única posibilidad, la cual nos da el valor 𝑁 = 1210.
 Si 𝑎 = 0, no es posible pues el número tiene cuatro cifras.
Por tanto las únicas soluciones al problema son 2020 y 1210.
44. Inicialmente hay un 1 en la pantalla de una computadora. Al presionar la tecla A, se multiplica
por 3 el número de la pantalla. Al presionar la tecla B, se resta 1 al número de la pantalla.
Utilizando una secuencia de teclas A y B hay que llegar a tener en la pantalla un 97. Determine
el número mínimo de teclas que se deben presionar.
Solución:
Ya que 97 no es divisible por 3, este debe ser obtenido a partir de un múltiplo de 3 presionando
la tecla B; usaremos la tecla el menor número de veces si partimos del múltiplo de 3 más cercano,
que es 99. Y como 99 = 9 ∙ 11, este puede ser alcanzado a partir de 11 con la tecla A
(claramente obtenerlo con B necesitaría más teclas). Aplicando exactamente el mismo
razonamiento, vemos que para llegar a 11 debemos haber pasado antes por 12, y a este llegamos
desde 4. Para llegar a 4 debemos haber partido del múltiplo de 3 más cercano, que es 6, y a partir
de este regresamos a 2.
Finalmente desde 2 retrocedemos a 3 y regresamos a 1. Por tanto la mínima secuencia de teclas
que nos lleva a 97 es:
1 → 3 → 2 → 6 → 5 → 4 → 12 → 11 → 33 → 99 → 98 → 97
En conclusión es necesario presionar por lo menos 7 + 2 + 2 = 11 teclas.

45. Los vértices de un triángulo son 𝐴(𝑝, 𝑞), 𝐵(𝑟, 𝑠) 𝑦 𝐶(𝑡, 𝑢), como
se muestra en la figura. Los puntos medios de los lados del
triángulo son 𝑀(– 2,1), 𝑁(2, – 1) 𝑦 𝑃(3,2). ¿Cuál es el valor de
𝑝 + 𝑞 + 𝑟 + 𝑠 + 𝑡 + 𝑢?
Solución:
Por definición de punto medio tenemos que:
𝑝+𝑟 𝑞+𝑠 𝑝+𝑡 𝑞+𝑢 𝑡+𝑟 𝑢+𝑠
= −2; = 1; = 3; = 2; = 2; = −1
2 2 2 2 2 2
Sumando todas las expresiones obtenidas anteriormente, tenemos que:

𝑝+𝑟 𝑞+𝑠 𝑝+𝑡 𝑞+𝑢 𝑡+𝑟 𝑢+𝑠


+ + + + + = −2 + 1 + 3 + 2 + 2 − 1
2 2 2 2 2 2
2𝑝 + 2𝑞 + 2𝑟 + 2𝑠 + 2𝑡 + 2𝑢
=5
2
2(𝑝 + 𝑞 + 𝑟 + 𝑠 + 𝑡 + 𝑢)
=5
2
𝑝+𝑞+𝑟+𝑠+𝑡+𝑢 =5
46. Tres de las cinco cartas que se muestran en la figura se le dan a Nadia,
y el resto a Raúl. Nadia multiplica los 3 valores de sus cartas y Raúl
los 2 de las suyas. Sucede que la suma de los dos productos obtenidos
es un número primo. ¿Cuál es la suma de los valores de las cartas de
Nadia?
Solución:
Lo primero que se debe de tener en cuenta es que hay 2 números pares el 4 y 6, por lo tanto,
estos números no pueden estar separados, porque al multiplicarse como se establece en el
problema se obtendrán dos productos pares y su suma también será un numero par mayor que 2
y el único primo par es el 2, por lo tanto, de esa manera no cumpliríamos con la condición del
problema.
Al multiplicar el producto de los 2 pares con uno de los tres impares para obtener el producto
de Nadia tenemos:
4 ∙ 6 ∙ 3 + 5 ∙ 7 = 72 + 35 = 107, que es primo.
4 ∙ 6 ∙ 5 + 3 ∙ 7 = 120 + 21 = 141, que no es primo, porque tiene también como factor el 3 y
47.
4 ∙ 6 ∙ 7 + 3 ∙ 5 = 168 + 15 = 183, que no es primo, porque tiene también como factor el 3 y
61.
Por lo tanto las cartas que tiene Nadia son 3, 4 y 6 y su suma es: 3 + 4 + 6 = 13
47. En el país de los colores, un billete verde equivale a 5 billetes rojos; un billete azul, a 10 billetes
verdes; uno amarillo, a 3 azules; y uno morado a 5 azules. Determinar a cuántos billetes rojos
equivalen 5 billetes amarillos más 5 billetes morados.
Solución:
Representemos con letras mayúsculas el color de los billetes de la siguiente manera:
V: Verde; R: Rojo, A: Azul, Am: Amarillo y M: Morado, entonces:
1𝑉 = 5𝑅; 1𝐴 = 10𝑉; 1𝐴𝑚 = 3𝐴 𝑦 1𝑀 = 5𝐴
5𝐴𝑚 = 5(3𝐴) = 15𝐴 = 15(10𝑉) = 150𝑉 = 150(5𝑅) = 750𝑅
5𝑀 = 5(5𝐴) = 25𝐴 = 25(1𝑂𝑉) = 250𝑉 = 250(5𝑅) = 1 250𝑅
5𝐴𝑚 + 5𝑀 = 750𝑅 + 1 250𝑅 = 2 000𝑅
Por lo tanto, equivalen a 2 000 billetes rojos.
48. Una lista de dígitos se escribe en una pizarra siguiendo el proceso: cada vez se escribe el ultimo
digito del producto 𝑎 × 𝑏, donde a y b son los últimos dos dígitos escritos, por ejemplo, si los
dígitos iniciales fueran 1, 8 entonces la lista seria 1, 8, 8, 4, .... Hallar el dígito de la posición
2020 si la lista inicia con los dígitos 3, 4.
Solución:
Veamos que los dígitos hasta la posición 9 son: 3, 4, 2, 8, 6, 8, 8, 4, 2
Entonces aparece un ciclo de 6 dígitos 4, 2, 8, 6, 8, 8 que se repite periódicamente, pero hay que
considerar que el 3 del inicio no forma parte del ciclo. Entonces los dígitos 4 aparecen en las
posiciones que tienen la forma 6k + 2 con 𝑘 ∈ 𝑍 + , y como 2020 = 6(336) + 4, entonces el
dígito en la posición 2020 es 8.
49. Lola ha comenzado a escribir algunos números en la tabla. Ella decide que cada fila y columna
contendrá los números 1, 2 y 3 exactamente una vez. ¿Cuál es la suma de los números que
escribirá en los dos cuadrados sombreados?
1
2 A
B
Solución:
Al completar la tabla nos queda:

1 3 2

3 2 1

2 1 3

Entonces la suma de 𝐴 + 𝐵 = 1 + 3 = 4
50. Todos los divisores del entero positivo N, diferentes de N y 1, se escriben en orden creciente.
¿Cuántos números naturales N son tales que el mayor de los divisores escritos es 45 veces más
grande que el menor?
Solución:
Si d es el menor divisor N mayor que 1, entonces el mayor divisor de N menor que N es 45d y
d. (45d) = 45𝑑 2 =N. Es claro que d debe ser primo y que sus únicos valores posibles son 2 y 3.
Luego, sólo hay dos números N posibles: 180 = 45.22 y 405 = 45.32 .
Nivel IV (Octavo Grado)

1. En la siguiente figura se muestra un cuadrado AMBO y un hexágono


regular TUMBES. Determine el valor de n para el cual los puntos U,
M, A (en ese orden) son vértices consecutivos de un polígono regular
de n lados.
Solución:
La medida del angulo interno de un poligono regular esta dado por:

180(𝑛 − 2)
∡𝑖 =
𝑛
El angulo unterior del cuadrado AMBO es 90º
El angulo interior del poligono regular TUMBES es:

180°(6 − 2)
∡𝑖 = = 30(4) = 120°
6
De forma grafica podemos observar que:

Si “𝑥” es la medida del ángulo interior del polígono regular de


“𝑛” lados, entonces tenemos que:

120° + 𝑥 + 90° = 360°

𝑥 + 210° = 360° ⇒ 𝑥 = 360° − 210° ⇒ 𝑥 = 150°


De lo anterior tenemos que:

180(𝑛 − 2) 180(𝑛 − 2)
∡𝑖 = ⇒ 150° =
𝑛 𝑛
150𝑛 = 180𝑛 − 360
150𝑛 − 180𝑛 = −360
−30𝑛 = −360
−360
𝑛= = 12
−30
Por lo tanto el polígono regular de “n” lados es el dodecágono.
2. Franco escribió un número que consta de 10 dígitos distintos. Luego, subrayó cada dígito que
es igual a la suma de sus dos dígitos vecinos (el de la izquierda y el de la derecha). ¿Cuántos
dígitos como máximo puede subrayar Franco?
Solución:
Los diez dígitos distintos son: 0; 1; 2; 3; 4; 5; 6; 7; 8; 9.
La suma de dos dígitos vecinos a lo más puede ser 9.
Los dígitos vecinos podrían ser:
1+2=3 2+3=5 3+4=7 4+5=9
1+3=4 2+4=6 3+5=8
1+4=5 2+5=7 3+6=9
1+5=6 2+6=8
1+6=7 2+7=9
1+7=8
1+8=9
Haciendo algunas combinaciones entre ellas podemos tener:

 1439682750
 7813264950
 1547396820
A lo más se pueden subrayar 4 dígitos agrupados en 9 cifras y la cifra cero sólo está para
completar las diez cifras.
Franco puede subrayar 4 dígitos como máximo.
3. María escribió un número de dos dígitos y luego invirtió el orden de sus dígitos para obtener
otro número de dos dígitos. Al hacer esto, el número original de María se incrementó en 45. Si
la suma de los dígitos del número original de María es 11, calcule el producto de estos dígitos.
Solución:
Sea el número original de dos dígitos: 𝑎𝑏
La suma de los dígitos del número natural es 11: 𝑎 + 𝑏 = 11 (1)
̅̅̅ + 45 = 𝑏𝑎
Al invertir el orden de sus dígitos el número se incrementó en 45: 𝑎𝑏 ̅̅̅ (2)
Descomponiendo polinomicamente la ecuación (2), tenemos que:
̅̅̅
𝑎𝑏 + 45 = ̅̅̅
𝑏𝑎
10𝑎 + 𝑏 + 45 = 10𝑏 + 𝑎
9𝑎 − 9𝑏 = −45
𝑎 − 𝑏 = −5 (3)
Con las ecuaciones (1) y (3) se forma el siguiente sistema de ecuaciones:
𝑎 + 𝑏 = 11
{
𝑎 − 𝑏 = −5
Sumando ambas ecuaciones tenemos que:
2𝑎 = 6
𝑎=3
Sustituyendo el valor de 𝑎 en (1) tenemos que:
𝑎 + 𝑏 = 11
3 + 𝑏 = 11
𝑏=8
El producto de sus dígitos es: 𝑎 × 𝑏 = 3 × 8 = 24
4. La sucesión 8; 10; 20; 22; 44;... se define de la siguiente forma: el primer término es 8 y para
obtener cada uno de los siguientes términos se suma 2 o se multiplica por 2, de forma alternada.
¿Cuál es el dígito de las unidades del término que está en el lugar 100?
Solución:
Planteando la sucesión:

Analizando las unidades de cada uno de los elementos de la sucesión:

Cada 8 términos vuelven a repetirse las cifras de las unidades.


Vamos a consideramos a partir del término ocho (t8). El término cien (𝑡100 )estaría dado por:
100 = 8(12) + 4. Como el residuo es 4, entonces la cifra de las unidades del término cien se
obtendría al recorrer cuatro casilleros más, es decir, hasta el número 2.
5. ¿Cuántos enteros positivos de 7 dígitos son múltiplos de 27 y cumplen que cada uno de sus
dígitos es 0 o 9? Aclaración: Tenga en cuenta que un entero positivo no empieza con el dígito
0.
Solución:
Como se tiene sólo cifras de 9 ó 0, entonces el número formado de siete dígitos será divisible
por 27 cuando la suma de sus cifras es múltiplo de 27.
PRIMER CASO: Cuando tiene tres nueves.

Se tienen 6 casilleros en la se pueden formar varios números con 2 nueves y 4 ceros. Ya que
importa el orden, entran todos los elementos y algunos se repiten, entonces tenemos una
permutación con repetición:
𝑛 𝑛!
𝑃𝛼;𝛽;… =
𝛼! 𝛽! ⋯
6
6! 6×5×4×3×2×1
𝑃2;4 = = = 3 × 5 = 15
2! 4! 2 × 1 × 4 × 3 × 2 × 1
SEGUNDO CASO: Cuando tiene seis nueves.

Se tienen 6 casilleros en los que se pueden formar varios números con 5 nueves y 1 cero, igual
que el primer caso tenemos una permutación con repetición:
𝑛 𝑛!
𝑃𝛼;𝛽;… =
𝛼! 𝛽! ⋯
6
6! 6 × 5 × 4×3×2×1
𝑃5;1 = = =6
5! 1! 5 × 4 × 3 × 2 × 1 × 1
Por lo tanto se tiene 15 + 6 = 21 números enteros positivos de 7 cifras múltiplos de 27 que
cumplen que cada uno de sus dígitos es 0 o 9
6. Alex tiene 2k palitos idénticos. Con k palitos puede formar el borde de un cuadrado y con los
otros k palitos puede formar el borde de un hexágono regular. Si en ningún caso fue necesario
que Alex rompa algún palito, ¿De que numero es múltiplo k?
Solución:
Alex tiene: 2k palitos idénticos.
“k” palitos puede formar el borde de un cuadrado
Al formar un cuadrado, el perímetro será: 4(𝑘/4), es decir, un múltiplo de 4.
“k” palitos puede formar el borde de un hexágono regular.
Al formar un hexágono regular, el perímetro será: 6(𝑘/6), es decir, un múltiplo de 6.
Para que se pueda construir dichos polígonos regulares y sin romper ningún palito,
necesariamente k tiene que ser simultáneamente múltiplo del MCM (Mínimo común múltiplo)
de 4 y 6. Entonces, k = MCM (4; 6) = 12. Por tanto, k es múltiplo de 12.
7. En la figura mostrada, ABC es un triángulo equilátero de perímetro 90
cm. Además, los segmentos PQ y AC son paralelos. Calcule la suma de
los perímetros de los polígonos PBQ y APQC (en cm), si se sabe que
estos números están en la relación de 3 a 14.
Solución:
Si el triángulo ABC es equilátero y tiene un perímetro de 90 cm, por tanto,
cada lado mide 30 cm.
Cada uno de sus ángulos interiores de un triángulo equilátero mide 60°.
Sea 𝑃𝐵 = 𝑥 𝑐𝑚 y realizamos el siguiente gráfico:
El triángulo PBQ también es equilátero porque
𝑃𝑄 ∥ 𝐴𝐶, en consecuencia 𝑃𝐵 = 𝐵𝑄 = 𝑃𝑄 = 𝑥.
Además, 𝑃𝐴 = 30 – 𝑥
Plateando la proporción de acuerdo al problema:
𝑃𝑒𝑟𝑖𝑚𝑒𝑡𝑟𝑜 ∆𝑃𝐵𝑄 3
=
𝑃𝑒𝑟𝑖𝑚𝑒𝑡𝑟𝑜 ∎𝐴𝑃𝑄𝐶 14

𝑥+𝑥+𝑥 3
=
30 + 30 − 𝑥 + 30 − 𝑥 + 𝑥 14
3𝑥 3
=
90 − 𝑥 14
14(3𝑥) = 3(90 − 𝑥)
42𝑥 = 270 − 3𝑥
45𝑥 = 270
𝑥=6
Hallando la suma de los perímetros de los polígonos PBQ y APQC (en cm)
Suma de perímetros: ∆𝑃𝐵𝑄 + ∎𝐴𝑃𝑄𝐶
Suma de perímetros: 3𝑥 + 90 − 𝑥
Suma de perímetros: 3(6) + 90 − 6
Suma de perímetros: 18 + 84
Suma de perímetros: 102 cm
Por lo tanto, La suma de los perímetros de los polígonos PBQ y APQC es 102 cm.
8. Dada la expresión:
𝑎𝑥 + 𝑏 𝑎
𝑃( )= 𝑥
𝑎𝑥 − 𝑏 𝑏
Halle el valor de: 𝑃(3) ∙ 𝑃(5) ∙ 𝑃(7) ∙ … ∙ 𝑃(2019)
Solución:
𝑎𝑥+𝑏
Hacemos 𝑚 = , despejamos x obtendremos,
𝑎𝑥−𝑏
𝑚(𝑎𝑥 − 𝑏) = 𝑎𝑥 + 𝑏
𝑎𝑥𝑚 − 𝑚𝑏 = 𝑎𝑥 + 𝑏
𝑎𝑥𝑚 − 𝑎𝑥 = 𝑚𝑏 + 𝑏
𝑥(𝑎𝑚 − 𝑎) = 𝑏(𝑚 + 1)
𝑏(𝑚 + 1)
𝑥=
𝑎(𝑚 − 1)
Reemplazamos y simplificamos obtenemos;
𝑚+1
𝑃(𝑚) =
𝑚−1
Para m = 3; y = 2
m = 5; y =3/2
m = 7; y = 4/3
m = 2017; y = 1009/1008
m = 2019; y = 1010/1009

3 4 1009 1010
𝑃(3) ∙ 𝑃(5) ∙ 𝑃(7) ∙ … ∙ 𝑃(2019) = 2 ( ) ( ) … ( )( ) = 1010
2 3 1008 1009
Po lo tanto, la respuesta es 1010.
9. Daniel escribirá un número en cada casilla del dibujo que se muestra. Ya escribió dos de los
números. Él quiere que la suma de todos los números sea 35, que la suma de los números en las
tres primeras casillas sea 22, y que la suma de los números en las últimas tres casillas sea igual
a 25. ¿Cuál es el producto de los números que escribirá en las casillas sombreadas?

Solución:
Sean a, b y c los números que aparecen en las casillas vacías, escritos de izquierda a derecha.
Tenemos que:
3 + 𝑎 + 𝑏 = 22 (1)
𝑏 + 𝑐 + 4 = 25 (2)
22 + 𝑐 + 4 = 35 (3)
De la tercera igualdad tenemos:
𝑐 = 35 − 26
𝑐=9
Sustituyendo este valor en (2):
𝑏 + 9 + 4 = 25
𝑏 = 25 − 13
𝑏 = 12
Sustituyendo este valor en (1)
3 + 𝑎 + 12 = 22
𝑎 = 22 − 15
𝑎=7
Luego el producto de las casillas sombreadas corresponde: a y c, entonces: 7 × 9 = 63
10. Hay 140 casas en el pueblo de Axel, que suman 1,2,3 … .140. El primer día Axel visitó y tomó
chocolates de las casas que mostraban los números 4,8,12 . .. (números que son divisibles por
4). Al día siguiente comió fideos en las casas que contenían números 10,20,30 . .. (números que
son divisibles por 10). Luego fue a esas casas a las que nunca había ido y tomó un helado. Ahora
diga, ¿en cuántas casas podría comer helado?
Solución
Primero tomó chocolates de las casas 4,8,12 . . . 140. Cuando divide estos números entre 4,
obtiene 1,2,3 . . . 35. Hasta ahora fue a 35 casas y 7 de esas casas son divisibles por 10.
Luego va a las casas de fideos 10,20,30 . . . 140 y va a 14 casas. Pero ya fue a
20, 40, 60, 80, 100, 120 y 140. Así que básicamente va a solo 7 casas nuevas.
Hasta ahora fue a 35 + 7 = 42 casas diferentes hasta ahora. Como hay 140 casas,
140 − 42 = 98. Fue a 98 casas a tomar helado.

11. Un libro tiene 136 páginas. Cada página tiene el mismo número de palabras, y cada página no
tiene más de 100 palabras. Al dividir el número de páginas entre 203 el residuo es 184.
¿Cuántas palabras hay en cada página?
Solución
136𝑥 ≡ 184 (𝑚ó𝑑 203)
17𝑥 ≡ 23 (𝑚ó𝑑 203)
17𝑥 ≡ 23 + 6(203) (𝑚ó𝑑 203)
17𝑥 ≡ 1241 (𝑚ó𝑑 203)
𝑥 ≡ 73 (𝑚ó𝑑 203)
En cada página hay 73 palabras.
12. En una reunión hay 40 personas entre hombres y mujeres, de los hombres la quinta parte están
bebiendo café y la séptima parte están tomando leche. ¿Cuántas mujeres hay en la reunión?
Solución
Si en la reunión hay 40 personas, entonces:
H + M = 40 donde: 0 < H < 40
0 < M < 40
Luego:
 La quinta parte de los hombres están bebiendo café, entonces: “H” es múltiplo de 5 (H =5̇).
 La séptima parte de los hombres están tomando leche, entonces: “H” es múltiplo de 7 (H =7̇).
0 < 𝐻 < 40
De las conclusiones: { 𝐻 = 5̇ H= 35
𝐻 = 7̇
Finalmente:
H + M = 40 → 35 + M = 40
M=5
13. Si la suma de los cuadrados de tres números impares, positivos y consecutivos es 155, halle la
suma de los tres números.
Solución:
Recordemos que:
(𝑎 + 𝑏)2 = 𝑎2 + 2𝑎𝑏 + 𝑏 2
(𝑎 − 𝑏)2 = 𝑎2 − 2𝑎𝑏 + 𝑏 2
Sean los números, 𝑛 − 2; 𝑛; 𝑛 + 2 (n: impar)
Por tanto
(𝑛 − 2)2 + 𝑛2 + (𝑛 + 2)2 = 155
→ 𝑛2 − 4𝑛 + 22 + 𝑛2 + 𝑛2 + 4𝑛 + 22 = 155
3𝑛2 + 8 = 155
3𝑛2 = 155 − 8
147
𝑛2 =
3
√𝑛 = √49
2

𝑛=7
Los números impares son 5,7 y 9
Por tanto la suma es 21.
14. Encuentra todos los números naturales de tres dígitos abc (a≠0) tales que a2 +b2 +c2 es divisor
de 26.
Solución:
S= a2 + b2 +c2 debe ser 1, 2, 13, 0, 26, con S= 1 se tiene el 100, con S=2 se tiene 101 y 110 con
S=13 se tiene 203, 230, 302 y 320 con S=26 se tiene 134, 143, 314, 341, 413, 431,105, 150, 501
y 510.
15. Cerca de una carretera hay cuatro pueblos M, N, P y Q, donde cada uno de ellos está unido a la
carretera por medio de un camino.

Utilizando exclusivamente los caminos y la carretera, sabemos que:


 Para ir de M a N se recorren 1,2 km.
 Para ir de M a P se recorren 1,8 km.
 Para ir de N a P se recorren 1,2 km.
 Para ir de N a Q se recorren 1,4 km.
¿Cuántos kilómetros se recorren para ir de M a Q?
Solución:
Asignando variables a los caminos:
Nos piden hallar la distancia para ir de M a Q, sea: x = a + b + d + f + g.
Para ir de M a P se recorren 1,8 km.
𝑎 + 𝑏 + 𝑑 + 𝑒 = 1,8 … (1)
Para ir de N a P se recorren 1,2 km.
𝑐 + 𝑑 + 𝑒 = 1,2 … (2)
Para ir de N a Q se recorren 1,4 km.
𝑐 + 𝑑 + 𝑓 + 𝑔 = 1,4 … (3)
Hallando (3) – (2)
𝑐 + 𝑑 + 𝑓 + 𝑔 = 1,4 … (3)
−𝑐 − 𝑑 − 𝑒 = −1,2 … (2)
𝑓 + 𝑔 – 𝑒 = 0,2
𝑓 + 𝑔 – 0,2 = 𝑒
Reemplazando “e” en (1)
𝑎 + 𝑏 + 𝑑 + 𝑒 = 1,8
𝑎 + 𝑏 + 𝑑 + 𝑓 + 𝑔 – 0,2 = 1,8
𝑎 + 𝑏 + 𝑑 + 𝑓 + 𝑔 = 1,8 + 0,2
𝑎 + 𝑏 + 𝑑 + 𝑓 + 𝑔 = 2
𝑥 = 2
Se recorren 2 km para ir de M a Q.
16. En la figura mostrada ABCDE y AFGHI son pentágonos regulares y ABF es un triángulo
equilátero. Calcule la medida del ángulo FCH.

Solución:
Calculando la medida del ángulo interior del pentágono regular, tenemos que:
180(𝑛 − 2) 180(5 − 2)
∡𝑖 = = = 36(3) = 108°
𝑛 5
Cada uno de los ángulos interiores del triángulo equilátero mide 60°.
Trazamos los segmentos CF y CH en la figura y se tiene:
El ∆𝐶𝐵𝐹 es isósceles, donde BC = BF, sea ∡𝐵𝐶𝐹 = 𝛼 y como es isósceles ∡𝐵𝐹𝐶 = 𝛼.

𝛼 + 𝛼 + 168° = 180°
2𝛼 = 180° − 168°
12
𝛼= = 6°
2
Trazar los segmentos CA y HA. Sea ∡𝐵𝐴𝐹 = 60° y sea ∡𝐻𝐼𝐴 = 108°.

El ∆𝐻𝐴𝐼 es isósceles, donde AI = HI, sea ∡ 𝐻𝐴𝐼 = 𝜃 y como es isósceles ∡ 𝐼𝐻𝐴 = 𝜃.


𝜃 + 𝜃 + 108° = 180°
2𝜃 = 180° − 108°
72
𝜃= = 36°
2
Si ∡ 𝐻𝐴𝐼 = 36°, entonces ∡ 𝐹𝐴𝐻 = 72°. ∆𝐻𝐴𝐼 ≅ ∆𝐶𝐴𝐵, de manera que ∡ 𝐶𝐴𝐵 = 36°.

El ∆𝐶𝐴𝐻 es isósceles, donde CA = HA, ∡ 𝐶𝐴𝐻 = 36° + 60° + 72° = 168°, sea ∡ 𝐴𝐶𝐻 = 𝛽 y
como es isósceles ∡ 𝐶𝐻𝐴 = 𝛽.

𝛽 + 𝛽 + 1608° = 180°
2𝛽 = 180° − 168°
12
𝛽= = 6°
2
El ∆𝐶𝐴𝐵 es isósceles, donde BC = BA, ∡ 𝐵𝐶𝐴 = 6° + 6° + 𝑥 = 12° + 𝑥.
12° + 𝑥 = 36°
𝑥 = 36° − 12° = 24°
Por lo tanto, ∡ 𝐹𝐶𝐻 = 24°
17. Se tiene un cubo ABCD-EFGH, como se muestra en la figura. Un plano
corta a las aristas AE, BF, CG y DH en los puntos P, Q, R y S,
respectivamente. Se sabe que 𝐸𝑃 = 11, 𝐹𝑄 = 24 y 𝐺𝑅 = 20, calcule la
longitud de HS.
Solución:
Por propiedad los lados opuestos del plano PQRS que cortan al cubo tienen la misma longitud,
es decir: 𝑃𝑄 = 𝑆𝑅 = 𝑛. Sea “m” la medida de la arista del cubo ABCD-EFGH y sea 𝐻𝑆 = 𝑥.
Vamos a graficar la cara ABFE (cuadrado) del cubo:

Trazamos el segmento PT paralelo a EF, de manera que 𝐸𝐹 = 𝑃𝑇 = 𝑚, si 𝐸𝑃 = 11, entonces


𝑇𝐹 = 11, si 𝐹𝑄 = 24, entonces 𝑄𝑇 = 13. Utilizando el teorema de Pitágoras en el ∆𝑃𝑄𝑇.

𝑛2 = 𝑚2 + 132
𝑛 − 𝑚2 = 132 (1)
2

Ahora vamos a graficar la cara DCGH (cuadrado) del cubo:

Trazamos el segmento SU paralelo a HG, de manera que 𝑆𝑈 = 𝐻𝐺 = 𝑚, si 𝑆𝐻 = 𝑥, entonces


𝐺𝑈 = 𝑥, si 𝐺𝑅 = 20, entonces 𝑅𝑈 = 20 – 𝑥. Utilizando el teorema de Pitágoras en el ∆𝑆𝑈𝑅.
𝑛2 = 𝑚2 + (20 − 𝑥)2
𝑛2 − 𝑚2 = (20 − 𝑥)2 (2)
Igualando (1) y (2), tenemos que:
132 = (20 − 𝑥)2
√132 = √(20 − 𝑥)2
13 = 20 − 𝑥
𝑥 + 13 = 20
𝑥 = 20 − 13
𝑥=7
Por lo tanto, la longitud de HS es 7
18. En la figura se muestra dos semicircunferencias de diámetros AC y BD, que se intersecan en P.
Si AB = 3 y BC = 2, calcule la longitud de CD.

Solución:
Ubicando los centros de las dos circunferencias E y F, luego trazamos los segmentos PE y PF.

Sea 𝐶𝐸 = 𝑥, entonces el radio de la circunferencia grande mide 𝐵𝐸 = 𝑃𝐸 = 2 + 𝑥, el radio de


3+2 5
la circunferencia pequeña mide: 2 = 2 = 2,5, en consecuencia 𝐹𝐵 = 0,5; también 𝐹𝑃 = 2,5.
El ∆𝐹𝑃𝐶 es isósceles (FP = FC), trazamos la altura FG.

Sea ∡𝐹𝑃𝐵 = 𝛼. En el ∆𝐹𝑃𝐺, ∡𝐹𝑃𝐺 = 𝛼 + 45°, de manera que ∡𝑃𝐹𝐺 = 90° − (𝛼 + 45°) =
90° − 𝛼 − 45° = 45° − 𝛼 y como el ∆𝐹𝑃𝐶 es isósceles, ∡𝐺𝐹𝐶 = 45° − 𝛼. En el ∆𝐹𝑃𝐵,
utilizando la propiedad de que la suma de dos ángulos interiores es igual al ángulo exterior del
tercer vértice:
∡𝑃𝐵𝐶 = ∡𝐹𝑃𝐵 + ∡𝑃𝐹𝐵
∡𝑃𝐵𝐶 = 𝛼 + 45° − 𝛼 + 45° − 𝛼
∡𝑃𝐵𝐶 = 90° − 𝛼
El ∆𝑃𝐸𝐵 es isósceles (𝑃𝐸 = 𝐵𝐸) y se cumple que:
∡𝑃𝐵𝐸 = ∡𝐵𝑃𝐸
90° − 𝛼 = 45° + ∡𝐶𝑃𝐸
∡𝐶𝑃𝐸 = 45° − 𝛼

∡𝐹𝑃𝐸 = 𝛼 + 45° + 45° − 𝛼 = 90°

Hallando "𝑥" en el ∆𝑃𝐸𝐹 utilizando el teorema de Pitágoras:

2,52 + (2 + 𝑥)2 = (2,5 + 𝑥)2


6,25 + 4 + 4𝑥 + 𝑥 2 = 6,25 + 5𝑥 + 𝑥 2
10,25 + 4𝑥 = 6,25 + 5𝑥
𝑥=4
De donde se tiene que:
𝐶𝐷 = 𝑥 + 2 + 𝑥 = 2𝑥 + 2 = 2(4) + 2 = 10
19. El cuadrilátero de la figura es un cuadrado. Si los puntos E y F están sobre los lados BC y CD
respectivamente, tal que BE = 4 y DF = 6, AE es bisectriz del ángulo ∠BAF. Determine el área
del cuadrilátero AECF.

Solución: Tracemos el segmento 𝐸𝐹, formando el triángulo 𝐴𝐸𝐹, al cual le trazaremos la altura
relativa al lado 𝐴𝐹, tomando los datos llamemos 𝑎 al lado del cuadrado, como 𝐵𝐸 = 4 y
𝐷𝐹 = 6, 𝐴𝐸 es bisectriz del ángulo ∠𝐵𝐴𝐹, 𝐸𝐶 = 𝑎 − 4 y 𝐶𝐹 = 𝑎 − 6, ∠𝐵𝐴𝐸 = ∠𝐸𝐴𝐻 = 𝜃,
por la bisectriz 𝐴𝐸, a como se muestra en la figura siguiente.

De la figura podemos decir que:∠𝐵𝐴𝐸 = ∠𝐸𝐴𝐻 = 𝜃, 𝑚∠𝐴𝐵𝐸 = 𝑚∠𝐴𝐻𝐸 = 90°, por tanto,
∠𝐴𝐸𝐻 = ∠𝐴𝐸𝐵, por propiedad reflexiva 𝐴𝐸 = 𝐴𝐸, por tanto.
∆𝐴𝐵𝐸 ≅ ∆𝐴𝐻𝐸
Dado que tenemos triángulos, entonces𝐵𝐸 = 𝐸𝐻 = 4.
 El área del cuadrilátero [𝐴𝐸𝐶𝐹] = [𝐴𝐵𝐶𝐷] − ([𝐴𝐵𝐸] + [𝐴𝐷𝐹]).
 [ABCD] = a2 ; [ABE] = 2a y [ADF] = 3a, entonces:
 [AECF] = 𝑎2 − (2𝑎 + 3𝑎) = 𝑎2 − 5𝑎.
También [𝐴𝐸𝐶𝐹] = [𝐴𝐸𝐹] + [𝐸𝐶𝐹]

 Calculamos (AF)2 = a2 + 62 ⟹ AF = √a2 + 36


4(√a2 +36) (a−4)(a−6)
 [AEF] = = 2(√a2 + 36) y [ECF] =
2 2
(a−4)(a−6)
 [AECF] = 2(√a2 + 36) + , pero [AECF] = a2 − 5a, entonces
2
(a−4)(a−6) a2 −10a+24
 a2 − 5a = 2(√a2 + 36) + ⟹ a2 − 5a − = 2(√a2 + 36)
2 2
 2a2 − 10a − a2 + 10a − 24 = 4(√a2 + 36) ⟹ a2 − 24 = 4(√a2 + 36), elevando al
cuadrado en ambos lados.
 a4 − 48a2 + 576 = 16(a2 + 36) ⟹ a4 − 48a2 + 576 = 16a2 + 576
 a4 − 64a2 = 0 ⟹ a2 (a2 − 64) = 0, se tiene que cumplir que a > 0, entonces:
 a2 − 64 = 0 ⟹ 𝑎 = 8
Por tanto, el área del cuadrilátero es [𝐴𝐸𝐶𝐹] = 82 − 5(8) = 24
20. Sea 𝑝(𝑛) el producto de las cifras de un número natural n. Por ejemplo, 𝑝(7) = 7 y
𝑝(25) = 2 ∙ 5 = 10. Determine el valor de la suma 𝑝(1) + 𝑝(2) + ⋯ + 𝑝(100).
Solución:
Antes que nada, notemos que los múltiplos de 10 no contribuyen en nada a la suma, así que
podemos ignorarlos. Claramente 𝑝(1) + 𝑝(2) + ⋯ + 𝑝(9) = 1 + 2 + ⋯ + 9. El siguiente
paso es sumar 𝑝(11) + 𝑝(12) + ⋯ + 𝑝(19) = 1 ∙ 1 + 1 ∙ 2 + ⋯ + 1 ∙ 9 = 1(1 + 2 + ⋯ + 9)
Para los números del 21 al 29 tenemos que:
𝑝(21) + 𝑝(22) + ⋯ + 𝑝(29) = 2 ∙ 1 + 2 ∙ 2 + ⋯ + 2 ∙ 9 = 2(1 + 2 + ⋯ + 9):
Y así sucesivamente hasta sumar 𝑝(91) + 𝑝(92) + ⋯ + 𝑝(99). Por tanto
𝑝(1) + 𝑝(2) + ⋯ + 𝑝(100) = (1 + 2 + ⋯ + 9) + (1 + 2 + ⋯ 9)2 = 45 + 452 = 2070
21. La pirámide siguiente se llena colocando un número entero en cada casilla. Los cuatro números
de la base deben cumplir que al sumarlos de como resultado 8, y los números de las demás
casillas deben ser el resultado de sumar los números en las dos casillas que están justo debajo.
Determine si es posible que el número en la casilla superior sea 7.

Solución:
No. Si 𝑎; 𝑏; 𝑐; 𝑑 son los números de la base escritos de izquierda a derecha, entonces los tres
números arriba son 𝑎 + 𝑏; 𝑏 + 𝑐; 𝑐 + 𝑑, y los dos números siguientes son 𝑎 + 2𝑏 + 𝑐;
𝑏 + 2𝑐 + 𝑑, por lo que el número en la cima es 𝑎 + 3𝑏 + 3𝑐 + 𝑑. Como 𝑎 + 𝑏 + 𝑐 + 𝑑 = 8
este número vale 8 + 2𝑏 + 2𝑐, que es par y por tanto no puede ser 7.
22. Varios miembros de la familia González estaban reunidos en una fiesta y se dieron cuenta que
la suma de sus edades era exactamente 300. Además, notaron que la última vez que se habían
reunido, hace algunos años, sus edades sumaban 278; y que si se volvían a reunir para celebrar
los 60 años del abuelo dentro de algunos años, sus edades sumarían 333. ¿Cuál es la cantidad
de personas que estaban reunidas en la fiesta?
Solución:
Cada año que pasa, la suma total de las edades aumenta un número igual a la cantidad de
personas que estaban reunidas. Como dentro de algunos años la suma habrá aumentado 33, el
número de personas debe ser divisor de 33. Como anteriormente la suma era 22 años menos,
también debe ser divisor de 22. Los únicos divisores comunes de 22 y 33 son 1 y 11, por lo que
la respuesta correcta es 11.

23. En la figura adjunta se muestra un triángulo ABC y puntos


D y E en sus lados tales que 𝐴𝐵 = 𝐵𝐷 = 𝐴𝐸 y 𝐷𝐸 = 𝐷𝐶. Si
se sabe que ∡𝐵𝐴𝐸 = 60° entonces cual es la medida ∠ 𝐸𝐷𝐶.
Solución:

Como 𝐴𝐵 = 𝐴𝐸, ∠ 𝐴𝐵𝐸 𝑦 ∠ 𝐴𝐸𝐷 tienen igual medida y


como ∡ 𝐵𝐴𝐸 = 60°, entonces ∡ 𝐴𝐵𝐸 = ∡ 𝐴𝐸𝐵 = 60°, entonces 𝐵𝐸 = 𝐴𝐵 = 𝐴𝐸 = 𝐵𝐷.

Por otro lado ∡ 𝐷𝐸𝐶 = ∡ 𝐷𝐶𝐸 = 𝛼, debido a que 𝐷𝐸 = 𝐷𝐶. Por el teorema del ángulo externo,
∡ 𝐵𝐷𝐸 = 2𝛼. Además ∡ 𝐵𝐷𝐸 = ∡ 𝐵𝐸𝐷, por que 𝐵𝐸 = 𝐵𝐷. Como ∡ 𝐴𝐸𝐵 + ∡ 𝐵𝐸𝐷 +
∡ 𝐷𝐸𝐶 = 180°, se tiene que 60° + 2𝛼 + 𝛼 = 180°, por lo que 𝛼 = 40°. Finalmente
∡ 𝐸𝐷𝐶 = 100°
24. La sucesión comienza con 𝑎1, 𝑎2 , 𝑎3 , ⋯, comienza con 𝑎1 = 49. Para 𝑛 ≥ 1, el término 𝑎𝑛+1 se
obtiene añadiendo 1 a la suma de las cifras de 𝑎𝑛 y elevando al cuadrado el resultado. Por
ejemplo: 𝑎2 = (4 + 9 + 1)2 = 196, ¿Cuánto vale 𝑎2020 ?
Solución:
Obteniendo otros términos a partir del 𝑎2 , tenemos que:
𝑎3 = (1 + 9 + 6 + 1)2 = 172 = 289
𝑎4 = (2 + 8 + 9 + 1)2 = 20 = 400
𝑎5 = (4 + 0 + 0 + 1)2 = 52 = 25
𝑎6 = (2 + 5 + 1)2 = 82 = 64
𝑎7 = (6 + 4 + 1)2 = 112 = 121
𝑎8 = (1 + 2 + 1 + 1)2 = 52 = 25
𝑎9 = (2 + 5 + 1)2 = 82 = 64
𝑎10 = (6 + 4 + 1)2 = 112 = 121
𝑎11 = (1 + 2 + 1 + 1)2 = 52 = 25
Como podemos observar a partir del 𝑎5 hay un ciclo de repetición de los números cada 3
números, como la secuencia es de 2020 números, si le restamos la cantidad de números que no
están en el ciclo los cuales son 4 números, entonces tenemos: 2020 − 4 = 2016.
Dividiendo 2016 entre la cantidad del ciclo tendremos el número que ocupa la posición 2020
2016 = 3(672) + 0
Por lo tanto, como la división es exacta el último número que ocupa la tercera posición del ciclo
también ocupara la posición 2020 de la sucesión, por lo tanto, este número es 121.
25. En el rectángulo ABCD se han dibujado los arcos de tres circunferencias con centros en los
vértices A, B y C, como se muestra en la gráfica y cada arco es un cuarto de circunferencia. La
circunferencia con centro en A tiene un radio de 3 y la circunferencia con centro en C tiene un
radio de 1. ¿Cuál es el área de ABCD?

Solución:
Veamos que AD es radio en la circunferencia que tiene centro en A, por lo que AD = 3, luego
BC = 3, por lo que el radio de la circunferencia con centro en B es 2 y así AB = 3 + 2 = 5, por
lo que (ABCD) = AB · AD = 5(3) = 15.

26. Se escriben en la pizarra todos los enteros positivos desde el 1 al 2020. Después, al azar se
borran dos números, y se los sustituye por su diferencia. Al final de este proceso, solo queda un
número. ¿Puede ser el 3?
Solución:
2020+2021
La suma de todos los números es = 1010 ∙ 2021 que es un número par.
2

Luego al quitarle dos números cualesquiera, representados por a y b tenemos:


2021 ∙ 1010 − 𝑎 − 𝑏
y finalmente se los sustituye por su diferencia, es decir:
2021 ∙ 1010 − 𝑎 − 𝑏 + (𝑎 − 𝑏) = 2021 ∙ 1010 − 2𝑏
que también es un número par, con lo cual vemos que al final el único número que puede quedar
es un número par, y por lo tanto es imposible que se tenga al final el número 3.

27. La abuela de Andrea olvidó la clave de 5 dígitos para abrir su maleta y solo recuerda que:
 no tiene ningún cero,
 tiene tres dígitos que son múltiplos de 4,
 tiene dos dígitos que son múltiplos de 3,
 no tiene dígitos consecutivos iguales.
¿Cuántas son las posibles claves?
Solución:
Si en la clave deben existir 3 múltiplos de cuatro, estos números solo pueden ser 4; 4; 8; 8; 8; 4;
4; 4; 4; 8; 8; 8, con lo cual solo analizaremos la mitad de los casos.
Imaginemos que solo existen 2 cuatros y 1 ocho, por lo tanto, solo podemos ubicar de 6 maneras
diferentes 2 cuatros y de 1 forma 3 cuatros:

En cada caso vamos a ubicar un 8 en alguna casilla, con lo cual quedaran dos casillas libres, en
las cuales debemos escribir los dos múltiplos 3 bajo la consideración que si las casillas están
juntas solo podemos escribir 6 números, y si las casillas están separadas podemos escribir 9
números.
Ahora ilustraremos todos los casos, teniendo en cuenta que el caso 1 e igual al caso 6, así como
el caso 2 e igual al caso 5, por ser simétricos.

Caso 1 y 6 tenemos: 2(6 + 9 + 9) = 2(24) = 48 claves posibles.


Caso 2 y 5 tenemos: 2(6 + 9 + 9) = 2(24) = 48 claves posibles.
Caso 3 tenemos: 6 + 6 + 9 = 21 claves posibles.
Caso 4 tenemos: 9 + 9 + 9 = 27 claves posibles.
Caso 7 tenemos: 9 claves posibles.
Puesto que solo hemos analizado la mitad de los casos, entonces la cantidad total de claves es
2(48 + 48 + 21 + 27 + 9) = 2(153) = 306
28. Sea N un número de cuatro dígitos. Si N + 25 es un múltiplo de 8, encuentra el menor valor
posible de N.
Solución:
El menor entero de cuatro dígitos es 1000, que también es múltiplo de 8, puesto que se le puede
extraer tres veces la mitad, con lo cual podemos ahora escribir:
Si 𝑁 + 25 = 1000, entonces 𝑁 = 1000 − 25 = 975
Si 𝑁 + 25 = 1008, entonces 𝑁 = 1008 − 25 = 983
Si 𝑁 + 25 = 1016, entonces 𝑁 = 1016 − 25 = 991
Si 𝑁 + 25 = 1024, entonces 𝑁 = 1024 − 25 = 999
Si 𝑁 + 25 = 1032, entonces 𝑁 = 1032 − 25 = 1007
Por lo tanto, en menor número de 4 dígitos que puede tomar N es 1007.

29. Hallar todos los números naturales de 4 cifras, escritos en base 10, que sean iguales al cubo de
la suma de sus cifras.
Solución:
Sea n un número verificando el enunciado, y s la suma de sus cifras.
Como 1000  n  9999 y n = s3, resulta
11  s  21 (1)
Si 𝑛 = 𝑥𝑦𝑧𝑡, tenemos:
1000x + 100y + 10z + t = s3 (2)
x+y+z+t=s
Restando queda:
999x + 99y + 9z = s3 - s (3)
Cuyo segundo miembro ha de ser múltiplo de 9 (por serlo el primero) y, habida cuenta de que
s3 - s = (s - 1) s (s + 1)
Y por (1), sólo hay tres valores de s3 - s que son múltiplos de 9: 16·17·18; 17·18·19 y 18·19·20
Sustituimos en (3) y analizamos cada caso.
 999x + 99y + 9z = 16·17·18  111x + 11y + z = 544
Resulta inmediatamente x = 4; y = 9; z = 1, valores que llevados a (2) con s = 17 se obtiene t =
3 y finalmente n = 4913
 999x + 99y + 9z = 17·18·19  111x + 11y + z = 646
De donde x = 5; y = 8; z = 3, valores que llevados a (2) con s = 18 se obtiene t = 2 y finalmente
n = 5832
 999x + 99y + 9z = 18·19·20  111x + 11y + z = 760
Resulta x = 6; y = 8; z = 6, valores que llevados a (2) con s = 19 resulta una contradicción.
 Resumiendo, las únicas soluciones son 4913 y 5832

30. Una sucesión pucelana es una sucesión creciente de dieciséis números impares positivos
consecutivos, cuya suma es un cubo perfecto. ¿Cuantas sucesiones pucelanas tienen solamente
números de tres cifras?
Solución:
1
Sea la sucesión 𝑛, 𝑛 + 2, . . . , 𝑛 + 30. Entonces la suma es 2 16(2n + 30) = 8(2n + 30). Por tanto,
es necesario que 2n + 30 sea un cubo perfecto. Ahora hay que contar el número de tales n que
son impares y verifican 101 ≤ n ≤ 969. Los cubos pares entre 232 y 1968 son 512, 1000 y 1728,
que corresponden a valores de n de 241, 485 y 849. Por lo tanto, hay exactamente Dos sucesiones
pucelanas.
Nivel V (Noveno Grado)
1. En la figura de la derecha se muestran dos rectángulos ABCD y APQR
tales que 𝐴𝐵 = 𝐴𝑃 = 2 y 𝐵𝐶 = 𝑃𝑄 = 4. Determine la medida del
ángulo DRQ.

Solución:
Completando los datos de acuerdo al problema planteado:
Sea ∡𝐷𝑅𝑄 = 𝑥

El triángulo APD es rectángulo y vamos hallar el lado


PD utilizando el Teorema de
Pitágoras:

El triángulo APD es notable cuyos lados proporcionales corresponden a los lados de un triángulo
de 30° y 60°.

Vamos a completar los demás ángulos para hallar el ángulo 𝑥.

∡𝑅𝐴𝐷 = 30°, porque es el complemento del ángulo PAD.


El triángulo RAD es isósceles (𝐴𝑅 = 𝐴𝐷 = 4). Si ∡𝑅𝐴𝐷 = 30°, entonces ∡𝐴𝑅𝐷 = ∡𝑅𝐷𝐴 =
75°.
Finalmente ∡𝐴𝑅𝐷 y 𝑥 son complementarios, por lo que se cumple que:

∡𝐴𝑅𝐷 + 𝑥 = 90°
75° + 𝑥 = 90°
𝑥 = 15°
Por lo tanto la medida del angulo DRQ es 15º
2. Si 2018 = 𝑎4 + 𝑏 4 + 𝑐 4 + 𝑑 4 , donde 𝑎, 𝑏, 𝑐 𝑦 𝑑, son números enteros positivos distintos.
Calcule el valor de 𝑎 + 𝑏 + 𝑐 + 𝑑.
Solución:
Hallando la cuarta potencia de los primeros números enteros positivos tenemos que
14 =1
24 = 16
34 = 81
44 = 256
54 = 625
64 = 1296
74 = 2401 (Este valor queda descartado porque es mayor que 2018)
Entonces como los números son distintos, vamos a sumar algunos de ellos de tal forma que
obtengamos como resultado 2018, obteniendo la siguiente combinación de números:
16 + 81 + 625 + 1296 = 2018
24 + 34 + 54 + 64 = 2018
Por lo tanto la suma de los números es: 2 + 3 + 5 + 6 = 16, entonces el valor de a + b + c +
d es 16.

3. Sean 𝑎 𝑦 𝑏 números reales tales que 8𝑎 ∙ 3𝑏 = 78 y 2𝑎 ∙ 9𝑏 = 76 . Calcule el valor de 2𝑎 .


Solución:
Despejando 3𝑏 de la primera ecuación, tenemos que:

8𝑎 ∙ 3𝑏 = 78
3𝑏 = 78 ∙ 8−𝑎
Elevando al cuadrado la expresión obtenida anteriormente:
(3𝑏 )2 = (78 ∙ 8−𝑎 )2
(32 )𝑏 = (78 )2 ∙ (8−𝑎 )2
9𝑏 = 716 ∙ 8−2𝑎
Reemplazando 9𝑏 en la segunda ecuación tenemos que:
2𝑎 ∙ 9𝑏 = 76
2𝑎 ∙ (716 ∙ 8−2𝑎 ) = 76
2𝑎 ∙ 8−2𝑎 = 76 ∙ 7−16
2𝑎 ∙ (23 )−2𝑎 = 7−10
2𝑎 ∙ 2−6𝑎 = 7−10
2−5𝑎 = 7−10
−5𝑎 −10
2 −5 = 7 −5
2𝑎 = 72
2𝑎 = 49
Por lo tanto el valor de 2𝑎 es 49
4. Un cuadrado como el mostrado en la figura de la derecha, está dividido en 4
rectángulos iguales y un cuadrado, como se muestra en la figura, Si el área del
cuadrado sombreado es 36 cm2 y el área de cada uno de los rectángulos iguales
es 720 cm2, calcular las longitudes de los lados de los rectángulos.

Solución:

Calcular primero el área del cuadrado grande, luego su lado y el lado del cuadradito sombreado.

El área de cada rectángulo es 720 cm2.


Entonces el área de los 4 rectángulos es 4 (720 cm2) = 2880 cm2.
El área del cuadrado sombreado es 36 cm2.
El área del cuadrado grande es igual al área de los 4 rectángulos más el área del cuadrado
sombreado: 2880 cm2 + 36 cm2 = 2916 cm2.
Del área del cuadrado grande conocemos el lado del cuadrado de la siguiente manera:

𝐴 = 𝑙2
𝑙 = √𝐴
𝑙 = √2916 𝑐𝑚2 = 54 𝑐𝑚

La medida del lado del cuadrado es igual a la bases más la altura de un rectángulo, es decir:

𝑙 = 𝑏 + ℎ = 54
Además el área del rectángulo es:

𝐴 = 𝑏ℎ = 720 𝑐𝑚2
Formándose el siguiente sistema de ecuaciones:

𝑏 + ℎ = 54 (1)
{
𝑏ℎ = 720 (2)

Despejando h en la primera ecuación y sustituyéndola en la segunda tenemos que:


ℎ = 54 − 𝑏 (3)
𝑏(54 − 𝑏) = 720
54𝑏 − 𝑏 2 = 720
𝑏 2 − 54𝑏 + 720 = 0
(𝑏 − 30)(𝑏 − 24) = 0
𝑏 = 30; 𝑏 = 24
Sustituyendo los valores obtenidos en la ecuación (3), tenemos que: Si 𝑏 = 30, ℎ = 24.
Si 𝑏 = 24, ℎ = 30
Por lo tanto, los lados del rectángulo miden 24 de ancho y 30 de largo.
5. Sea x1 y x2 raíces de la ecuación x 2 + x − 3 = 0. Encuentre el valor de: x13 − 4x22 + 19
Solución:
Por el teorema de Viete
𝑠: 𝑥1 + 𝑥2 = −1
{
𝑝: 𝑥1 ∙ 𝑥2 = −3

Sea 𝐴 = 𝑥13 − 4𝑥22 + 19, 𝐵 = 𝑥23 − 4𝑥12 + 19 luego

𝐴 + 𝐵 = (𝑥13 + 𝑥23 ) − 4(𝑥12 + 𝑥22 ) + 38


𝐴 + 𝐵 = (𝑥1 + 𝑥2 )[(𝑥1 + 𝑥2 )2 − 3𝑥1 𝑥2 ] − 4[(𝑥1 + 𝑥2 )2 − 2𝑥1 𝑥2 ] + 38
𝐴 + 𝐵 = −(1 + 9) − 4(1 + 6) + 38 = 0
𝐴 − 𝐵 = (𝑥13 − 𝑥23 ) + 4(𝑥12 − 𝑥22 )
𝐴 − 𝐵 = (𝑥1 − 𝑥2 )[(𝑥1 + 𝑥2 )2 − 𝑥1 𝑥2 + 4(𝑥1 + 𝑥2 )]
𝐴 − 𝐵 = (𝑥1 − 𝑥2 )[1 + 3 − 4] = 0
Así, 2𝐴 = (𝐴 + 𝐵) + (𝐴 − 𝐵) = 0, 𝑖, 𝑒, 𝐴 = 0
6. Dados que los enteros a, b satisfacen la ecuación
1 1
1 1 1 2
[ 𝑎 − 𝑏 ]∙( − )∙ =
1 1 1 1 𝑎 𝑏 1 3
𝑎−𝑏 𝑎+𝑏 𝑎2 + 𝑏 2

Encuentra el valor de 𝑎 + 𝑏
Solución:
1 1
Hacer 𝑥 = 𝑎 , 𝑦 = 𝑏 , entonces el lado izquierdo de la ecuación dada se convierte
𝑥 𝑦 1 𝑥2 + 𝑦2 1 1
[ − ] (𝑥 − 𝑦) ∙ 2 = ∙ (𝑥 − 𝑦) ∙ =
𝑥−𝑦 𝑥+𝑦 𝑥 + 𝑦 2 (𝑥 − 𝑦)(𝑥 + 𝑦) 𝑥2 + 𝑦2 𝑥 + 𝑦
Entonces la ecuación dada se simplifica a
1 2
=
𝑥+𝑦 3
1 2 𝑎𝑏 2
= ∨ =
1 1 3 𝑎+𝑏 3
𝑎+𝑏
De esta tenemos que 3𝑎𝑏 − 2𝑎 − 2𝑏 = 0 que produce 9𝑎𝑏 − 6𝑎 − 6𝑏 + 4 = 4
(3𝑎 − 2)(3𝑏 − 2) = 4
Por lo tanto, 𝑎 ≠ 𝑏 Por simetría, podemos suponer 𝑎 > 𝑏, entonces
(i)Cuando 3𝑎 − 2 = 4, 3𝑏 − 2 = 1 luego 𝑎 = 2, 𝑏 = 1, 𝑎 + 𝑏 = 3;
(ii) Cuando 3𝑎 − 2 = −1, 3𝑏 − 2 = −4 entonces a no tiene solucion entera.
Por lo tanto, 𝑎 + 𝑏 = 3
7. Se muestran dos hexágonos regulares, uno dentro del otro. Si los
puntos A, B y C pertenecen a una misma recta y el perímetro del
hexágono mayor es 120 cm, determine el perímetro del hexágono
menor (en cm).
Solución:
Si el perímetro del hexágono mayor es 120 cm, entonces cada lado
mide: 120 ÷ 6 = 20 𝑐𝑚.
Medida del ángulo interior de un hexágono regular es:
180°(𝑛 − 2) 180°(6 − 2)
= = 30°(4) = 120°
𝑛 6
Como son hexágonos regulares, el vértice B es el punto medio del segmento AC.
Como el ∆𝐴𝐷𝐶 es isósceles (𝐴𝐷 = 𝐷𝐶) entonces DB es la altura y bisectriz del ∆𝐴𝐷𝐶.
Entonces ∡𝐶𝐷𝐵 = 60°.

El ∆𝐷𝐶𝐵 es notable cuyos ángulos interiores son 30° y 60°. Utilizando los lados proporcionales
del triángulo notable de 30° y 60°.
Hallado 𝑘
20 = 2𝑘
10 = 𝑘
Por tanto, 𝐵𝐶 = √3𝑘 = 10√3 𝑐𝑚
Como 𝐷𝐶 ∥ 𝐵𝐸, entonces por ángulos alternos internos se cumple: ∡𝐶𝐵𝐸 = 30°.
Vamos a trazar el segmento CE, que une dos vértices de los hexágonos regulares y que CE es
perpendicular un lado del hexágono regular menor.
El ∆𝐵𝐸𝐶 es notable cuyos ángulos interiores son 30° y 60°. Utilizando los lados proporcionales
del triángulo notable de 30° y 60°.

Hallado 𝑝:
2𝑝 = 10√3
𝑝 = 5√3

Por tanto, 𝐵𝐸 = √3𝑝 = √3 × 5√3 = 5√32 = 5 × 3 = 15 𝑐𝑚


Finalmente, el perímetro del hexágono regular menor es: 6(15) = 90 cm.
Por lo tanto, el perímetro del hexágono menor es 90 cm.
8. Se tienen los cuadrados ABCD y WXY Z, con lados de longitud 2 y 1, respectivamente. El lado
WX está sobre el lado DC. Se trazan las rectas DZ y CY, que se cortan en P. Determine la razón
entre el área sombreada y el área blanca.
Solución:
Puesto que CD y YZ son paralelas, los triángulos PY Z y PCD son semejantes, y la razón de
𝑌𝑍 1
semejanza es 𝐶𝐷 = 2; esto último implica que Y y Z son los puntos medios de PC y PD,
respectivamente. Ahora bien, sea PR la altura relativa a CD, y sea S su punto de corte con YZ.
𝑃𝑆 𝑃𝑌 1
Entonces 𝑃𝑅 = 𝑃𝐶 = 2, de donde 𝑃𝑅 = 2 pues RS es congruente a un lado del cuadrado WXYZ,
𝑃𝑅 2
que tiene lado 1. Luego el área del triángulo CDP es 𝐶𝐷 ∙ 2 = 2 ∙ 2 = 2. Al sustraer el área del
cuadrado WXYZ obtenemos el valor del área sombreada, que es 1, y como la región blanca
1
tiene obviamente área 5 se concluye que la razón pedida es 5.
1
9. Sea S el conjunto de todos los enteros positivos 𝑛 tales que 𝑛 tiene la representación decimal
0. ̅̅̅
𝑎𝑏 = 0. 𝑎𝑏𝑎𝑏𝑎𝑏 … con 𝑎 𝑦 𝑏 dígitos distintos. Determine el valor de la suma de todos los
elementos de S.
Solución:
̅̅̅̅
𝑎𝑏
La fórmula para calcular decimales periódicos nos dice que 0. ̅̅̅
𝑎𝑏 = 99 . Luego, si 𝑛 ∈ 𝑆 entonces
1 ̅̅̅̅
𝑎𝑏 99
= 99 para ciertos enteros 𝑎 ≠ 𝑏. Despejando n vemos que 𝑛 = ̅̅̅̅ es entero; esto puede suceder
𝑛 𝑎𝑏
si y sólo si ab es un divisor de 99, es decir ̅̅̅
𝑎𝑏 = 01; 03; 09; 11; 33; 99. Debido a la restricción
99
𝑎 ≠ 𝑏 los únicos casos posibles son 1, 3 y 9; los valores respectivos de 𝑛 son 1 = 99,
99 99
= 33 𝑦 = 11. Por tanto la suma buscada es 99 + 33 + 11 = 143.
3 9

10. Se tienen 11 cajas grandes. Algunas de ellas contienen, cada una, 8 cajas medianas y el resto
están vacías. A su vez, algunas de las cajas medianas contienen, cada una, 8 cajas pequeñas y
el resto están vacías. Todas las cajas pequeñas están vacías. Si hay 102 cajas vacías, determine
el número total de cajas.
Solución:
Si 𝑎 es el número de cajas grandes que contienen cajas medianas, entonces hay 8𝑎 cajas
medianas y 11 − 𝑎 cajas grandes vacías. Asimismo, si 𝑏 es el número de cajas medianas que
contienen cajas pequeñas, entonces hay 8𝑏 cajas pequeñas y 8𝑎 − 𝑏 cajas medianas vacías.
Luego hay 11 − 𝑎 + 8𝑎 − 𝑏 + 8𝑏 = 102 cajas vacías, de donde 𝑎 + 𝑏 = 13. En consecuencia,
el número total de cajas es 11 + 8𝑎 + 8𝑏 = 11 + 8 ∙ 13 = 115

11. Si las longitudes 𝑎, 𝑏 𝑦 𝑐 de los lados de un triángulo satisfacen las condiciones


𝑎 + 𝑏 − 𝑐 = 2 y 2𝑎𝑏 − 𝑐 2 = 4, demuestre que el triángulo es equilátero.
Solución:
La primera condición puede reescribirse como 𝑐 = 𝑎 + 𝑏 − 2, y sustituyendo en la segunda
ecuación obtenemos 2𝑎𝑏 − (𝑎 + 𝑏 − 2)2 = 4. Expandiendo y reordenando queda
0 = 𝑎2 + 𝑏 2 − 4𝑎 − 4𝑏 + 4 = (𝑎 − 2)2 + (𝑏 − 2)2 . Ya que una suma de cuadrados puede ser
cero sólo si cada uno de los cuadrados es cero, se sigue que 𝑎 − 2 = 𝑏 − 2 = 0, así
𝑎 = 𝑏 = 𝑐 = 2. Luego el triángulo es equilátero.

12. Juan Pablo escribe la siguiente sucesión de números:


999 998; 9 999 998; 99 999 998; ⋯ ; 999 ⋯ 98
2020 veces
Luego suma todos los números en la sucesión y a este número le llama S; es decir:
𝑆 = 999 998 + 9 999 998 + 99 999 998 + ⋯ + 999 ⋯ 98
2020 veces
Determine la suma de los dígitos de S
Solución:
Observe que los números se pueden escribir como:

1 000 000 − 2; 10 000 000 − 2; 100 000 000 − 2; ⋯ ; 1000 ⋯ 0 − 2


2020 veces
La sucesión tiene 2020 − 4 = 2016 números. Al hacer la suma, cada 1 ocupa una posición
diferente, y el último 1 a la derecha está en la posición de las unidades de millón, la resta al final
es (2020 − 4) ∙ 2 = 4032, es decir:
𝑆 = 111111 ⋯ 11000000 − 4032 = 111111 ⋯ 110995968
2015 veces
De donde la suma de los dígitos D está dada por 𝐷 = 2015 + 3 ∙ 9 + 5 + 6 + 8 = 2061
̅̅̅̅. Sean D y
̅̅̅̅ ∥ 𝐴𝐵
13. Considere el ∆𝐴𝐵𝐶 y sean E y F puntos, tales que 𝐴 − 𝐸 − 𝐶; 𝐵 − 𝐹 − 𝐶 y 𝐸𝐹
𝐷𝐺 7
G puntos tales que 𝐷 − 𝐸 − 𝐹 − 𝐺. Si ∡ 𝐶𝐴𝐵 = 2 ∡ 𝐷𝐶𝐸, ∡ 𝐹𝐶𝐺 + ∡ 𝐷𝐶𝐸 = ∡ 𝐴𝐵𝐶, 𝐴𝐵 = 3
𝐴𝐶 5 𝐷𝐶
y 𝐷𝐶 = 3. Determine 𝐸𝐹 .

Solución:
Considere la figura:

Sea 𝛼 = ∡ 𝐷𝐸𝐶, entonces ∡ 𝐶𝐴𝐵 = 2𝛼.


Sea 𝛽 = ∡ 𝐴𝐵𝐶, entonces ∡ 𝐹𝐶𝐺 + 𝛼 = 𝛽 ⇒ ∡ 𝐹𝐶𝐺 = 𝛽 − 𝛼
∡ 𝐶𝐸𝐹 = ∡ 𝐶𝐴𝐵 = 2𝛼 y ∡ 𝐶𝐹𝐸 ≤ ∡ 𝐴𝐵𝐶 = 𝛽
Luego, ∡ 𝐷𝐸𝐶 = 180° − 2𝛼 y ∡ 𝐶𝐹𝐺 = 180° − 𝛽
En ∆𝐷𝐸𝐶, ∡ 𝐸𝐷𝐶 + (180° − 2𝛼) − 𝛼 = 180° ⇒ ∡ 𝐸𝐷𝐶 = 𝛼
∴ ∆ 𝐷𝐸𝐶, es isósceles con 𝐷𝐸 = 𝐸𝐶.

En ∆𝐺𝐹𝐶, ∡ 𝐹𝐺𝐶 + 180° − 𝛼 + 𝛽 − 𝛼 = 180° ⇒ ∡ 𝐹𝐺𝐶 = 𝛼


∴ ∆ 𝐷𝐺𝐶, es isósceles con 𝐷𝐶 = 𝐺𝐶.
𝐸𝐹 𝐸𝐶 𝐸𝐹
∆𝐴𝐵𝐶~∆𝐸𝐹𝐶 ⇒ 𝐴𝐵 = 𝐴𝐶 ⇒ 𝐸𝐶 = 𝐴𝐶 ∙ 𝐴𝐵
𝐷𝐸 𝐷𝐶
∆𝐷𝐵𝐶~∆𝐷𝐶𝐺 ⇒ 𝐷𝐶 = 𝐷𝐵

⇒ (𝐷𝐶)2 = 𝐷𝐸 ∙ 𝐷𝐺
⇒ (𝐷𝐶)2 = 𝐸𝐶 ∙ 𝐷𝐺
𝐸𝐹
⇒ (𝐷𝐶)2 = 𝐴𝐶 ∙ 𝐴𝐵 ∙ 𝐷𝐺
𝐴𝐶 𝐷𝐺
⇒ 𝐷𝐶 = 𝐷𝐶 ∙ 𝐸𝐹 ∙ 𝐴𝐵
5 7
⇒ 𝐷𝐶 = 3 ∙ 𝐸𝐹 ∙ 3
𝐷𝐶 35
⇒ =
𝐸𝐹 9

14. Carlos juega con Luis el juego llamado AMOCLO. En este juego se cuenta con varios puños de
bolitas, el tamaño de los puños se puede modificar siguiendo las reglas:
 Se pueden juntar dos puños de bolitas en uno solo.
 Si un puño tiene un número par de bolitas, se puede dividir en dos puños con el mismo
número de bolitas cada uno.
En determinado momento el juego AMOCLO cuenta con tres puños, uno de 5 bolitas, otro de
49 bolitas y el último de 51 bolitas.
Carlos le pide a Luis que jueguen hasta lograr 105 puños de bolitas y el que lo logre será el
ganador. Sin embargo, Luis le indica que el juego, bajo esas condiciones, no tendría ganador.
Pruebe que la afirmación de Luis es verdadera.
Solución:
En el primer movimiento se tiene que juntar dos de los tres puños de bolitas ya que ninguno de
los tres tiene una cantidad par de bolitas, si se junta los puños de 49 y 51 entonces en el siguiente
paso la cantidad de bolitas será múltiplo de 5.

Ahora bien, los posibles pasos son dividir el puño que tiene cantidad par o bien sumar los puños.
Los pasos siguientes igual se tendrán cantidad de bolitas múltiplo de 5, por lo que es imposible
conseguir puños de una bolita.
En las otras opciones de juntar dos puños se tendría 5 más 49, esto daría un puño de 44 y otro
de 51, ambos son múltiplos de 3, utilizando el mismo análisis se concluye que es imposible
lograr puños de una bolita.

La última posibilidad es juntar 5 y 51, por lo que quedaría puños de 49 y 56, ambos múltiplos
de 7, análogamente se concluye lo mismo. Observe que se tiene un total de 105 bolitas.

Por lo tanto, como lo indica Luis, es imposible obtener un ganador.


15. Considere el número 𝑛 = 7𝑎 931 41𝑏 de ocho dígitos que es divisible por 792. ¿Cuál es el valor
del dígito 𝑎?
Solución:
Como 𝑎 𝑦 𝑏 son dígitos, sus valores están entre 0 y 9.
Se tiene que 7𝑎93141𝑏 es divisible por 792, observe que 792 = 8 ∙ 9 ∙ 11, entonces 7𝑎93141𝑏
es divisible por 8, 9 y 11.
Veamos primero que 7a93141b es divisible por 8, esto nos dice que las últimas tres cifras de
dicho número es divisible por 8. En este caso, se tiene que 41𝑏 es divisible por 8. Observe que
entre 410 y 419 solamente existe un número que es divisible por 8, el número es 416. Por lo tanto
el valor de 𝑏 = 6.
Luego, 7𝑎93141𝑏 es divisible por 9, esto nos dice que la suma de sus cifras es divisible por 9.
Entonces, 7 + 𝑎 + 9 + 3 + 1 + 4 + 1 + 𝑏 = 25 + 𝑎 + 𝑏 es divisible por 9. Observe que 𝑎 + 𝑏
puede ser 2 o 11, pues 0 ≤ 𝑎 + 𝑏 ≤ 18.
Como 𝑏 = 6, cuando 𝑎 + 𝑏 = 2 ⇒ 𝑎 = −4, se descarta, pues a está entre 0 y 9. Cuando
𝑎 + 𝑏 = 11 ⇒ 𝑎 = 5.
Por lo tanto, el valor de 𝑎 es 5.
16. Sea 𝑛 un entero. Si el dígito de las decenas de 𝑛2 es 7, hallar todos los posibles valores del dígito
de las unidades de 𝑛2 .
Solución:
El dígito de las unidades que deja un cuadrado son 0, 1, 4, 5, 6, 9. Luego las posibles dos últimas
cifras son 70, 71, 74, 75, 76, 79. Los restos que deja un cuadrado en la división por 4 son 0 o 1,
los restos que dejan los números que terminan en 70, 71, 74, 75, 76, 79 en la división para 4 son
2, 3, 2, 3, 0, 3, respectivamente.
Luego la única posibilidad es 76. Veamos que 242 = 576 es un ejemplo, y concluimos que el
último dígito es 6.
17. En un pizarrón están escritos los enteros del 1 al 10000, en orden. Se borran los múltiplos de 5 y
después todos los múltiplos de 11. De los números que quedan sin borrar, ¿Cuál queda en la
posición 2020?
Solución 1:
Cada 55 números se borra un total de 15 números: pues son 11 múltiplos de 5 y 5 múltiplos de
11, pero los múltiplos de 55 son múltiplos comunes a 11 y a 5. Entonces de cada 55 números se
quedan 40. Ahora, 2000 = 40 × 50 así que el número 55 × 50 = 2750 está en la posición
2000.
Entre 2750 y 2750 + 20 = 2770 eliminamos los números: 2755, 2760, 2761, 2765, 2770. Con lo
cual el número 2769 ocupa la posición 2020 − 5 = 2015.
Los siguientes cinco números que no son divisibles entre 5 y 11 son: 2771, 2773, 2774, 2776,
2777.
Finalmente eso quiere decir que la posición 2020 la ocupa el número 2777.
Solución 2:
Para saber la cantidad de números que debemos tomar de los 10000 números, de tal manera que
al quitar los múltiplos de 5 y 11 nos queden 2020 números, debemos resolver la ecuación:
𝑛 𝑛 𝑛
𝑛 − [ ] − [ ] + [ ] = 2020
5 11 55
Por otra parte tenemos que:
𝑛≤𝑛<𝑛
𝑛 𝑛 𝑛
≤[ ]< +1
5 5 5
𝑛 𝑛 𝑛
≤[ ]< +1
11 11 11
𝑛 𝑛 𝑛
≤[ ]< +1
55 55 55
Que podemos reescribir de la siguiente forma:
𝑛≤𝑛<𝑛
𝑛 𝑛 𝑛
− − 1 ≤ −[ ] < −
5 5 5
𝑛 𝑛 𝑛
− −1 ≤ −[ ] < −
11 11 11
𝑛 𝑛 𝑛
≤[ ]< +1
55 55 55
Sumando estas desigualdades tenemos que:
𝑛 𝑛 𝑛 𝑛 𝑛 𝑛 𝑛 𝑛 𝑛
𝑛− −1− −1+ ≤ 𝑛−[ ]−[ ]+[ ]<𝑛− − + +1
5 11 55 5 11 55 5 11 55
𝑛 𝑛 𝑛 𝑛 𝑛 𝑛
𝑛− −1− −1+ ≤ 2020 < 𝑛 − − + +1
5 11 55 5 11 55
8𝑛 8𝑛
− 2 ≤ 2020 < +1
11 11
8𝑛 8𝑛
La desigualdad se cumple para 11 = 2020 y para 11 − 1 = 2020
8𝑛 2020×11
Si 11 = 2020 ⇒ 𝑛 = y puesto que 𝑛 es entero positivo tenemos que:
8

2020 × 11
𝑛= = 2 777
8
8𝑛 8𝑛 2021×11
Si 11 − 1 = 2020 ⇒ = 2021 ⇒ 𝑛 = y puesto que 𝑛 es entero positivo tenemos que:
11 8

2021 × 11
𝑛= = 2 778
8
Probando para 𝑛 = 2 778
2778 2778 2778
2 778 − [ ]−[ ]+[ ] = 2778 − 555 − 252 + 50 = 2021 ≠ 2020
5 11 55
Probando para 𝑛 = 2 777
2777 2777 2777
2 777 − [ ]−[ ]+[ ] = 2777 − 555 − 252 + 50 = 2020 = 2020
5 11 55
Finalmente eso quiere decir que la posición 2020 la ocupa el número 2777.
18. Al colocar los dígitos 1, 2, 3 y 4 en cada una de las casillas del cuadrado que se muestra, se
obtienen cuatro números de dos dígitos cuando se leen los números de arriba hacia abajo y de
izquierda a derecha. Por ejemplo, si se colocan los números así

Los números serían 12, 34, 13, 24 y la suma de estos cuatro números es 12+34+13+24 = 83.
Hallar el máximo valor que puede tomar la suma.
Solución:
Colocando letras que representan los números, de la siguiente forma:

Se obtiene la suma:
̅̅̅ + 𝑎𝑐
𝑆 = 𝑎𝑏 ̅̅̅ + 𝑏𝑑
̅̅̅ + 𝑐𝑑 ̅̅̅̅
= 20𝑎 + 11𝑏 + 11𝑐 + 2𝑑
= 2(𝑎 + 𝑏 + 𝑐 + 𝑑) + 18𝑎 + 9𝑏 + 9𝑐
= 2(10) + 18𝑎 + 9𝑏 + 9𝑐
= 20 + 18𝑎 + 9𝑏 + 9𝑐
= 20 + 9(𝑎 + 𝑏 + 𝑐) + 9𝑎
Luego el valor de 𝑑 debe ser lo más pequeño ya que la suma no depende de 𝑑, es decir 𝑑 = 1 ⇒
𝑎 + 𝑏 + 𝑐 = 2 + 3 + 4 = 9, entonces 𝑚𝑎𝑥{𝑆} = 20 + 9(9) + 9𝑎 = 101 + 9𝑎, con lo que
𝑎 = 4 y el máximo 𝑆 es 101 + 9(4) = 137 que puede ser obtenido cuando (𝑏; 𝑐) ∈
{(2; 3), (3; 2)}, entonces una distribución que maximiza la suma es:

19. Daniela escoge un entero positivo 𝑁 < 1000 y descubre que al dividirlo para 7 deja resto 5, al
dividirlo para 5 deja resto 3, al dividirlo para 3 deja resto 2 y al dividirlo para 2 deja resto 1. Halle
todos los posibles valores que puede tomar N.
Solución:
Si un número deja resto 5 en la división para 7, entonces deja resto 5, 12, 19, 26 o 33 en la división
para 35. De los valores anteriores, solo 33 deja resto 3 en la división para 5. Luego N deja resto 33
en la división para 35. Si un número deja resto 2 en la división para 3, entonces deja resto 2 o 5 en
la división para 6. De los valores anteriores, solo 5 deja resto 1 en la división para 2. Luego N deja
resto 5 en la división para 6.

Se tiene que 𝑁 = 35𝑘 + 33. 33 deja resto 3 en la división para 6 y 35 deja resto 5 en la división,
entonces N deja resto 5𝑘 + 3 en la división para 6. Si k deja resto 4 en la división para 6, entonces
N deja resto 5 ∙ 4 + 3 = 23 en la división para 6, es decir, N deja resto 5 en la división para 6.
Finalmente concluimos que 𝑁 = 35(6𝑟 + 4) + 33 = 210𝑟 + 173. Los posibles valores de 𝑁 <
1000 son 173, 383, 593, 803.
20. Sea ABCD un cuadrilátero convexo con AB = AD = 13, BC = CD = 20 y BD = 24. Se sabe que existe una
circunferencia que es tangente interiormente a los 4 lados del cuadrilátero, hallar el radio de dicha
circunferencia.

Solución:
Sea E el punto de intersección de AC y BD. Primero veamos que dado que 𝐴𝐵 = 𝐴𝐷 = 13 y
𝐵𝐶 = 𝐶𝐷 = 20, entonces AC es un eje de simetría del cuadrilátero. De lo anterior se tiene que
AC es perpendicular a BD y 𝐵𝐸 = 𝐸𝐷 = 12.
Usando el teorema de Pitágoras se tiene que:

𝐴𝐸 = √(𝐴𝐵)2 − (𝐵𝐸)2 = √132 − 122 = √169 − 144 = √25 = 5

𝐸𝐶 = √(𝐵𝐶)2 − (𝐵𝐸)2 = √202 − 122 = √400 − 144 = √256 = 16

(𝐵𝐷)(𝐴𝐸) 24 × 5 (𝐵𝐷)(𝐸𝐶) 24 × 16
𝐴∆𝐴𝐵𝐷 = = = 60; 𝐴∆𝐵𝐶𝐷 = = = 192
2 2 2 2
𝐴𝐴𝐵𝐶𝐷 = 60 + 192 = 252
Sea 𝑟 el radio de la circunferencia, entonces el área del cuadrilátero es igual al semi perímetro por el radio,
entonces:

𝐴𝑏 + 𝐵𝐶 + 𝐶𝐷 + 𝐷𝐴 13 + 20 + 20 + 13 66
𝐴𝐴𝐵𝐶𝐷 = 𝑠𝑟 = ×𝑟 = ×𝑟 = 𝑟 = 33𝑟
2 2 2

𝐴𝐴𝐵𝐶𝐷 = 33𝑟
252 = 33𝑟
252 84
𝑟= =
33 11
Nivel VI (Décimo Grado)
1. En la figura se muestra una recta ℒ y dos puntos P y Q a un mismo lado de ella. Las distancias de
P y Q a ℒ son 3 cm y 5 cm, respectivamente. La distancia entre los pies de las proyecciones de P y
Q sobre ℒ es 4 cm.

Hay dos circunferencias tales que cada una pasa por los puntos P y Q, y es tangente a ℒ.
Calcule la suma de los radios de esas dos circunferencias, en cm.

Solución:
Vamos a trazar las dos circunferencias que pasen por los puntos P y Q y que sea tangente
a la recta ℒ

Vamos a trazar el segmento PQ y su prolongación hasta la recta ℒ que corta en el punto B.


Por la propiedad de las circunferencias secantes: “En dos circunferencias secantes el
segmento que une los centros es parte de la mediatriz de la cuerda común a las
circunferencias”.

El segmento 𝑂1 𝑂2 , une los centros de las dos circunferencias, además ̅̅̅̅


𝑃𝑄 ⊥ ̅̅̅̅̅̅̅
𝑂1 𝑂2, por lo
que 𝑃𝐷 = 𝐷𝑄.
Sea el punto A, vamos a trazar el segmento 𝑃𝐸 ∥ ℒ. Sea ∡𝑃𝐵𝐴 = 𝜃, por ángulos
correspondientes ∡𝑄𝑃𝐸 = 𝜃, 𝑃𝐸 = 4 𝑐𝑚.
Sea 𝑥 = 𝑅1 + 𝑅2 , también los puntos F y G. Vamos a trazar el segmento BC, donde
̅̅̅̅̅̅̅̅̅̅
𝐵𝐶 ⊥ ℒ, como el ∆𝐵𝐶𝐷 es rectángulo, entonces ∡𝐵𝐶𝐷 = 𝜃.

Por la propiedad de las circunferencias secantes: “La recta que pasa por los puntos de
intersección de las circunferencias secantes, pasa por el punto medio del segmento cuyos
extremos son los puntos de tangencia”. Es decir, FB = BG, en consecuencia, BC es la
mediana del trapecio rectángulo 𝑂1 𝑂2 𝐺𝐹, por tanto:

𝑅1 + 𝑅2
𝐵𝐶 =
2
𝑥
𝐵𝐶 =
2

Hallando PQ, utilizando el teorema de Pitágoras en el ∆𝑃𝐸𝑄.

(𝑃𝑄)2 = 22 + 42
(𝑃𝑄)2 = 4 + 16
√(𝑃𝑄)2 = √20
𝑃𝑄 = 2√5

El punto D se encuentra en la mitad de la hipotenusa del ∆𝑃𝐸𝑄.


∆𝑃𝐴𝐵~∆𝑃𝐸𝑄 por AA, calculando el valor de BA:

2 4
=
3 𝐵𝐴
2𝐵𝐴 = 12
𝐵𝐴 = 6
Calculando PB, utilizando el teorema de Pitagoras en el ∆𝑃𝐴𝐵.

(𝑃𝐵)2 = 32 + 62
(𝑃𝐵)2 = 9 + 36
√(𝑃𝐵)2 = √45
𝑃𝐵 = 3√5

𝐵𝐷 = 𝑃𝐵 + 𝑃𝐷 = 3√5 + √5 = 4√5. Entonces, los lados del ∆𝐵𝐶𝐷 serán:


𝑠𝑒𝑛 𝜃 en el ∆𝐶𝐵𝐷 𝑠𝑒𝑛 𝜃 en el ∆𝑃𝐵𝐴

4√5 3
𝑠𝑒𝑛 𝜃 = 𝑥 (1) 𝑠𝑒𝑛 𝜃 = 3√5 (2)
2

Igualando las ecuaciones (1) y (2), tenemos que:

4√5 3
𝑥 =
3√5
2
3
4√5 × 3√5 = 𝑥
2
2
12√52 × = 𝑥
3
𝑥 =4×5×2
𝑥 = 40
Por lo tanto la suma de los radios de las dos circunferencias es 40
2. Determine cuántos enteros positivos a cumplen que 𝑎 ≤ 8575 y además: 𝑚𝑐𝑑(𝑎; 8575) =
𝑚𝑐𝑑(𝑎 + 1; 8575) = 1.
Aclaración: mcd(r; s) denota al máximo común divisor de los enteros positivos r y s.
Solución:
Descomponiendo en sus factores primos 8575 = 52 × 73 .
Vamos agrupar números de 35 en 35, porque 35 es múltiplo de 5 y 7.
Probando con los primeros números enteros positivos:
Vamos hallar todos los números que no cumplen con dicha condición:
 Hallando todos los múltiplos de 5 que están comprendidos entre 5 y 8575 (color morado):
Del 1 al 8575 se pueden contar 1715 múltiplos de 5, pero como al anterior número respecto
al múltiplo de 5 se le suma 1, entonces también es múltiplo de 5, por tanto, se podrán contar
2 × 1715 = 3430 números que no cumplen la condición.
 Hallando todos los múltiplos de 7 que están comprendidos entre 7 y 8575 (color verde):

Del 1 al 8575 se pueden contar 1225 múltiplos de 7, pero como al anterior número respecto
al múltiplo de 7 se le suma 1, entonces también es múltiplo de 7, por tanto, se podrán contar
2 × 1225 = 2450 números que no cumplen la condición.
 Hallando todos los múltiplos de 35 que están comprendidos entre 35 y 8575:

Del 1 al 8575 se pueden contar 245 múltiplos de 35, pero cada 35 números se cuentan dos
veces 4 números (color rojo), por tanto, se podrán contar 4 × 245 = 980 números que no
cumplen la condición y se han contado dos veces.
Calculando el total de números que no cumplen la condición:
Total = 3430 + 2450 − 980 = 4900
Calculando el total de números que cumplen la condición:
Total = 8575 − 4900 = 3675
Por lo tanto, 3675 números enteros positivos cumplen con la condición del problema.

3. En la pizarra está escrito un número de cuatro dígitos ̅̅̅̅̅̅̅


𝑎𝑏𝑐𝑑 . Luego, Rafael escribe el número
̅̅̅̅̅ y Enrique escribe el número 𝑐𝑏𝑑
𝑏𝑎𝑐 ̅̅̅̅̅ . ¿Cuál es la cantidad de números de cuatro dígitos, tales
que el número escrito en la pizarra es igual a la suma de los números que escribieron Rafael y
Enrique?
Solución
Sean 𝑎, 𝑏, 𝑐 y 𝑑 dígitos. Denotamos 𝑎𝑏𝑐𝑑 al número cuyos dígitos son 𝑎, 𝑏, 𝑐 y 𝑑,
̅̅̅̅̅̅̅ = 𝑏𝑎𝑐
respectivamente. Nos piden encontrar la condición 𝑎𝑏𝑐𝑑 ̅̅̅̅̅ + 𝑐𝑏𝑑
̅̅̅̅̅?
1000𝑎 + 100𝑏 + 10𝑐 + 𝑑 = 100𝑏 + 10𝑎 + 𝑐 + 100𝑐 + 10𝑏 + 𝑑
990𝑎 = 91𝑐 + 10𝑏
Sin embargo, como 1 ≤ 𝑎 ≤ 9, pues de lo contrario el número no sería de cuatro dígitos,
entonces:
990 ≤ 990𝑎 ≤ 8910
Mientras que:
0 ≤ 91𝑐 + 10𝑏 ≤ 909
Pues el valor mayor se alcanza cuando 𝑐 = 9 y 𝑏 = 9. De modo que es imposible que se cumpla
la igualdad.
4. ¿Cuántos números hay que satisfacen la siguiente conclusión: su cuadrado es más pequeño que
107 ?
Solución
Sea 𝑥 algún número que cumple la condición dada.
Según el problema:
𝑥 2 < 107
𝑥 < ±103 √10
Por lo tanto, el máximo valor de 𝑥 que se puede obtener es 3612.
El más pequeño es −3612.
Por lo tanto, hay 3612 ∙ 2 + 1 = 6325 números que cumplen la condición.
5. Sea 𝑚 ≥ 1 un entero positivo, a y b enteros positivos distintos mayores estrictamente que 𝑚2
y menores estrictamente que 𝑚2 + 𝑚. Hallar todos los enteros d que dividen al producto 𝑎𝑏 y
cumplen que 𝑚2 < 𝑑 < 𝑚2 + 𝑚.
Solución:
Sea d un entero positivo que divide a 𝑎𝑏 y tal que 𝑑 ∈ (𝑚2 , 𝑚2 + 𝑚).Entonces d divide a
(𝑎 − 𝑑)(𝑏 − 𝑑) = 𝑎𝑏 − 𝑑𝑎 − 𝑑𝑏 + 𝑑 2 , como |𝑎 − 𝑑| < 𝑚 𝑦 |𝑏 − 𝑑| < 𝑚, deducimos que
|(𝑎 − 𝑑)(𝑏 − 𝑑)| < 𝑚2 < 𝑑 lo que implica que (𝑎 − 𝑑)(𝑏 − 𝑑) = 0. Así d = a o´ d = b.
6. Los números enteros positivos a, b y c satisfacen las siguientes desigualdades:
𝑎 < 2𝑏, 𝑏 < 2𝑐 𝑦 𝑐 < 18.
Determine el mayor valor posible de 𝑎.
Solución:
Según los datos del problema se tiene: 𝑎 < 2𝑏
También tenemos la expresión: 𝑏 < 2𝑐
A esta última expresión le vamos a multiplicar por 2 a cada miembro de la inecuación:
2 × 𝑏 < 2 × 2𝑐
2𝑏 < 4𝑐
Asimismo, tenemos la expresión: 𝑐 < 18
A esta última expresión le vamos a multiplicar por 4 a cada miembro de la inecuación:
4 × 𝑐 < 4 × 18
4𝑐 < 72
De las tres inecuaciones: 𝑎 < 2𝑏; 2𝑏 < 4𝑐 𝑦 4𝑐 < 72, podemos obtener: 𝑎 < 2𝑏 < 4𝑐 < 72
4𝑐, representa el máximo múltiplo de 4 que sea menor que 72 y es el número 68.
2𝑏, representa el máximo múltiplo de 2 que sea menor que 68 y es el número 66.
𝑎 es menor que 66. Por tanto, para que a sea el mayor posible, 𝑎 = 65.
Por lo tanto, el mayor valor posible de a es 65.
7. Sean 𝑥 e 𝑦 números reales positivos tales que 𝑥 ≠ 𝑦 y además:

1 1 2
+ =
1 + 𝑥 2 1 + 𝑦 2 1 + 𝑥𝑦
Determine el menor valor posible de (1 + 2𝑥 2 )(1 + 18𝑦 2 )
Solución:
Simplificando la ecuación planteada:
1 1 2
2
+ 2
=
1+𝑥 1+𝑦 1 + 𝑥𝑦
1 + 𝑦2 + 1 + 𝑥2 2
2 2
=
(1 + 𝑥 )(1 + 𝑦 ) 1 + 𝑥𝑦
(2 + 𝑥 2 + 𝑦 2 )(1 + 𝑥𝑦) = 2(1 + 𝑥 2 )(1 + 𝑦 2 )
2 + 2𝑥𝑦 + 𝑥 2 + 𝑥 3 𝑦 + 𝑦 2 + 𝑥𝑦 3 = 2(1 + 𝑦 2 + 𝑥 2 + 𝑥 2 𝑦 2 )
2 + 2𝑥𝑦 + 𝑥 2 + 𝑥 3 𝑦 + 𝑦 2 + 𝑥𝑦 3 = 2 + 2𝑦 2 + 2𝑥 2 + 2𝑥 2 𝑦 2
𝑥 3 𝑦 + 𝑥𝑦 3 − 2𝑥 2 𝑦 2 = 𝑥 2 + 𝑦 2 − 2𝑥𝑦
𝑥𝑦(𝑥 2 + 𝑦 2 − 2𝑥𝑦) = (𝑥 2 + 𝑦 2 − 2𝑥𝑦)
𝑥𝑦 = 1
De la expresión anterior, se puede afirmar que el producto de dos números reales positivos es
igual a uno, para ello vamos a probar con los primeros números enteros positivos reemplazando
en: (1 + 2𝑥 2 )(1 + 18𝑦 2 )

 Si: 𝑥 = 2 → 𝑦 = 1⁄2, reemplazando se tiene que:


2
[1 + 2(2)2 ] [1 + 18(1⁄2) ]
(1 + 8)(1 + 9⁄2)
99
(9)(11⁄2) = = 49,5
2
 Si: 𝑥 = 3 → 𝑦 = 1⁄3, reemplazando se tiene que:
2
[1 + 2(3)2 ] [1 + 18(1⁄3) ]
(1 + 18)(1 + 2)
(19)(3) = 57
 Si: 𝑥 = 4 → 𝑦 = 1⁄4, reemplazando se tiene que:
2
[1 + 2(4)2 ] [1 + 18(1⁄4) ]
(1 + 32)(1 + 9⁄8)
561
(33)(17⁄8) = = 70,125
8
De los ejemplos presentados se puede concluir que a medida que los primeros factores crecen
(9; 19 y 33) los segundos factores se hacen más pequeños (11/2; 3 y 17/8); pero los productos
finales también crecen (49,5; 27 y 70,125), esto sucede porque los factores son diferentes,
entonces para que el producto sea el menor posible los factores tienen que ser iguales, es decir:
1 + 2𝑥 2 = 1 + 18𝑦 2
2𝑥 2 = 18𝑦 2
𝑥 2 = 9𝑦 2
𝑥 = 3𝑦
1
𝑥 = 3×
𝑥
𝑥2 = 3
𝑥 = √3
1
Por lo tanto, tenemos que: 𝑦 =
√3

Reemplazando se tiene que:


(1 + 2𝑥 2 )(1 + 18𝑦 2 )
2 2
[1 + 2(√3) ] [1 + 18 (1⁄ ) ]
√3
(1 + 6)(1 + 6)
(7)(7) = 49
El menor valor posible de (1 + 2𝑥 2 )(1 + 18𝑦 2 ) es 49
8. Una progresión aritmética consta de k enteros positivos. Si se cumple que cada término a partir
del segundo es al menos 22% más que el término anterior, determine el mayor valor posible de
k.
Solución:
Enumerando los términos de la progresión aritmética:
𝑎1 ; 𝑎2 ; 𝑎3 ; 𝑎4 ; 𝑎5 ; 𝑎6 ; 𝑎7 ; ⋯
𝑎1 ; 𝑎1 + 𝑟; 𝑎1 + 2𝑟; 𝑎1 + 3𝑟; 𝑎1 + 4𝑟; 𝑎𝑎 + 5𝑟; 𝑎1 + 6𝑟; ⋯
Cada término a partir del segundo es al menos 22% más que el término anterior:
𝑎𝑛 ≥ 122%𝑎(𝑛−1) , 𝑛 ≥ 2
Analizando el segundo término:
𝑎2 ≥ 𝑎1
𝑎1 + 𝑟 ≥ 122% 𝑎1
𝑟 ≥ 122% 𝑎1 − 𝑎1
𝑟 ≥ 122% 𝑎1 − 100%𝑎1
𝑟 ≥ 22% 𝑎1
Analizando el tercer término:
𝑎3 ≥ 𝑎2
𝑎1 + 2𝑟 ≥ 122% (𝑎1 + 𝑟)
𝑎1 + 2𝑟 ≥ 122% 𝑎1 + 122%𝑟
2𝑟 − 122%𝑟 ≥ 122% 𝑎1 − 𝑎1
200%𝑟 − 122%𝑟 ≥ 122% 𝑎1 − 100%𝑎1
78%𝑟 ≥ 22% 𝑎1
Analizando el cuarto término:
𝑎4 ≥ 𝑎3
𝑎1 + 3𝑟 ≥ 122% (𝑎1 + 2𝑟)
𝑎1 + 3𝑟 ≥ 122% 𝑎1 + 244%𝑟
3𝑟 − 244%𝑟 ≥ 122% 𝑎1 − 𝑎1
300%𝑟 − 122%𝑟 ≥ 122% 𝑎1 − 100%𝑎1
56%𝑟 ≥ 22% 𝑎1
Analizando el quinto término:
𝑎5 ≥ 𝑎4
𝑎1 + 4𝑟 ≥ 122% (𝑎1 + 3𝑟)
𝑎1 + 4𝑟 ≥ 122% 𝑎1 + 366%𝑟
4𝑟 − 366%𝑟 ≥ 122% 𝑎1 − 𝑎1
400%𝑟 − 366%𝑟 ≥ 122% 𝑎1 − 100%𝑎1
34%𝑟 ≥ 22% 𝑎1
Analizando el sexto término:
𝑎6 ≥ 𝑎5
𝑎1 + 5𝑟 ≥ 122% (𝑎1 + 4𝑟)
𝑎1 + 5𝑟 ≥ 122% 𝑎1 + 488%𝑟
5𝑟 − 488%𝑟 ≥ 122% 𝑎1 − 𝑎1
500%𝑟 − 488%𝑟 ≥ 122% 𝑎1 − 100%𝑎1
12%𝑟 ≥ 22% 𝑎1
Analizando el séptimo término:
𝑎7 ≥ 𝑎6
𝑎1 + 6𝑟 ≥ 122% (𝑎1 + 5𝑟)
𝑎1 + 6𝑟 ≥ 122% 𝑎1 + 610%𝑟
6𝑟 − 610%𝑟 ≥ 122% 𝑎1 − 𝑎1
600%𝑟 − 610%𝑟 ≥ 122% 𝑎1 − 100%𝑎1
−10%𝑟 ≥ 22% 𝑎1
El séptimo término nos sale negativo y esto contradice con el enunciado del problema, porque
los términos de la progresión aritmética son enteros positivos. Por tanto, la progresión
aritmética sólo tiene seis términos, es decir, 𝑘 = 6
Por lo tanto el mayor valor posible de 𝑘 es 6.
9. Las circunferencias mostradas son tangentes y sus radios son 4 y 2. Si AB = BC = CD, calcule
la longitud de AD.

Solución:
Sea “2𝑥” en: 𝐴𝐵 = 𝐵𝐶 = 𝐶𝐷 = 2𝑥, entonces: 𝐴𝐷 = 6𝑥. Vamos a ubicar los centros de las
circunferencias (G y E), luego trazamos los radios de las circunferencias a los puntos B, C y
F.

Vamos a trazar las mediatrices GH y EI a los segmentos AB y CD respectivamente, de manera


que 𝐴𝐻 = 𝐻𝐵 = 𝑥, asimismo 𝐶𝐼 = 𝐼𝐷 = 𝑥.
Sea la longitud de la mediatriz 𝐸𝐼 = 𝑦.
Sea la longitud de la mediatriz 𝐺𝐻 = 𝑧.
La mediatriz se define como, recta perpendicular a un segmento que se traza a su punto medio.
La medida del segmento 𝐻𝐼 = 𝑥 + 2𝑥 + 𝑥 = 4𝑥
Vamos a trazar un segmento paralelo al segmento HI que pase por el punto E y que corta en J
al segmento GH. Como HJEI es un rectángulo, entonces 𝐻𝐽 = 𝑦, por diferencia de segmentos
𝐽𝐺 = 𝑧 – 𝑦, además 𝐽𝐸 = 4𝑥.

Utilizando el teorema de Pitágoras en ∆𝐶𝐼𝐸 y ∆𝐻𝐵𝐺.

𝑥 2 + 𝑦 2 = 22 𝑥 2 + 𝑧 2 = 42
𝑥2 + 𝑦2 = 4 𝑥 2 + 𝑧 2 = 16
𝑦 2 = 4 − 𝑥 2 (1) 𝑧 2 = 16 − 𝑥 2 (2)

Utilizando el teorema de Pitágoras en el ∆𝐽𝐸𝐺.

(𝑧 − 𝑦)2 + (4𝑥)2 = 62
𝑧 − 2𝑧𝑦 + 𝑦 2 + 16𝑥 2 = 36
2

Reemplazando (1) y (2) en la ecuación:


16 − 𝑥 2 − 2𝑧𝑦 + 4 − 𝑥 2 + 16𝑥 2 = 36
20 − 2𝑧𝑦 + 14𝑥 2 = 36
14𝑥 2 − 2𝑧𝑦 = 36 − 20
14𝑥 2 − 2𝑧𝑦 = 16
14𝑥 2 − 16 = 2𝑧𝑦
7𝑥 2 − 8 = 𝑧𝑦
Elevando al cuadrado la expresión anterior:
(7𝑥2 − 8)2 = (𝑦𝑧)2
49𝑥 4 − 112𝑥 2 + 64 = 𝑦 2 𝑧 2
Reemplazando (1) y (2) en la ecuación:
49𝑥 4 − 112𝑥 2 + 64 = (4 − 𝑥 2 )(16 − 𝑥 2 )
49𝑥 4 − 112𝑥 2 + 64 = 64 − 20𝑥 2 + 𝑥 4
49𝑥 4 − 112𝑥 2 + 64 − 𝑥 4 + 20𝑥 2 − 64 = 0
48𝑥 4 − 92𝑥 2 = 0
12𝑥 4 − 23𝑥 2 = 0
𝑥 2 (12𝑥 2 − 23) = 0
𝑥 2 = 0 ∨ 12𝑥 2 − 23 = 0

√𝑥 2 = 0 ∨ 12𝑥 2 = 23
23
𝑥 = 0 ∨ 𝑥2 =
12

23
𝑥 = 0 ∨ √𝑥 2 = √
12

23
𝑥=0 ∨ 𝑥=√
12
Reemplazando 𝑥 en 𝐴𝐷 = 6𝑥, tenemos que:

𝐴𝐷 = 6𝑥
23
𝐴𝐷 = 6√
12

23 √12
𝐴𝐷 = 6√ ×
12 √12

√23 × 2√3 12√69


𝐴𝐷 = 6 2 = = √69
(√12) 12

Por lo tanto la longitud de AD es √69


̅̅̅̅
𝑎𝑏
10. Supongamos que el mínimo valor posible de la expresión |𝑏𝑎 ̅̅̅̅
− 3| donde a y b son dígitos no
nulos, es q. Si q se puede expresar como 𝑚𝑛 donde m y n son enteros positivos coprimos, calcule
el valor de m + n.
Solución:
De acuerdo al problema se tiene:
̅̅̅
𝑎𝑏 𝑚
| − 3| = 𝑞 =
̅̅̅
𝑏𝑎 𝑛
𝑚 𝑦 𝑛 son números primos entre sí, es decir, que ambos tienen divisor común a la unidad.
̅̅̅̅
𝑎𝑏
La división de ̅̅̅̅ debe aproximarse lo más cerca posible al valor de tres, para que el resultado
𝑏𝑎
sea el mínimo posible. Entonces debe ocurrir:
̅̅̅
𝑎𝑏
≈3
̅̅̅
𝑏𝑎
̅̅̅
𝑎𝑏 ≈ 3 × ̅̅̅
𝑏𝑎
Probando con algunos números:
31 ≈ 3 × 13 41 ≈ 3 × 14 62 ≈ 3 × 26
31 ≈ 39 41 ≈ 42 62 ≈ 78
Su diferencia es: 39 − 31 = 8 Su diferencia es: 42 − 41 = 1 Su diferencia es: 78 − 62 = 16

La menor diferencia es 1, por lo tanto, ̅̅̅


𝑎𝑏 = 41; 𝑎 = 4 𝑦 𝑏 = 1
Reemplazando se tiene que:
𝑚 ̅̅̅
𝑎𝑏
= | − 3|
𝑛 ̅̅̅
𝑏𝑎
𝑚 41 41 − 42 1
= | − 3| = | | = |− |
𝑛 14 14 14
𝑚 1
=
𝑛 14
Por tanto, 𝑚 = 1, 𝑏 = 14. Sumando: 𝑚 + 𝑛 = 1 + 14 = 15.
11. Una partícula se mueve a través del primer cuadrante siguiendo el patrón que se indica en la
figura. Durante el primer minuto se mueve desde el origen hasta (1; 0), moviéndose una unidad
de distancia paralela a un eje cada minuto. Determine en qué punto se encontrará la partícula
después de exactamente 2 horas.
Solución:
Poniendo un poco de atención nos daremos cuenta de que, si n es impar, la partícula necesita 𝑛2
minutos para llegar al punto (𝑛; 0). En efecto, para llegar de (𝑛; 0) a (𝑛 + 2; 0) debemos
movernos 𝑛 pasos hacia arriba, n hacia la izquierda, 1 hacia arriba, 𝑛 + 1 a la derecha, 𝑛 + 1
hacia abajo y 1 hacia la derecha. Luego para llegar del origen a (𝑛 + 2; 0) se requieren
𝑛2 + 2𝑛 + 1 + 2(𝑛 + 1) + 1 = (𝑛 + 2)2 pasos, y el resultado se sigue por inducción. Por
último, ya que 120 = 112 − 1, la posición de la partícula luego de 120 minutos es (10; 0).
12. Sean 𝑎; 𝑏; 𝑐 enteros distintos de cero, con 𝑎 ≠ 𝑐, tales que:
𝑎 𝑎2 + 𝑏 2
=
𝑐 𝑐 2 + 𝑏2
Demuestre que 𝑎2 + 𝑏 2 + 𝑐 2 no puede ser un número primo
Solución:
Quitando denominadores tenemos que 𝑎𝑐 2 + 𝑎𝑏 2 = 𝑐𝑎2 + 𝑐𝑏 2 , o bien
2 2 2 2 2
𝑎𝑐 − 𝑐𝑎 + 𝑎𝑏 − 𝑐𝑏 = 0. El miembro izquierdo factoriza como (𝑎 − 𝑐)(𝑏 − 𝑎𝑐), por lo
que la condición 𝑎 ≠ 𝑐 implica que 𝑏 2 − 𝑎𝑐 = 0.
En consecuencia:
𝑎2 + 𝑏 2 + 𝑐 2 = 𝑎2 + 𝑎𝑐 + 𝑐 2 = (𝑎 + 𝑐)2 − 𝑎𝑐 = (𝑎 + 𝑐)2 − 𝑏 2
= (𝑎 + 𝑐 + 𝑏)(𝑎 + 𝑐 − 𝑏)
𝑎2 +𝑏 2 𝑎
Ahora bien, la fracción 𝑐 2 +𝑏2 es siempre positiva, por lo que 𝑐 también lo es y así 𝑎 𝑦 𝑐 tienen
el mismo signo. Ya que la igualdad de estas fracciones se sigue cumpliendo al cambiar el signo
de 𝑎 𝑦 𝑐, podemos asumir que ambos son positivos. Si 𝑏 > 0 entonces 𝑎 + 𝑐 + 𝑏 ≥ 3, ya que
𝑎; 𝑏; 𝑐 > 0, y además tenemos que 𝑎 + 𝑐 + 𝑏 < 𝑎2 + 𝑏 2 + 𝑐 2 puesto que 𝑥 2 ≥ 𝑥 para
cualquier entero 𝑥 ≥ 1. Por otro lado, si 𝑏 < 0 entonces 3 < 𝑎 + 𝑐 − 𝑏 < 𝑎2 + 𝑏 2 + 𝑐 2 .
En cualquier caso 𝑎2 + 𝑏 2 + 𝑐 2 tiene un divisor mayor que 1 y menor que si mismo, y por tanto
no puede ser primo.
13. Sean el hexágono regular ABCDEF inscrito en una circunferencia de centro O, N un punto tal
que 𝐸 − 𝑁 − 𝐶, M un punto tal que 𝐴 − 𝑀 − 𝐶 y R un punto en la circunferencia, tal que
̂ = 90°, 𝐴𝑀 = 𝐶𝑁 y 𝐴𝐶 = √3. Determine la longitud de 𝐴𝑀
𝐷 − 𝑁 − 𝑅. Si 𝑚 𝐸𝐹𝑅 ̅̅̅̅̅ .
𝐴𝐶 𝐶𝐸

Solución:
Considere la figura:

Así tenemos que:


𝐴𝑀 𝐶𝑁
 = 𝑦 𝐴𝐶 = 𝐶𝐸 ⇒ 𝐴𝑀 = 𝐶𝑁 ⇒ 𝐸𝑁 = 𝐶𝑀
𝐴𝐶 𝐶𝐸

 𝐶𝐵 = 𝐸𝐷

 ∡ 𝐴𝐶𝐵 = ∡𝐶𝐸𝐷 = 30°

 ∆ 𝐵𝑀𝐶 ≅ ∆𝐷𝑁𝐸 (𝐿𝐴𝐿)

 ∠ 𝑀𝐵𝐶 ≅ ∠𝐶𝐸𝐷
1
̂ = 45° = ∡ 𝑀𝐵𝐶
 ∡ 𝑁𝐷𝐸 = ∡𝑅𝐷𝐸 = 2 𝑚𝐸𝐹𝑅

 ∡ 𝐵𝐶𝐸 = 90°

 ∡ 𝑩𝑁𝑪 = 45°

 𝐵𝐶 = 𝑁𝐶

 ∆ 𝐵𝐶𝐸 es recto en C, ∡ 𝐵𝐸𝐶 = 30°


1 𝐵𝐶
 𝐸𝐶 = √3𝐵𝐶 ⇒ = 𝐸𝐶
√3

𝐴𝑀 𝐶𝑁 𝐵𝐶 1
 = = 𝐶𝐸 =
𝐴𝐶 𝐶𝐸 √3

1 1
 𝐴𝑀 = 𝐴𝐶 = ∙ √3 = 1
√3 √3

14. Un número entero positivo es nefelibata si al tomar el último dígito de izquierda a derecha y
colocarlo de primero, manteniendo los demás en el mismo orden, el número resultante es
exactamente el doble del número original. ¿Cuál es el menor número nefelibata?
Solución:
Sea 𝑥 el número en cuestión. Expresemos 𝑥 = 10𝑏 + 𝑎, donde a es el último dígito del número,
y por lo tanto, es un entero no negativo menor que 10. Si 𝑏 tiene 𝑘 dígitos, entonces, sabemos
que 2𝑥 = 10𝑘 𝑎 + 𝑏 = 2(10𝑏 + 𝑎) = 20𝑏 + 2𝑎.
De aquí obtenemos que 19𝑏 = (10𝑘 − 2)𝑎. Como 19 es primo y 0 < 𝑎 < 10, necesariamente
se cumple que 19 no divide a 𝑎, y por lo tanto, divide a 10𝑘 𝑘 − 2.
Por lo tanto, estamos buscando la combinación, si esta existe, del menor posible valor de 𝑘 y el
menor valor de a. Además, se tiene que 10𝑘 ≅ 2 𝑚𝑜𝑑 19.
Nótese que:
101 ≅ 10 𝑚𝑜𝑑 19; 102 ≅ 5 𝑚𝑜𝑑 19; 103 ≅ 12 𝑚𝑜𝑑 19; 104 ≅ 6 𝑚𝑜𝑑 19;
105 ≅ 3 𝑚𝑜𝑑 19; 106 ≅ 11 𝑚𝑜𝑑 19; 107 ≅ 15 𝑚𝑜𝑑 19; 108 ≅ 17 𝑚𝑜𝑑 19;
109 ≅ 18 𝑚𝑜𝑑 19; 1010 ≅ 9 𝑚𝑜𝑑 19; 1011 ≅ 14 𝑚𝑜𝑑 19; 1012 ≅ 7 𝑚𝑜𝑑 19;
1013 ≅ 17 𝑚𝑜𝑑 19; 1014 ≅ 16 𝑚𝑜𝑑 19; 1015 ≅ 8 𝑚𝑜𝑑 19; 1016 ≅ 4 𝑚𝑜𝑑 19;
1017 ≅ 2 𝑚𝑜𝑑 19
De aquí se concluye que 𝑏 tiene al menos 17 dígitos. Más aún, se tiene que:
𝑎(10𝑘 − 2)
𝑏=
19
Si se intenta 𝑘 = 17 𝑦 𝑎 = 1, se obtiene 𝑏 = 5 263 157 894 736 842, que tiene únicamente 16
dígitos, y por lo tanto, no es una solución. Si se intenta 𝑎 = 2, se obtiene
𝑏 = 10 526 315 789 473 684, y por lo tanto, 𝑥 = 105 263 157 894 736 842 y nótese que
105 263 157 894 736 842 × 2 = 210 526 315 789 473 684
Que es nuestra solución.
15. Un juego consiste de una cuadrícula de 4 _ 4 y fichas de dos colores (Amarillas y Blancas). Un
jugador escoje un tipo de ficha y se la da al segundo jugador quien la coloca donde quiera, luego
el segundo jugador escoje un tipo de ficha y se la da al primero quien la coloca donde quiera,
continúan de este modo y gana el que logre formar una línea con tres fichas del mismo color
(horizontal, vertical o diagonal y sin importar si es la ficha con la que inició o no).
Antes de iniciar la partida ya se encuentran colocadas dos fichas amarillas y dos blancas tal
como muestra la figura siguiente
María y Carolina juegan una partida. Si María inicia (escogiendo la ficha y dándosela a Carolina para
que esta la coloque) indique si existe una estrategia ganadora para alguna de las dos jugadoras y, en caso
de existir, describa la estrategia.
Solución:
Llamemos a las fichas amarillas A y a las blancas B. Veamos primero que una posición
perdedora es aquella en la que, sin importar la ficha que le toque escoger al jugador, al dársela
al otro ganará.
Vemos también que en la primera jugada María debe escoger A, pues si escoge B Carolina
ganará. Por su parte Carolina debe colocar esta ficha A en una esquina, pues de lo contrario
quedará en una posición perdedora. Igualmente, en la segunda jugada Carolina debe escoger A
y María debe colocarla en la otra esquina. Entonces, luego de las dos primeras jugadas se tendrá
el siguiente acomodo.

Veamos ahora que María siempre puede ganar si escoge una A para darle a Carolina.
Si Carolina la coloca en una casilla central, debe escoger una B para darle a María y esta ganará
con solo colocarla en la otra casilla central y escoger una A para darle a Carolina, pues sin
importar donde la coloque quedará en una posición perdedora.

Si Carolina la coloca en una casilla de los lados tiene dos posibilidades, pues las posiciones 1
son simétricas entre sí, al igual que las posiciones 2:

I Caso:
Si la coloca en una posición 1, debe escoger una B y María la colocará tapando la fila de fichas
A. Ahora María gana escogiendo una A, pues sin importar donde la coloque Carolina quedará
en una posición perdedora.
II Caso:
Si la coloca en una posición 2, debe escoger una B. María la colocará tapando la fila de fichas
A (como indica la figura) y escoge una A para darle a Carolina. Observe que la única posibilidad
de Carolina para colocar esta A es la casilla marcada con *, pues en todas las demás llevaría a
una posición perdedora. Además, para no perder inmediatamente, debe escoger una B para darle
a María.

Ahora María gana colocando esta B en la casilla indicada y escogiendo una A para que Carolina
coloque, pues al igual que el I Caso, sin importar donde la coloque Carolina quedará en una
posición perdedora.

16. De un libro de 1000 páginas se han arrancado una cantidad consecutiva de hojas. Se sabe que la
suma de los números de las páginas arrancadas es 2020. Determine la numeración de las páginas
arrancadas.

Solución:
Como son consecutivas las hojas faltantes, llamamos 𝑎; 𝑎 + 1; 𝑎 + 2; ⋯ ; 𝑎 + (𝑛 − 1) la
numeración de las páginas faltantes. Como cada hoja contiene dos páginas, 𝑛 debe ser par.
Sumando las numeraciones se obtiene que:
𝑎 + 𝑎 + 1 + 𝑎 + 2 + ⋯ + 𝑎 + (𝑛 − 1) = 𝑛𝑎 + (1 + 2 + ⋯ + (𝑛 − 1)) = 2020
Entonces, usando que la descomposición en factores primos de 2020 = 22 ∙ 5 ∙ 101.
(𝑛 − 1)𝑛
𝑛𝑎 + = 2020 ⇒ 2𝑛𝑎 + (𝑛 − 1)𝑛 = 2 ∙ 22 ∙ 5 ∙ 101 = 23 ∙ 5 ∙ 101
2
De esta manera,
𝑛[2𝑎 + (𝑛 − 1)] = 23 ∙ 5 ∙ 101
Como 𝑛 es par, 2𝑎 + (𝑛 − 1) es impar. y los divisores impares de 23 ∙ 5 ∙ 101, son 5; 101 y
505, entonces tenemos que 𝑛 = 23 = 8 y 2𝑎 + (𝑛 − 1) = 5; 101 𝑦 505, tomando en cuenta
cada caso, tenemos que:
 Si 𝑛 = 8 y 2𝑎 + (𝑛 − 1) = 5:
2𝑎 + (8 − 1) = 5
2𝑎 + 7 = 5
2𝑎 = −2
𝑎 = −1
Lo que es imposible, por lo tanto, para este valor no es posible tener el número de la primera
página.
 Si 𝑛 = 8 y 2𝑎 + (𝑛 − 1) = 101:
2𝑎 + (8 − 1) = 101
2𝑎 + 7 = 101
2𝑎 = 94
𝑎 = 47
Por lo tanto, la numeración de las paginas seria: 47; 48; 49; 50; 51; 52; 53 que para comprobar
si es la solución correcta se suman los números y se obtiene como resultado: 350 ≠ 2020,
estos no cumplen la condición dada por lo tanto no es la numeración de las paginas arrancadas.
 Si 𝑛 = 8 y 2𝑎 + (𝑛 − 1) = 505:
2𝑎 + (8 − 1) = 505
2𝑎 + 7 = 505
2𝑎 = 498
𝑎 = 249
Por lo tanto, la numeración de las paginas seria: 249; 250; 251; 252; 253; 254; 255 que para
comprobar si es la solución correcta se suman los números y se obtiene como resultado:
2020 = 2020, estos cumplen la condición dada por lo tanto es la numeración de las paginas
arrancadas.
17. En un rombo ABCD se traza una circunferencia con centro en el punto medio del lado AB y
con diámetro AB, que corta al lado BC en el punto K. Similarmente se traza una circunferencia
con centro en el punto medio del lado AD y de diámetro AD que corta al lado CD en el punto
L. Suponga que ∡𝐴𝐾𝐿 = ∡𝐴𝐵𝐶. Determine los ángulos del rombo. Recordar que un rombo es
un cuadrilátero cuyos cuatro lados son iguales.
Solución:
Representando gráficamente la situación brindada en el problema, tenemos que:
Los triángulos 𝐴𝐿𝐷 y 𝐴𝐾𝐵 son rectángulos porque uno
de sus lados es diámetro de la circunferencia.
Además, por propiedad de los ángulos en un rombo
tenemos que:
∡𝐴𝐷𝐿 = ∡𝐴𝐵𝐾 = 𝛼 y ∡𝐷𝐶𝐵 = ∡𝐵𝐴𝐷 = 𝛽.
En el triangulo 𝐴𝐿𝐷, tenemos que: ∡𝐴𝐷𝐿 = 𝛼 y
∡𝐷𝐴𝐿 = 90° − 𝛼.
En el triángulo 𝐴𝐾𝐵, tenemos que: ∡𝐴𝐵𝐾 = 𝛼 y ∡𝐵𝐴𝐾 = 90° − 𝛼, y como 𝐴𝐷 = 𝐴𝐵 por ser
lados del rombo, entonces ∆ 𝐴𝐿𝐷 ≅ ∆ 𝐴𝐾𝐵, por lo tanto, 𝐴𝐿 = 𝐴𝐾.
De la información brindada en el problema tenemos que:
∡𝐴𝐾𝐿 = ∡𝐴𝐵𝐶, además 𝐴𝐿 = 𝐴𝐾, entonces ∆ 𝐿𝐴𝐾 es
isósceles, por lo que ∡𝐴𝐾𝐿 = ∡𝐴𝐿𝐾 = 𝛼 y por suma de
ángulos internos en un triángulo:
∡𝐴𝐾𝐿 + ∡𝐴𝐿𝐾 + ∡𝐿𝐴𝐾 = 180°
𝛼 + 𝛼 + ∡𝐿𝐴𝐾 = 180°
∡𝐿𝐴𝐾 = 180° − 2𝛼
Por la suma de los ángulos internos de un rombo, tenemos que:
2𝛼 + 2𝛽 = 360°
2𝛼 + 2(∡𝐷𝐴𝐿 + ∡𝐿𝐴𝐾 + ∡𝐾𝐴𝐵) = 360°
2𝛼 + 2(90° − 𝛼 + 180° − 2𝛼 + 90° − 𝛼) = 360°
2𝛼 + 2(360° − 4𝛼) = 360°
2𝛼 + 720 − 8𝛼 = 360°
−6𝛼 = −360°
𝛼 = 60°
Sustituyendo el valor encontrado para 𝛼, tenemos que:
2𝛼 + 2𝛽 = 360°
2(60°) + 2𝛽 = 360°
2𝛽 = 360° − 120°
240°
𝛽= = 120°
2
18. Aburrido de esperar su avión para viajar a la Olimpiada Internacional de Matemáticas, Daniel
empezó a escribir potencias de 2 en una lista en su cuaderno de la siguiente manera:
Empezando con el número 1, Daniel escribe la siguiente potencia de 2 al final de su lista e
invierte el orden de los números de la lista.
Denominemos como “movimiento” a dicha modificación de la lista, incluyendo el primer paso.
La lista en cada uno de los 4 primeros “movimientos” se ve de la siguiente forma:
1 → 2, 1 → 4, 1, 2 → 8, 2, 1, 4
Daniel se plantea realizar operaciones hasta que llegue su avión, pero le preocupa que la lista
crezca demasiado. Luego de 2020 movimientos, ¿cuál es la suma de los primeros 1010 números?
Solución:
Sigamos con la lista por 6 pasos:
𝟏
𝟐, 𝟏
𝟒, 𝟏, 𝟐
𝟖, 𝟐, 𝟏, 𝟒
𝟏𝟔, 𝟒, 𝟏, 𝟐, 𝟖
𝟑𝟐, 𝟖, 𝟐, 𝟏, 𝟒, 𝟏𝟔
Notemos que luego de un número par de pasos, la primera mitad de la lista consiste de las
potencias de dos con exponente impar. Consideremos la suma S de los números de la lista luego
de los 2020 movimientos.
𝑺 = 22019 + 22017 + ⋯ + 2 + 1 + ⋯ + 22016 + 22018
Es conocido que la suma de todas las potencias de dos desde 1 hasta 2𝑘 es 2𝑘+1 − 1. Se sigue
que:
𝑆 = 22020 − 1
Por otro lado, la observación clave es que la suma de los primeros 1010 números es el doble de
la suma K de los últimos 1010, pero de forma invertida:

22019 + 22017 + ⋯ + 2 = 2(22018 + 22016 + ⋯ + 1) = 2𝑘


Por lo tanto,
22020 − 1 = 𝑆 = 2𝑘 + 𝑘 = 3𝑘
De donde:
22020 − 1
𝑘=
3
La suma de los 1010 números es:
22020 − 1
2𝑘 = 2 ( )
3
19. Halle cuántos números de dos dígitos son divisibles para la suma de sus dígitos.
Solución:
Supongamos que 𝑛 = 10𝑎 + 𝑏 cumple, luego:
10𝑎 + 𝑏 = 𝑘(𝑎 + 𝑏) ⇒ (10 − 𝑘)𝑎 = (𝑘 − 1)𝑏
Reemplazamos 𝑟 = 𝑘 − 1 entonces (9 − 𝑟)𝑎 = 𝑟𝑏. Sabemos que 𝑎 > 0. Si 𝑏 = 0, entonces
9 − 𝑟 = 0 y 𝑎 puede tomar cualquier valor. Luego hay 9 casos.
Ahora consideremos 𝑏 > 0. 𝑎, 𝑏 son dígitos no nulos, se tiene que 1 ≤ 𝑟 ≤ 8. También
1 ≤ 9 − 𝑟 ≤ 8 y 9 − 𝑟 + 𝑟 = 9, luego por simetría basta considerar los casos 1 ≤ 𝑟 ≤ 4 y el
total será el doble de los casos considerados
𝑟 = 1, entonces 8𝑎 = 𝑏. Luego 8 divide a b y hay un caso.
𝑟 = 2, entonces 7𝑎 = 2𝑏. Luego 7 divide a b y hay un caso.
𝑟 = 3, entonces 2𝑎 = 𝑏. Luego b es par y hay 4 casos.
𝑟 = 4, entonces 5𝑎 = 4𝑏. Luego 5 divide a b y hay un caso.
Concluimos que hay en total 9 + 2(3 + 4) = 23 números que cumplen con la propiedad.
20. Cada letra es equivalente a un dígito. A cada letra distinta se le asigna un número distinto y la
palabra no puede empezar con cero (0). Encuentra la equivalencia de la palabra AGUA en
números.

Solución:
Un problema equivalente es:

Con lo cual podemos escribir las siguientes ecuaciones:


3𝐸 = 𝐴 𝑜 3𝐸 = 𝐴 + 10𝑙1 𝑐𝑜𝑛 𝑙1 = 0; 1; 23𝐴 + 𝑙1 = 𝑈 𝑜 3𝐴 + 𝑙1 = 𝑈 + 10𝑙2 𝑐𝑜𝑛 𝑙2
= 0; 1; 23𝑅 + 𝑙2 = 𝐺 𝑜 3𝑅 + 𝑙2 = 𝐺 + 10𝑙3 𝑐𝑜𝑛 𝑙3 = 0; 1; 23𝑇 + 𝑙3 = 𝐴
3𝐴 + 𝑙1 = 𝑈 𝑜 3𝐴 + 𝑙1 = 𝑈 + 10𝑙2 𝑐𝑜𝑛 𝑙2 = 0; 1; 23𝑅 + 𝑙2 = 𝐺 𝑜 3𝑅 + 𝑙2 = 𝐺 + 10𝑙3 𝑐𝑜𝑛 𝑙3
= 0; 1; 23𝑇 + 𝑙3 = 𝐴
3𝑅 + 𝑙2 = 𝐺 𝑜 3𝑅 + 𝑙2 = 𝐺 + 10𝑙3 𝑐𝑜𝑛 𝑙3 = 0; 1; 23𝑇 + 𝑙3 = 𝐴
3𝑇 + 𝑙3 = 𝐴
De la última ecuación se desprende que 𝐴 ≥ 3 y 𝑇 ≤ 3
Puesto que 𝐴 ≠ 1, entonces 𝐸 ≠ 7.
Si 𝐸 = 5, entonces 𝐸 = 𝐴 = 5, y por tanto 𝐸 ≠ 5.
Si 𝐸 = 1, entonces 𝐴 = 3, luego 𝑇 = 𝐸 = 1, por tanto, 𝐸 ≠ 1.
Si 𝐸 = 2, entonces 𝐴 = 6, luego 3𝑇 + 𝑙3 = 6, con lo cual 𝑇 = 𝐸 = 2 y 𝑙3 = 0 por tanto 𝐸 ≠ 2.
Si 𝐸 = 4, entonces 𝐴 = 2, ya que 𝐴 ≥ 3 por tanto 𝐸 ≠ 4.
Si 𝐸 = 3, entonces 𝐴 = 0, con lo cual 𝑇 = 𝐸 = 3 por tanto 𝐸 ≠ 3.
Si 𝐸 = 8, entonces 𝐴 = 4, con lo cual 𝑈 = 𝐴 = 4 por tanto 𝐸 ≠ 8.
Si 𝐸 = 9, entonces 𝐴 = 7, con lo cual 𝑇 = 2 por tanto 3𝑇 + 𝑙3 = 7 es decir 𝑙3 = 1 𝑦 con ningún
dígito R se consigue la igualdad, y concluimos que 𝐸 ≠ 9.
Si 𝐸 = 6, entonces 𝐴 = 8, con lo cual 𝑇 = 2 y solamente con 𝑅 = 7 se consigue la igualdad, y
concluimos que 𝑇 = 2 ; 𝑅 = 7 ; 𝐴 = 8 ; 𝐸 = 6 ; 𝐺 = 3 ; 𝑈 = 5 para que 2786 × 3 = 8358.

También podría gustarte